You are on page 1of 173

GOYAL BROTHERS PRAKASHAN

To buy these books


www.goyal-books.com

also available at

Reach us goyalpublisher goyalpublisher goyalpublisher


GOYAL A DIGITAL LEARNING APP
BROTHERS
PRAKASHAN
Learn @ ` 1 Per Day

Strictly according to the latest reduced and Bifurcated syllabus and the latest sample Question Paper
released by the central Board of secondary education, New Delhi,
for term ii examination to be held in March-april, 2022

CBSE Sample Question Paper and

N
HA
Model Test Papers in

English
AS
AK
Language and Literature PR
[subject code – 184]
For Class
RS

10
HE

Term II
Exam
OT
BR

 As per the latest reduced and Bifurcated syllabus for term ii examination to be held in
March-april, 2022
 cBse sample Question Paper (Solved with Marking Scheme) for term ii examination
L

(March-april, 2022)
YA

 10 Model test Papers (solved) based on the latest cBse sample Question Paper issued
by cBse for term ii examination to be held in March-april, 2022
O

 5 Model test Papers (unsolved) based on the latest cBse sample Question Paper issued
by cBse for term ii examination to be held in March-april, 2022
G

GOYAL BROTHERS PRAKASHAN


GOYAL A DIGITAL LEARNING APP
BROTHERS
PRAKASHAN
Learn @ ` 1 Per Day

Goyal Brothers Prakashan


educational PuBlishers
Sales & Registered Office :
11/1903, Chuna Mandi, Paharganj, New Delhi - 110055
Post Box : 5720, Phones : 43551877, 43561877
E-mail : info@goyal-books.com
Website : www.goyal-books.com
Administrative Office :

N
D-231, Sector 63, Noida-201301 (U.P.)

HA
Phone : 0120-4655555
SHOWROOMS
Ahmedabad : Y-10, Dev Castle Complex Opp. Radhekrishan Villa, Jaymala-Govindwadi

AS
Road Isanpur, Ahmedabad–382 443 (Gujarat) Tel : 09925004030
Chennai : No. 50/39, Ground Floor, Five Furlolng Road, Guindy,
Chennai-600 032 (Tamil Nadu) Phones : 044-22552776, 044-22552767

AK
Guwahati : Plot No. 1179, KRB Road, Bharalumukh, near Post Office, Railway Gate
No. 8, Guwahati-781009 (Assam), Phone : 0361-2735337
Hyderabad : F-7/8/41, First Floor, Agarwal Chambers, 5-9-1121, King Kothi Road,
PR
Hyderabad - 500001, Tele-fax : 23211332
Kochi : Fair Heavens, H.No. 52-2783/A, Shine Road, Vyttila
Ernakulam - 682019 (Kerala) Phone : 0484-2305288
RS

Kolkata : No. 203, 2nd Floor, Fomra Towers, 84-A, Acharya Jagdish Chandra Bose
Road, Kolkata - 700014, Phone : 033-22262683
Bengaluru : 1681/5/4, Ground Floor, 3rd Main Ramamohanapuram,
HE

Bengaluru - 560 021, Phone : 080-23320655


Bhopal : Plot No. 203, Zone-II, M.P. Nagar, Bhopal-462 011 (M.P.),
Tele-fax : 0755-4271371
OT

Lucknow : B-319, Sector-B, Mahanagar, Opposite P.A.C Main Gate,


Lucknow - 226 006, Phone : 0522-2320841
Patna : Ravi Kiran Apartment, Flat No. 2-A, Second floor, Near Water Pumping
BR

Station, Pani Tanki Lane, Pirmohani, Patna – 800003


Nagpur : Thakkar Villa House No. 436 Joshiwadi, Sitabuldi, Nagpur - 440012,
Phone : 9823655077
L

To buy our books online log on to www.goyal-books.com


YA

© Reserved
Edition : January, 2022
Price : ` 150
O

Disclaimer
G

Due care and diligence has been taken while writing, editing and printing the book.
Neither the author nor the publishers of the book hold any responsibility for any
mistakes that may have inadvertently crept in.

Published & Printed by


Roshan Lal Goyal for Goyal Brothers Prakashan, New Delhi
GOYAL A DIGITAL LEARNING APP
BROTHERS
PRAKASHAN
Learn @ ` 1 Per Day

Reduced Syllabus for Academic Year 2021–22


ENGLisH LANGuAGE ANd LiTERATuRE
[Code No. 184]
Class 10 (2021-22)
Term-ii syllabus
Term II

N
REAdiNG

HA
Question based on the following kinds of unseen passages to assess inference, evaluation,
vocabulary, analysis and interpretation:
1. Discursive passage (400-450 words)

AS
2. Case based Factual passage (with visual input/ statistical data/ chart etc. 300-350 words)
WRiTiNG sKiLL

AK
1. Formal letter based on a given situation
Letter of Order Letter of Enquiry
Analytical Paragraph (based on outline/chart/cue/map/report etc.)
PR
GRAMMAR
1. Tenses 2. Modals 3. Subject Verb Concord
4. Determiner 5. Reported Speech 6. Commands and Requests
RS

7. Statements 8. Questions
LiTERATuRE
HE

Questions based on extracts / texts to assess interpretation, inference, extrapolation beyond the text
and across the texts.
FiRsT FLiGHT
OT

1. Glimpses of India 2. Madam Rides the Bus 3. The Sermon at Benares


4. The Proposal (Play)
PoEMs
BR

1. Amanda! 2. Animals
3. The Tale of Custard the Dragon
FooTPRiNTs WiTHouT FEET
L

1. The Making of a Scientist 2. The Necklace 3. The Hack Driver


YA

4. Bholi
EACH sEMEsTER
O

SECTION WEIGHTAGE (IN MARKS)


READING 10
G

WRITING & GRAMMAR 10


LITERATURE 20
TOTAL 40
INTERNAL ASSESSMENT 10
GRAND TOTAL 50
GOYAL A DIGITAL LEARNING APP
BROTHERS
PRAKASHAN
Learn @ ` 1 Per Day

contents

section a : cBse saMPle Question PaPer


[Released by CBSE in January 2022
for the Term II Examination to be held in March-April, 2022]

 CBSE Sample Question Paper with Answers ... ... 5 – 13

N
 Marking Scheme ... ... 14 – 30

HA
section B : Model test PaPers (solved)
Based on the Latest CBSE Sample Paper

AS
for the Term II Examination to be held in March-April, 2022
1. Model Test Paper-1 ... ... 31 – 37

AK
2. Model Test Paper-2 ... ... 38 – 44
3. Model Test Paper-3 ... ... 45 – 51
PR
4. Model Test Paper-4 ... ... 52 – 57
5. Model Test Paper-5 ... ... 58 – 64
RS

6. Model Test Paper-6 ... ... 65 – 71


7. Model Test Paper-7 ... ... 72 – 79
HE

8. Model Test Paper-8 ... ... 80 – 86


9. Model Test Paper-9 ... ... 87 – 93
OT

10. Model Test Paper-10 ... ... 94 – 100


BR

section c : Model test PaPers (unsolved)


Based on the Latest CBSE Sample Paper
for the Term II Examination to be held in March-April, 2022
L

11. Model Test Paper-11 ... ... 101 – 104


YA

12. Model Test Paper-12 ... ... 105 – 108


13. Model Test Paper-13 ... ... 109 – 112
O

14. Model Test Paper-14 ... ... 113 – 116


G

15. Model Test Paper-15 ... ... 117 – 120


GOYAL A DIGITAL LEARNING APP
BROTHERS
PRAKASHAN
Learn @ ` 1 Per Day

CBSE SAMPLE QUESTION PAPER (Solved)


(Released by CBsE for the academic year 2021-22)

Time allowed: 2 Hours Maximum Marks: 40

General Instructions:
1. The Question Paper contains THREE sections-READING, WRITING & GRAMMAR and

N
LITERATURE.
2. Attempt questions based on specific instructions for each part.

HA
sECTioN A - REAdiNG (10 marks)

AS
1. Read the passage given below.
Technology is making advancements at a rapid rate but at the cost of a

AK
valued tradition—the crafts industry. The traditional crafts industry is
losing a lot of its trained and skilled craftsmen. With that, the art of
embellishing brass and copper utensils with fine engravings is also
5
languishing craft.
PR
disappearing. The government has identified around 35 crafts as

The speciality of handcrafted items is its design, an association with long


RS
traditions belonging to a specific region. The word ‘handcrafted’ does not
imply the involvement of dexterous human fingers or an agile mind with
10 a moving spirit anymore. Lessening drudgery, increasing production and
HE

promoting efficiency have taken precedence. The labour-saving devices are


taking the place of handcrafted tools and this has jeopardized the skills of
these artisans.
OT

Mechanisation has made its way into everything - cutting, polishing,


15 edging, designing etc. Ideally, the use of machinery should be negligible
BR

and the handicrafts should be made purely by hand with a distinguishable


artistic appeal. However, with the exception of small-scale industries, the
export units are mostly operated by machines. The heavily computerised
designs contribute to a faster production at lower costs.
L

20 Although mechanization of crafts poses a challenge to safeguarding


YA

traditional crafts, the artisans are lured with incentives in order to impart
handicrafts training. Some makers do see machines as a time-saving blessing
since they are now able to accomplish difficult and demanding tasks with
O

relative ease. These machines might give a better finesse to these products
25 but they don’t stand out as handcrafted. Quantity has overtaken quality in
G

this industry.
A need to highlight the importance of the handmade aspect is required by
both the government and private sectors, in order to amplify awareness
and also support the culture of making handicrafts. A few artisans are still
30 trying their best to rejuvenate and revive their culture and heritage but it’s
an uphill task competing with the machine-made goods. A multitude of
Score Plus CBSE Sample Question Paper with MTPs in English Language & Literature for Class 10 (Term II) 5
GOYAL A DIGITAL LEARNING APP
BROTHERS
PRAKASHAN
Learn @ ` 1 Per Day

artisans have changed their professions and are encouraging their progeny
to follow suit. There are others who have stayed their ground but are clearly
inclined towards buying machines.
35 Nearly two decades ago, there were around 65 lakh artisans in the country.
Three years ago, when the government started the process of granting a
unique number to the artisans based on the Aadhaar card, 25 lakhs were
identified. Loss of traditional crafts is clearly a worrying issue, but it stands
to reason that forcing any artisan to follow old ways when concerns of

N
livelihood overrule other considerations, is unfair.

HA
Adapted from: https://www.outlookindia.com/magazine/story/business-news-
handicraft-uncrafted-by-artisan-hands/302877
Based on your understanding of the passage, answer ANY FiVE questions

AS
from the six given below. 1*5
(i) What does the writer mean by calling handicrafts a ‘valued tradition’? 1

AK
Ans. By calling handicrafts a ‘valued tradition’, the writer meant that handicrafts showcase the
talents of trained and skilled craftsmen representing the rich artistic culture and tradition.
(ii) Rewrite the following sentence by replacing the underlined phrase with a word that
PR
means the same from lines 5–15. 1
If it continues, the workcation (work + vacation) trend will be a
powerful boost to domestic tourism operators failing to make progress
RS

in the economic slump caused due to the pandemic.


Ans. If it continues, the workcation (work + vacation) trend will be a powerful boost to
HE

domestic tourism operators languishing in the economic slump caused due to the
pandemic.
(iii) State any two reasons why artisans are choosing to work via machines rather than
OT

handcrafted tools? 1
Ans. Two reasons why artisans were choosing to work via machines rather than handcrafted
BR

tools were that these tools save extra labour and reduce drudgery.
(iv) Why do the artisans need to be ‘lured with incentives’ to impart handicrafts training? 1
Ans. The artisans need to be ‘lured with incentives’ to impart handicrafts training because
L

if artisans impart training to mass/bulk producers, they risk losing their traditional
YA

livelihood and hence need to be tempted via benefits or rewards to do so.


(v) List one likely impact of the support of government and private sectors towards the
culture of making handicrafts. 1
O

Ans. One likely impact of the support of government and private sectors towards the culture
of making handicrafts is creating awareness and supporting the culture of making
G

handicrafts.
(vi) How does the writer justify an artist’s act of abandoning her/his traditional craft for a
more lucrative option? 1
Ans. The writer justifies an artist’s act of abandoning her/his traditional craft for a more
lucrative option, arguing that the artisans cannot be expected to continue their profession
if it is not profitable, even though the loss of traditional crafts is perturbing.
6 Score Plus CBSE Sample Question Paper with MTPs in English Language & Literature for Class 10 (Term II)
GOYAL A DIGITAL LEARNING APP
BROTHERS
PRAKASHAN
Learn @ ` 1 Per Day

2. Read the following excerpt from a case study titled Impacts of Festivities on Ecology.
Festivals are synonymous with celebration, ceremony and joy. However,
festivals bring to fore the flip side of celebrations – pollution – air,
water, soil and noise. This led to the need of assessing the awareness
level among people about ecological pollution during festivals. So, a
5 study was conducted by scholars of an esteemed university in India. This
study was titled Awareness Towards Impact of Festivals on Ecology.

N
There were two main objectives of the study. The first one was to assess the
awareness level among people about ecological protection during festivities.

HA
Exploring solutions to bring awareness about celebrating festivals without
10 harming ecology was the second objective. The method used to collect data
was a simple questionnaire containing 6 questions, shared with 50 respondents

AS
across four selected districts of a state in the southern region of India.
The research began by understanding the socio-economic conditions of
the respondents before sharing the questionnaire. Once the responses were

AK
15 received, the data collected was tabulated (Table 1), for analysis.
Table-1: Awareness level among respondents
QUESTIONS YES %
PR NO % CAN’T SAY%
1. Do you feel that bursting crackers 46 54 0
is a must during festivities?
RS

2. Do you think most people abuse 72 28 0


environmental resources during
celebration of festivals?
HE

3. Do you think that celebrations 64 32 4


& festivities result in uniting
people?
OT

4. Do you enjoy bursting crackers 68 32 0


for amusement?
BR

5. Do you feel pressured to burst 82 12 6


crackers during festivals as an
expectation of your social status?
6. Are you aware of waste 56 40 4
L

segregation & disposal guidelines


YA

for better ecology?

The study recommended the imposition of strict rules and regulations as


opposed to a total ban on all festive activities which have a drastic impact
O

on our environment. The researchers believed that such measures would


help in harnessing some ill-effects that add to the growing pollution and
G

20 suggested further studies be taken up across the country to assess awareness


about ecological degradation.
The observations made in the study pointed to the environmental groups
and eco-clubs fighting a losing battle due to city traffic issues, disposal
of plastics, garbage dumping and all sorts of ecological degradation. The
25 researchers stressed that the need of the hour is increasing awareness among
Score Plus CBSE Sample Question Paper with MTPs in English Language & Literature for Class 10 (Term II) 7
GOYAL A DIGITAL LEARNING APP
BROTHERS
PRAKASHAN
Learn @ ` 1 Per Day

people to reduce ecological pollution which can be facilitated by celebrating


all festivals in an eco-friendly manner.
Largely adapted from
https://www.researchgate.net/publication/325366454_Awareness_towards_Impact_of_Festivals_
on_Ecology_Need_for_Social_Work_Intervention

on the basis of your understanding of the passage, answer ANY FiVE questions
from the six given below. 1*5

N
(i) Why do the researchers call pollution the ‘flip side’ of festivals? 1
Ans. The researchers call pollution the ‘flip side’ of festivals because festivals and pollution

HA
are two sides of the same coin. We associate festivals with joy and celebration, but we
can’t deny that pollution is an ignored reality.

AS
(ii) Comment on the significance of the second objective of the study with reference to
lines 7-12. 1
Ans. The second objective of the study refers to the steps that need to be taken to address the

AK
problem and implement and focus on the actions to be corrected or avoided to solve the
problem. PR
(iii) Justify the researchers’ recommendation for limiting the drastic impact of festival
pollution on the environment with reference to lines 16-21. 1
Ans. The researchers recommended imposing strict rules and regulations to limit the impact of
RS

festival pollution as strict rules pose some restrictions but still give the needed freedom.
On the contrary, in the case of a total ban, people do not generally conform to it and
tend to revolt.
HE

(iv) Why do the researchers feel that environmental groups and eco-clubs
are fighting a losing battle in the given scenario? 1
OT

Ans. The researchers felt that environmental groups and eco-clubs are fighting a losing battle
in the given scenario because festivals cause pollution and other issues like city traffic
BR

issues, disposal of plastics, garbage dumping, etc. Therefore, spreading awareness is the
only solution to this issue.

(v) Even though a larger number of people say ‘no’ to bursting crackers than those who
L

say ‘yes’, festival pollution persists. How does evidence from table 1 support this
1
YA

statement?
Ans. Even though a fair percentage of people say ‘no’ to bursting crackers, festival pollution
persists because a large percentage of people (72%) abuse environmental resources to
O

celebrate festivals. Also, a high percentage of people (82%) use crackers to celebrate
festivals to live up to the expectation of their social status.
G

(vi) What purpose does the ‘Can’t Say’ column serve in the questionnaire (table 1)? 1
Ans. The ‘Can’t Say’ column in the questionnaire allows the respondents to choose whether to
answer or not in case they lack clarity or even if they are unwilling to respond.

8 Score Plus CBSE Sample Question Paper with MTPs in English Language & Literature for Class 10 (Term II)
GOYAL A DIGITAL LEARNING APP
BROTHERS
PRAKASHAN
Learn @ ` 1 Per Day

sECTioN B - WRiTiNG ANd GRAMMAR (10 marks)


3. Attempt ANY oNE from (i) and (ii).
(i) Study the concept chart from the self-help magazine section of a monthly publication. 5
Situation: Setback

Experience: Feeling upset

N
Healthy Processing Inability to process fully

HA
Desire to:
Feel it
Block Learning:

AS
Learn from it Block feeling
Let it go Overwhelm
Self-criticism
Self-Doubt

AK
Feel/Reflect/Learn/Align:
Observe the pattern! Can’t come to terms
Stuck in rumination
Rework beliefs on adequacy and with:
Experience listlessness
process childhood memories I can fail sometimes
Attention diverted
Assess magnitude of setback
Assess difficulty of work & likelihood
of future success by doing the work
controlled
PR
Everything can’s be

Learning takes time


from potential learning

Build grit to do the work and effort


Access healing states to experience and
RS
transform anxiety

Unacceptance of
Situations Triggers:
Apathy
HE

Inadequacy
Incompetency Anger
Danger and Threat Anxiety
Blaming others Depression
OT

Unconscious/Implicit memories:
Habitual process blocks
BR

Scary early life experiences of being


reprimanded for not meeting
expectations
L

Write a paragraph in not more than 120 words, analysing the listed responses
to the situation when one faces setbacks.
YA

FoR THE VisuALLY iMPAiREd CANdidATEs


Read the following excerpt from an article that appeared in a monthly
O

newsletter circulated among residents of a township.


The by-laws of some residential associations and management that ban
G

owners and tenants from keeping pets in their apartments, are justified.
Write a paragraph to analyse the given argument.
You could think about what alternative explanations might weaken the given
conclusion and include rationale / evidence that would strengthen / counter
the given argument.

Score Plus CBSE Sample Question Paper with MTPs in English Language & Literature for Class 10 (Term II) 9
GOYAL A DIGITAL LEARNING APP
BROTHERS
PRAKASHAN
Learn @ ` 1 Per Day

Ans. Setbacks often leave one with a feeling of disappointment and sometimes even worse. The
given concept chart displays two ways to handle setbacks–healthy or unhealthy processing.
Healthy processing helps one learn from and let go of the emotions inside. Contrarily, the
inability to process healthily leads to blocked learning, self-criticism, and self-doubt. An
optimistic way of seeing a setback processes the ability to feel, reflect, learn and align
childhood memories, build grit to work, etc. On the contrary, negative perspective can lead to
listlessness, diverted attention, excessive rumination, and inability to accept failures that, in
turn, trigger a feeling of anger, anxiety, depression, and apathy. It is evident that experiencing

N
a setback leads to feelings of inadequacy and incompetency and an unwillingness to accept
situations. Responding positively to setbacks leads to success.

HA
(ii) You are Samina Zaveri, Class X, Vadodara, Gujarat. You come across the following
information on a local library’s notice board. 5
Create Your own Board Game Competition!

AS
Create an educational board game, and send it to us at Teen-Toggle
Games Pvt. Ltd, 307, Satija building, Colaba, Mumbai by July 2022. The

AK
top 10 winning board games will be featured on our international portal.
Attractive scholarships for the winners!
PR
You wish to participate but require more information. Write a letter to
Teen-Toggle Games Pvt.Ltd in about 120 words, enquiring about rules,
scholarship details and deadlines. Also enquire about specifications for solo
RS

or group entries.
Ans. 23, XYZ Colony
HE

Vadodara, Gujarat

1 June 2022
OT

The Director
Teen-Toggle Games Pvt. Ltd.
BR

307, Satija Building


Colaba, Mumbai

Sir
L

Subject: 'Create Your Own Board Game Competition!' enquiry


YA

This is regarding the participation in 'Create Your Own Board Game Competition!'. I
have seen your advertisement on a local library's noticeboard and realised that it could be
O

a golden opportunity to enter the international arena. I belonged to a middle-class family,


and this scholarship would really be helpful for my career prospects. I have already
G

participated in a game creating competition in which I had bagged the second position. I
am keen to participate in the competition. Kindly furnish me with the following details:
1. Rules
2. Scholarship details
3. Deadlines
Also, please provide me with the information about the solo or group entries.

10 Score Plus CBSE Sample Question Paper with MTPs in English Language & Literature for Class 10 (Term II)
GOYAL A DIGITAL LEARNING APP
BROTHERS
PRAKASHAN
Learn @ ` 1 Per Day

Yours truly
Samina Zaveri
Class X
4. The following paragraph has not been edited. There is one error in each line. Identify the
error and write its correction against the correct blank number. Remember to underline the
correction. The first one has been done for you. 1*3

N
Error Correction
E.g. learn

HA
Have you ever learn from a mistake you have made? learnt
Many shouldn’t admit doing so. For those who do, a)
there was no need for guilt. We often make mistakes b)

AS
while taking risks, but all brush them aside and learn. c)
With that, they usually don’t make mistakes the next time.
Ans. (a) won't/don't

AK
(b) is
(c) some/many
PR
5. Read the conversation between a teacher and student and complete the passage that
follows. 1*2

Biology Teacher: I instructed you to draw the diagram of bacteria.


RS

Why did you submit a blank sheet?


sameer: Sir, I had drawn the diagram of bacteria, but you can’t see it
HE

because it is not visible to the naked eye.

The biology teacher had instructed Sameer to draw the diagram of a bacterial
OT

cell and asked him (a) …………………………………………… a blank


sheet. Sameer respectfully answered that he had drawn the diagram but
BR

(b) ……………………………………………………to the naked eye.


Ans. (a) why he had submitted
(b) he/the teacher couldn’t see it because it is not visible
L

sECTioN C - LiTERATuRE (20 marks)


YA

6. Answer ANY siX questions in 30-40 words each. 2*6


(i) What is the significance of the Buddha’s request for a handful of mustard seeds and the
O

addition of a condition to it? 2


Ans. To show that death is universal, Buddha instructed Kisa Gautami to bring a handful of
G

mustard seeds from a house where no one had ever died. Accordingly, she could not find
any house where nobody had died. Buddha wanted her to see the truth of life and death
and accept the destiny of human beings.
(ii) Justify how ‘Animals’ by Walt Whitman is a criticism of mankind and its ways? 2
Ans. Walt Whitman feels that animals are better than humans because they do not make
the poet sick by discussing their duties to God. Human society is blemished by ego,
Score Plus CBSE Sample Question Paper with MTPs in English Language & Literature for Class 10 (Term II) 11
GOYAL A DIGITAL LEARNING APP
BROTHERS
PRAKASHAN
Learn @ ` 1 Per Day

hypocrisy, hatred, materialism, fake display. Unlike humans, animals do not whine about
their condition or regret any past sins.
(iii) Comment on the tone of the speaker when she says ‘Will you please look at me when
I’m speaking to you, Amanda!’. 2
Ans. The speaker wants Amanda to look at him when he is addressing her. But Amanda was
not listening to the speaker and chose not to respond. So, the speaker was annoyed and
angry as her daughter was not listening to her.
(iv) A ballad includes the telling of a tale as well as a surprise ending. Using evidence from

N
the poem, explain how these features are included in ‘The Tale of Custard the Dragon’. 2
Ans. The poem tells the story of Custard, the dragon. It includes the setting, characters, rising

HA
action, climax, and resolution. The poem portrays Custard’s life with Belinda and the
other pets, where he is considered a coward. Also, there is a surprise ending, showing
how Custard rose to the occasion, gobbled the pirate and proved his bravery.

AS
(v) Which two issues about himself convinced Lomov of his decision to get married? 2
Ans. Lomov wanted to marry Natalya because he was suffering due to a weak heart and sleep-

AK
sickness. He wanted a companion who could look after him. Secondly, he expected to marry
and couldn’t stay unmarried. He ought to lead a quiet, settled and regular life at his age.
(vi) Briefly state how Matilda invited ‘a dreadful life of necessity’ into her family. 2
PR
Ans. Matilda’s downfall was caused by her unhappiness with whatever life offered her. She
was always in a bad mood and felt as if she had been born for all the luxuries and
delicacies. She had lost her friend’s necklace and her life had become miserable. In
RS

paying for the necklace, she invited a horrible life of necessity and deprivation.
(vii) The hack driver successfully trapped the narrator in his web of words. Comment. 2
HE

Ans. The hack driver was a conversationalist, cheerful, and friendly. He befriended the
narrator, who trusted him for everything he said. He asked the narrator to stay behind,
made inquiries, offered homemade lunch and minted money. He befooled and outwitted
OT

the narrator in the guise of helping.


7. Answer ANY TWo of the following in about 120 words each. 4*2
BR

(i) Parents play a crucial role in the upbringing of their children. Critically examine the
parents of Bholi and Ebright, highlighting their impact on their children’s lives. 4
Ans. Bholi’s parents were quite ordinary. Her father, Ramlal, was the Numberdar of his
village. He was quite prosperous; he was quite traditional in his outlook. He was quite
L

much worried about Bholi because of her looks and stuttering. Ramlal had to send Bholi
YA

to school because of pressure from Tehsildar.


Ramlal’s wife was very backward in thinking; she believed that if the girls were allowed
to go to school, nobody would marry them.
O

Bholi's parents were uncaring, indifferent, biased, insensitive, and patriarchal. They did not
G

believe in girls' education, were indifferent to Bholi and her needs, and neglected her.
Bholi needed special care, but neither of her parents did that; they just wanted to get her
married. Despite their prosperity, they left Bholi to her own misery, worst was when they
decided to marry her off to Bishamber.
Ebright's mother was crucial in his development as a scientist. She encouraged him to
broaden his horizons by teaching him new skills. She took him on trips and purchased
telescopes, microscopes, cameras, mounting materials, and other items for him.
12 Score Plus CBSE Sample Question Paper with MTPs in English Language & Literature for Class 10 (Term II)
GOYAL A DIGITAL LEARNING APP
BROTHERS
PRAKASHAN
Learn @ ` 1 Per Day

Every evening, around the dinner table, mother and son communicated. If Ebright had
nothing to do, she would find his work—learning work.
Ebright's mother also sent him a book called ‘The Travels of Monarch X’, which introduced
him to the realm of science. She encouraged him to pursue a career in science by nurturing
his enthusiasm for it.
(ii) Pranjol and Rajvir discuss their next vacation destination. They shortlist Coorg and Goa.
Rajvir is keen on Coorg and tries to convince Pranjol. Develop a conversation between
the two, based on your understanding of Glimpses of India. 4

N
You may begin like this:

HA
Rajvir: Hey Pranjol! I think we should be visiting Coorg. It is a beautiful place with
coffee plantations. I can smell the aroma already!
Pranjol: We visited a tea plantation last year, in Assam; I want to…

AS
Ans. Rajvir: Hey Pranjol! I think we should be visiting Coorg. It is a beautiful place with
coffee plantations. I can smell the aroma already!
Pranjol: We visited a tea plantation last year, in Assam; I want to visit Goa this time.

AK
Rajvir: There is a lot more to do in Coorg than smelling the coffee! The place has
rainforests, so the mega fauna will be worth watching. Not just this, Coorg provides
PR
opportunities to indulge in adventure sports like river rafting, rappelling, mountain biking,
to name a few.
Pranjol: That sounds interesting, but I would prefer some serene moments too, away
from this post-pandemic hustle-bustle.
RS

Rajvir: Oh! The answer is Coorg again!


Pranjol: Oh, come on! You can’t be serious.
HE

Rajvir: Believe me, I am. Coorg has beautiful natural walking trails, and the Brahmagiri
hills offer a panoramic view. I read that the place has the largest Tibetan settlement, so
OT

the environment will reflect peace and spirituality, I’m sure.


Pranjol: Have to say, you’ve presented a fine case in favour of Coorg and convinced
me. Let’s plan to leave for Coorg next Wednesday!
BR

(iii) Farce is a kind of comedy which includes situations and dialogues that are ridiculous,
exaggerated and even absurd. Evaluate the play, The Proposal, as a farce. 4
Ans. For most people in Russia in the past, marriage was a source of financial security. They
L

married for the sake of accumulating wealth and goods or to appease social pressure. In
YA

the play, the sarcasm is skillfully communicated by stressing the couple's silly disputes
about little issues. The primary points of contention in the play are 'The Oxen Meadows'
and 'the two dogs', Guess and Squeezer.
O

We can detect farcical characteristics as we read the play. Lomov's attire and demeanour
G

are both amusing. When Chubukov and Natalya quarrel with him, we can see how he
reacts. On the other hand, Natalya can be described as a quarrel-loving, dominating young
lady. Chubukov is a wise man, but when he sees Natalya and Lomov fighting, he joins in
and supports his daughter. There are several comical scenarios. According to the concept
of farce, it consists solely of story development rather than character development, and it
skillfully produces comedic circumstances rather than any change in character behaviour.
All of the characters are realistic; however, their emotions are often exaggerated.

Score Plus CBSE Sample Question Paper with MTPs in English Language & Literature for Class 10 (Term II) 13
GOYAL A DIGITAL LEARNING APP
BROTHERS
PRAKASHAN
Learn @ ` 1 Per Day

N
HA
AS
AK
PR
RS
HE
OT
BR
L
YA
O
G

27
BROTHERS
PRAKASHAN
GOYAL A DIGITAL
Learn @LEARNING
` 1 Per DayAPP
BROTHERS
PRAKASHAN
Learn @ ` 1 Per Day

N N
HA HA
AS AS
AK AK
PR PR
RS RS
HE HE
OT OT
BR BR
L L
YA YA
O O
G G

To view Sample or Purchase of Books


WWW.goyal-books.com/mtpcx
To view Sample or Purchase of Books
7R YLHZ 6DPSOH RU 3XUFKDVH RI %RRNV 8
www.goyal-books.com/mtpcx
To view Sample or Purchase of Books
:::JR\DOERRNVFRPPWSF[
www.goyal-books.com/mtpcx
28

To view Sample or Purchase of Books
G

WWW.goyal-books.com/mtpcx
To view Sample or Purchase of Books
7R YLHZ 6DPSOH RU 3XUFKDVH RI %RRNV 8
www.goyal-books.com/mtpcx
To view Sample or Purchase of Books
:::JR\DOERRNVFRPPWSF[
www.goyal-books.com/mtpcx

28
GOYAL A DIGITAL LEARNING APP
BROTHERS
PRAKASHAN
Learn @ ` 1 Per Day

MARKING SCHEME
(Released by CBsE for the academic year 2021-22)

Time allowed: 2 Hours Maximum Marks: 40

READING (10 MARKS)


Note:

N
(i) Since the Reading Section focuses on testing a candidate’s ability to comprehend, no deductions
are to be made for errors in spelling, grammar or punctuation.

HA
(ii) Marks should be awarded only if the answer reveals formation of a response to the question.
(iii) No marks to be awarded if a chunk/exact line/s is/are transcribed from the passage without
evidence of structure or semblance of coherent thought, in an attempt to pass off as a response.

AS
objective: This section evaluates the reading and comprehension skills of the students and their
ability to analyse, infer (information/meanings) and evaluate the given information.

AK
1. uNsEEN PAssAGE 1*5=5
(ANY 5 ouT oF 6)
PR
(i) What does the writer mean by calling handicrafts a ‘valued tradition’? (1 mark)

Ans. Value Points Guidance


RS

• valued – showcase talents that are The learner is required to respond to—
associated with artisans’ lifestyle and Why are handicrafts valued? Why are they
history
HE

considered a part of our tradition?


• tradition – represent rich our artistic ▪ Award maximum 1 mark for the complete
culture / heritage / tradition correct answer
OT

▪ Award partial credit of ½ mark if just


‘valued’/ ‘tradition’ is addressed in the
response.
BR

(ii) Rewrite the following sentence by replacing the underlined phrase with a word that
means the same, from lines 5– 15. (1 mark)
L

If it continues, the workcation (work + vacation) trend will be a powerful boost to


YA

domestic tourism operators failing to make progress in the economic slump caused due
to the pandemic.

Ans. VALuE PoiNTs GuidANCE


O

If it continues, the work cation (work + ▪ 1 mark for the correct answer
G

vacation) trend will be a powerful boost ▪ No partial credit


to domestic tourism operators languishing
in the economic slump caused due to the ▪ No marks to be deducted if the learner
pandemic. fails to rewrite the complete sentence
and just lists the chosen meaning—the
purpose of the given sentence is to share
a context.

14 Score Plus CBSE Sample Question Paper with MTPs in English Language & Literature for Class 10 (Term II)
GOYAL A DIGITAL LEARNING APP
BROTHERS
PRAKASHAN
Learn @ ` 1 Per Day

(iii) State any two reasons why artisans are choosing to work via machines rather than
handcrafted tools? (1 mark)

Ans. VALuE PoiNTs GuidANCE


Saves labour / reduces drudgery / increases The learner is required to state 2 clear points
production / finesse / efficiency that indicate the advantage of using machines
over working with hands and tools
▪ Award 1 mark for any two points
(½ + ½)

N
▪ ½ mark as partial credit for one point

HA
stated
(iv) Why do the artisans need to be ‘lured with incentives’ to impart handicrafts training?(1 mark)

AS
Ans. VALuE PoiNTs GuidANCE
Because if artisans impart training to mass/ The learner is required to rationalise why the

AK
bulk producers, they run the risk of losing bulk producers have to tempt/ attract/ entice
their traditional livelihood to them and artisans to share their craft secrets/ training
hence need to be tempted via benefits or with them.
rewards, to do so. PR ▪ Award 1 mark for complete rationalisation.
▪ ½ mark for partial explanation–just
elaborating on either the reason or the
RS

effect.
(v) List one likely impact of the support of government and private sectors towards the
HE

culture of making handicrafts. (1 mark)

Ans. VALuE PoiNTs GuidANCE


OT

Creation of awareness and public support The learner is required to list an advantage
for the artisans and their work /more that is the outcome of the support of
artisans would be incentivized to continue government and private sectors towards
BR

their tradition /artisans wouldn’t change handicraft making.


their professions seeking better livelihood ▪ Award 1 mark for any one impact
▪ No partial credit
L

(vi) How does the writer justify an artist’s act of abandoning her/his traditional craft for a
YA

more lucrative option? (1 mark)

Ans. VALuE PoiNTs GuidANCE


O

Via the argument that the artisans cannot The learner is required to state how the writer
be expected to continue their profession if agrees that an artisan is justified in moving
G

it is not profitable, even though the loss of to options that pay better than traditional
traditional crafts is perturbing. handicraft making, in order to support his
livelihood.
▪ Award 1 mark for the correct answer.
▪ No partial credit
▪ No marks for lines just copied from the text

Score Plus CBSE Sample Question Paper with MTPs in English Language & Literature for Class 10 (Term II) 15
GOYAL A DIGITAL LEARNING APP
BROTHERS
PRAKASHAN
Learn @ ` 1 Per Day

2. uNsEEN PAssAGE 1*5=5


(ANY 5 ouT oF 6)
(i) Why do the researchers call pollution the ‘flip side’ of festivals? (1 mark)

Ans. VALuE PoiNTs GuidANCE


▪ because the accepted norm is that festivals The learner is required to explain how
are synonymous with celebration/joy and pollution is the darker side of festivals and
people fail to see the other side, which is unfortunately goes hand-in-hand, often, with

N
pollution festivities
▪ Award maximum 1 mark for the complete

HA
OR
correct answer.
▪ festivals and pollution are two sides of
▪ There is no partial credit.
the same coin currently/ polar opposites/

AS
completely in contrast, yet together with ▪ Accept any other similar complete
each other—where we associate festivals interpretation, with reference to the given
passage that convey the reason why

AK
with joy and celebration, we can’t deny
that pollution is an ignored reality pollution is the flip side of festivals.

(ii) Comment on the significance of the second objective of the study with reference to lines
PR
7-12. (1 mark)

Ans. VALuE PoiNTs GuidANCE


▪ Award maximum 1 mark for the mention
RS

Second objective-Exploring solutions


Significance- important for knowing steps of any one valid significance.
that need to be taken to address the problem ▪ There is no partial credit.
HE

investigated in the research/ knowing what


are the immediate actions that need to be
implemented to address the issue studied/
OT

helps focus on what needs to be corrected


& what needs to be avoided to solve the
problem focussed on in the research
BR

(iii) Justify the researchers’ recommendation for limiting the drastic impact of festival
pollution on the environment with reference to lines 16-21. (1 mark)
L

Ans. VALuE PoiNTs GuidANCE


YA

▪ Researcher’s recommendation: Strict The learner is required to state the


rules recommendation and then defend it against
▪ Justification: Strict rules are better than a the other stated alternative.
O

total ban because banning does not serve ▪ Award maximum 1 mark for the complete
G

the purpose of awareness/ people do not correct answer.


generally conform to banning and tend to ▪ Award partial credit of ½ mark if just the
revolt/ strict rules pose some restrictions recommendation is listed or ‘strict rules’
but still give the needed freedom. is the response, without substantiation.

16 Score Plus CBSE Sample Question Paper with MTPs in English Language & Literature for Class 10 (Term II)
GOYAL A DIGITAL LEARNING APP
BROTHERS
PRAKASHAN
Learn @ ` 1 Per Day

(iv) Why do the researchers feel that environmental groups and eco-clubs are fighting a
losing battle, in the given scenario? (1 mark)

Ans. VALuE PoiNTs GuidANCE


▪ the festivals cause pollution along with The learner is required to rationalise why
other issues (give examples) that add to environmental groups and eco-clubs aren’t
it. succeeding in their purpose.
▪ awareness is the only solution but the ▪ Award maximum 1 mark for the complete

N
lack of it only adds to the problem. correct answer including both situation
and the reason

HA
▪ Award partial credit of ½ mark if just
either aspect is listed.

AS
(v) Even though a fair percentage of people say ‘no’ to bursting crackers, festival pollution
persists. How does evidence from table 1 support this statement? (1 mark)

AK
Ans. VALuE PoiNTs GuidANCE
▪ a large percentage of people (72%) abuse The learner is required to study table 1 and
environmental resources to celebrate ▪ choose data that indicates what number
PR
festivals percentage of people partake in activities
▪ a high percentage of people (82%) use that add to pollution.
crackers to celebrate festivals in order to ▪ Then, check to see if the number is higher
RS

live up to the expectation of their social than the number about ‘no to bursting
status crackers’
HE

▪ Finally rationalise the data to prove the


Q statement, by using the evidence data.
▪ Award maximum 1 mark for the mention
OT

of any one point—identification + listing


of percentage
▪ Award partial credit of ½ mark if the
BR

questionnaire point is listed without the


percentage/ percentage is listed without
the mention of the questionnaire point.
L

(vi) What purpose does the ‘Can’t Say’ column serve in the questionnaire (table 1)? (1 mark)
YA

Ans. VALuE PoiNTs GuidANCE


▪ gives a provision to/ allows the The learner is required to explain the role
O

respondents to choose not to express/ not of the “can’t say’ section, with reference to
to answer/ allows an option to those who study table 1.
G

lack clarity/ are unwilling to respond ▪ Award maximum 1 mark for the complete
correct answer-- any one point.
▪ No partial credit

Score Plus CBSE Sample Question Paper with MTPs in English Language & Literature for Class 10 (Term II) 17
GOYAL A DIGITAL LEARNING APP
BROTHERS
PRAKASHAN
Learn @ ` 1 Per Day

sECTioN B - WRiTiNG ANd GRAMMAR (10 MARKs)


3. Attempt ANY oNE from (i) and (ii). 5*1
(i) ANALYTiCAL PARAGRAPH WRiTiNG
Study the concept chart from the self-help magazine section of a monthly publication.
Situation: Setback

N
Experience: Feeling upset

HA
Healthy Processing Inability to process fully

AS
Desire to:
Feel it
Block Learning:

AK
Learn from it Block feeling
Let it go Overwhelm
Self-criticism
PR Self-Doubt

Feel/Reflect/Learn/Align:
Observe the pattern! Can’t come to terms
Stuck in rumination
Rework beliefs on adequacy and with:
Experience listlessness
process childhood memories I can fail sometimes
Attention diverted
RS

Assess magnitude of setback Everything can’s be


from potential learning
Assess difficulty of work & likelihood controlled
of future success by doing the work Learning takes time
Build grit to do the work and effort
HE

Access healing states to experience and


transform anxiety

Unacceptance of
OT

Situations Triggers:
Inadequacy Apathy
Incompetency Anger
Danger and Threat Anxiety
BR

Blaming others Depression

Unconscious/Implicit memories:
L

Habitual process blocks


Scary early life experiences of being
YA

reprimanded for not meeting


expectations
O

Write a paragraph in not more than 120 words, analysing the listed responses to setbacks.
The question tests the following writing Los:
G

▪ convey ideas convincingly using appropriate language


▪ organize the content and structure the ideas logically, sequentially, cohesively
▪ use a range of vocabulary and sentence structure appropriate to the content and context
▪ use of functional language to show comparison, contrast, emphasis, conclusion etc.

18 Score Plus CBSE Sample Question Paper with MTPs in English Language & Literature for Class 10 (Term II)
GOYAL A DIGITAL LEARNING APP
BROTHERS
PRAKASHAN
Learn @ ` 1 Per Day

GuidANCE
Award 3 marks for content—
▪ Topic sentence identifying the two responses to setbacks in the concept map – ½ mark
For Instance:
Setbacks often leave one with a feeling of disappointment and sometimes even worse. / The
concept chart given below displays two ways to handle setbacks, for the better or for the worse/
the two ways of processing setbacks – healthy and unhealthy.

N
▪ Any 2 points of contrast/ comparison with evidence – 2 marks

HA
For instance:
Healthy processing helps one learn from and let go the emotions inside. On the contrary, inability

AS
to process leads to a block, self-criticism and self-doubt.
Or

AK
An optimistic way of seeing a setback processes the ability to feel, reflect, learn and align. On
the other hand, negative perspective of a failure can lead to listlessness, diverted attention,
excessive rumination and unacceptance of failures.
PR
▪ Concluding sentence, tied to the content of the topic sentence, showcasing a perspective/
rationalising the importance of healthy processing of setbacks encountered. – ½ mark
For instance, one might want to point to the fact that the responding to setbacks negatively leads
RS

to a vicious cycle of undesirable feelings):


It is evident that experiencing a setback leads to feelings of inadequacy and incompetency along
HE

with an unwillingness to accept situations. Responding to setbacks the positive way is empowering
as it leads to success.
Note- Just listing concept map matter without evidence of analysis carries no credit.
OT

Award 2 marks for organisation & expression –


▪ ½ mark—
BR

Inclusion of a single paragraph organisation with a suitable topic sentence supporting


sentences and a suitable concluding sentence.
No requirement of a title (because the purpose is analysis, not publication). No penalty if title
L

is written.
YA

▪ 1 mark—
use of appropriate functional language to show comparison/contrast & emphasis:
Comparison/ Contrast: in contrast with, in comparison to, on the contrary, however, whereas, as
O

opposed to, while, a striking difference, a noticeable difference, despite etc.


G

Emphasis: in other words, /especially/ specifically/ to emphasise/ to demonstrate/such as/in


particular etc.
full credit 1 mark to be allotted if the functional language has been used consistently
partial credit ½ mark to be allotted if the functional language has been used occasionally/
sparingly
No credit of marks if functional language is missing (not used at all)
Score Plus CBSE Sample Question Paper with MTPs in English Language & Literature for Class 10 (Term II) 19
GOYAL A DIGITAL LEARNING APP
BROTHERS
PRAKASHAN
Learn @ ` 1 Per Day

FoR THE VisuALLY iMPAiREd CANdidATEs


The by-laws of some residential associations and management that ban owners and
tenants from keeping pets in their apartments, are justified.
Write a paragraph to analyse the given argument.
You could think about what alternative explanations might weaken the given conclusion
and include rationale / evidence that would strengthen / counter the given argument.

N
use of appropriate functional language to agree or disagree with the premise

HA
Agree: completely agree as it clearly points…. / Offers credible arguments to
support…/ …is difficult to contradict/undeniably/ hard to disagree etc.
Disagree: Unlike…what matters is…/even though …claims that…, it stands to reason

AS
that…/fails to convince/ is overstated/ this opinion is contentious etc.
Justify: Answer the ‘why’? and use words like – because, since, due to, as a
consequence of, based on and etc.

AK
full credit 1 mark to be allotted if the functional language has been used
consistently
PR
partial credit ½ mark to be allotted if the functional language has been used
occasionally/sparingly
No credit of marks if functional language is missing (not used at all)
RS

▪ ½ mark—
HE

Unity of ideas in the complete paragraph with ideas arranged logically –sentences
within paragraph follow expected organizational frameworks*
OT

*[Categorical – in order of importance; Evaluative - a problem is introduced, and the pros and
cons are weighed; Comparative – similarities and differences; Cause and Effect; Descriptions-
from general to specific attributes]
BR

Accuracy-
Deduct from the overall score if the error density is high as this impacts the communicative
function.
L

½ mark for a total of 2-3 spelling and grammatical errors


YA

1 mark for a total of more than 3 spelling and grammatical errors

(ii) Letter of Enquiry 5*1


O

You are Samina Zaveri, Class X, Vadodara, Gujarat. You come across the following information
G

on a local library’s notice board.


Create Your own Board Game Competition!
Create an educational board game, and send it to us at Teen-Toggle
Games Pvt. Ltd, 307, Satija building, Colaba, Mumbai by July 2022.
The top 10 winning board games will be featured on our international portal.
Attractive scholarships for the winners!

20 Score Plus CBSE Sample Question Paper with MTPs in English Language & Literature for Class 10 (Term II)
GOYAL A DIGITAL LEARNING APP
BROTHERS
PRAKASHAN
Learn @ ` 1 Per Day

You wish to participate but require more information. Write a letter to Teen-Toggle Games Pvt.
Ltd in about 120 words, enquiring about rules, scholarship details and deadlines. Also enquire
about specifications for solo or group entries.
Content -2 Expression-2 Accuracy -1
Value points- Content
• Reference to the source of information
• Conveying interest

N
• Seeking information about rules

HA
• Enquiring about scholarship details
• Asking about defined articles
• Confirm type of entries allowed

AS
Any other relevant information
descriptors for Content

AK
NoTE-Dedicated marks at a level are to be awarded only if ALL descriptors match. If one or
more descriptors do not match, the marks are awarded at a level lower.
2 marks
PR
All points included
RS
Well-developed with sustained clarity
1½ marks
HE

Almost all points incorporated


Reasonably well-developed
OT

1 mark
Some points incorporated
BR

Fair attempt at developing ideas with some impact on clarity of response


½ mark
Most of the points of the given task not incorporated
L

Limited awareness of task development


YA

Expression -2 marks
Marks descriptors for Expression
O

NoTE-Dedicated marks at a level are to be awarded only if ALL descriptors match. If one or
G

more descriptors do not match, the marks are awarded at a level lower.
2 • Highly effective style capable of conveying the ideas convincingly with appropriate layout
of a formal letter viz. addresses, salutation, subscription, and ending.
• Carefully structured content with organised paragraphing presented cohesively.
• Highly effective register (formal tone and vocabulary), relevant and appropriate sentences
for conveying the ideas precisely and effectively.

Score Plus CBSE Sample Question Paper with MTPs in English Language & Literature for Class 10 (Term II) 21
GOYAL A DIGITAL LEARNING APP
BROTHERS
PRAKASHAN
Learn @ ` 1 Per Day

1½ • Frequent clarity of expression most of the times, layout of a formal letter largely
accurate.
• Ideas generally well sequenced and related to the given topic maintaining overall
cohesion of ideas.
• Range of vocabulary is mostly relevant and conveys the overall meaning and the purpose
of the writing.
1 • Inconsistent style, expression sometimes awkward, layout of a formal letter basically

N
accurate.
• Sequencing of ideas is somewhat clear and related to the given topic attempting to

HA
maintain a general overall cohesion.
• Range of vocabulary is limited but manages to convey the overall meaning and the
purpose of the writing.

AS
½ • Expression unclear, layout partially followed affecting the format of the letter.
• Poor sequencing of ideas but ideas are related to the given topic in a disjointed manner

AK
exhibiting a lack of coherence of ideas.
• Very limited vocabulary or copying from the question.
PR
Accuracy -1 mark
descriptors for Accuracy
1 mark
RS

Spelling, punctuation and grammar consistently/largely accurate, with occasional minor errors,
that do not impede communication.
HE

½ mark
Spelling, punctuation and grammar display some errors spread across, causing minor impediments
OT

to the message communicated.


No credit
BR

Frequent errors in spelling, punctuation and grammar, impeding communication.

3. EdiTiNG 1*3 = 3
L

Q. The following paragraph has not been edited. There is one error in each line. Identify the error
YA

and write its correction against the correct blank number. Remember to underline the correction.
The first one has been done for you.
Error Correction
O

Have you ever learn from a mistake you have made? E.g. learn learnt
G

Many shouldn’t admit doing so. For those who do, a)


there was no need for guilt. We often make mistakes b)
while taking risks, but all brush them aside and learn. c)
With that, they may not make mistakes the next time.

22 Score Plus CBSE Sample Question Paper with MTPs in English Language & Literature for Class 10 (Term II)
GOYAL A DIGITAL LEARNING APP
BROTHERS
PRAKASHAN
Learn @ ` 1 Per Day

ANsWERs
Error Correction
Have you ever learn from a mistake you have made? E.g. learn learnt
Many shouldn’t admit doing so. For those who do, a) shouldn’t won’t/don’t
there was no need for guilt. We often make mistakes b) was is
while taking risks, but all brush them aside and learn. c) all some/many

N
With that, they may not make mistakes the next time.

HA
GuidANCE
Award 1 mark for each correct answer

AS
▪ ½ mark for identification of the error
▪ ½ mark for the writing of the correction

AK
Editing is incomplete if either aspect (identification or rectification) is missing.

4. PAssAGE CoMPLETioN - REPoRTEd sPEECH 1*2


PR
Read the conversation between a teacher and student and complete the passage that follows.

Biology Teacher: I instructed you to draw the diagram of bacteria. Why did you
RS
submit a blank sheet?
sameer: Sir, I had drawn the diagram of bacteria, but you can’t see it because it is
not visible to the naked eye.
HE

The biology teacher had instructed Sameer to draw the diagram of bacteria
and asked him (a) ……………………………………………………. a blank
OT

sheet. Sameer respectfully answered (b) that he had drawn the diagram but
…………………………………………………………………………to the naked
eye.
BR

ANsWER GuidANCE
(a) why he had submitted Award 1 + 1 mark for each correct answer.
No partial credit.
L

(b) he/ the teacher couldn’t see it because it direct speech indirect speech
YA

is not visible (a) simple past tense → Past perfect tense


(b) Past perfect tense → No change
O

(c) fact/universal truth → No change

sECTioN C - LiTERATuRE (20 MARKs)


G

5. sHoRT QuEsTioNs 2*6 = 12


(ANY 6 oF 7)
objective: This section evaluates the questions based on texts to assess interpretation, inference,
extrapolation beyond the text and across the texts.
Content -1 mark

Score Plus CBSE Sample Question Paper with MTPs in English Language & Literature for Class 10 (Term II) 23
GOYAL A DIGITAL LEARNING APP
BROTHERS
PRAKASHAN
Learn @ ` 1 Per Day

• Value points based on Q asked.


Expression- 1 mark
• Answer organised effectively/ logically (instead of a careless group of sentences strung
loosely together)
• Use of required functional language/ expressions
No marks deducted for exceeding word limit

N
(i) What is the significance of the Buddha’s request for a handful of mustard seeds and the addition
of a condition to it? (2 marks)

HA
VALuE PoiNTs GuidANCE
▪ mustard seeds easy to procure/ available in The question indicates that the examiner is
every household she knocked at familiar with the Buddha’s request and hence

AS
▪ added condition – must be procured form that requires no elaboration.
a house where no one had lost a child, The question requires an answer to:

AK
husband, parent or friend Why - mustard seeds with an added condition?
▪ made Kisa Gotami realize the universal What did the Buddha want Kisa Gotami to
nature of death realize?
PR
Content -
Award 1 mark for full explanation of the two
RS

strands.
Award ½ mark for partial explanation.
HE

Expression –
1 mark when both aspects included
OT

Answer organised effectively


usage of words supporting the reasoning/
expression of significance - since, because,
BR

therefore, so that etc.


½ mark when either aspect is missing
Deduct ½ mark from the overall score if the
L

error density is high (more than a total of 2


YA

spellings and grammatical errors).

(ii) Justify how ‘Animals’ by Walt Whitman is a criticism of mankind and its ways? (2 marks)
O

VALuE PoiNTs GuidANCE


▪ human beings seem to have dropped/ shed The question indicates that the examiner
G

their values/virtues/attributes knows that the poet appreciates the animals


▪ human society blemished by ego, hypocrisy, for the display of virtues and hence that
hatred, materialism, fake display requires no elaboration.

▪ profusion of complaints, race for greed and The question requires an explanation of
lack of contentment criticism of mankind and evidence to justify
the same.

24 Score Plus CBSE Sample Question Paper with MTPs in English Language & Literature for Class 10 (Term II)
GOYAL A DIGITAL LEARNING APP
BROTHERS
PRAKASHAN
Learn @ ` 1 Per Day

▪ the poet wishes to leave the human world Content -


and join animals as he finds them better Award 1 mark for full explanation of the two
than humans (a testimony) strands-criticism & evidence
Award ½ mark for partial explanation.
Expression –
1 mark when both aspects included
Answer organised effectively

N
usage of words supporting explanation &

HA
justification (that’s why, because, therefore
etc.)
½ mark when either aspect is missing

AS
Deduct ½ mark from the overall score if the
error density is high (more than a total of 2

AK
spellings and grammatical errors).

(iii) Comment on the tone of the speaker when she says ‘Will you please look at me when I’m
speaking to you, Amanda!’. PR (2 marks)

VALuE PoiNTs GuidANCE


Tone The question indicates that the examiner
RS

▪ exasperated knows that Amanda is being questioned and


she chooses not to respond and hence that
▪ irritated requires no elaboration.
HE

▪ frustrated The question needs an answer to the tone


▪ annoyed of the speaker for the line mentioned in the
OT

Or words with similar meanings question.

Illustration- Exclamation mark, suggestive of Content -


the inherent emotion Award 1 mark for full explanation of the
BR

identification of the tone, supported by


reference to the supporting evidence.
Award ½ mark for partial explanation.
L

Expression –
YA

1 mark when both aspects included


Answer organised effectively
O

usage of words supporting illustration (as


supported by, as illustrated by, as can be
G

seen from etc.)


½ mark when either aspect is missing
Deduct ½ mark from the overall score if the
error density is high (more than a total of 2
spellings and grammatical errors).

Score Plus CBSE Sample Question Paper with MTPs in English Language & Literature for Class 10 (Term II) 25
GOYAL A DIGITAL LEARNING APP
BROTHERS
PRAKASHAN
Learn @ ` 1 Per Day

(iv) A ballad includes the telling of a tale as well as a surprise ending. Using evidence from the
poem, explain how these features are included in ‘The Tale of Custard the Dragon’. (2 marks)

VALuE PoiNTs GuidANCE


▪ Tale: The poem tells the story of Custard, The question indicates that the examiner
the dragon (setting, characters, rising action, knows the elements of a ballad and that the
climax, resolution). poem has a few and hence that requires no
▪ portrays his life with Belinda and the other elaboration.
The question needs an answer identifying the

N
pets where he is considered a coward
features that match to the listed aspects of a
▪ Surprise ending: The end of the poem shows

HA
ballad
how Custard rose to the occasion; gobbled
the pirate and proved his bravery. Content -
Award 1 mark for full explanation of the two

AS
strands (Tale and Surprise ending).
Award ½ mark for partial explanation.
Expression –

AK
1 mark when both aspects included
PR Answer organised effectively
usage of words linking ballad to evidence
from the poem (similarly, just as etc.)
½ mark when either aspect is missing
Deduct ½ mark from the overall score if the
RS

error density is high (more than a total of 2


spellings and grammatical errors).
HE

(v) Which two issues about himself convinced Lomov of his decision to get married? (2 marks)

VALuE PoiNTs GuidANCE


OT

▪ He is expected to marry and cannot stay The question needs an answer to the points
unmarried he thought about himself. Those favouring
BR

▪ He ought to lead a quiet, settled and regular Natalaya are not relevant here.
life at his age(‘critical age’/ thirty-five). Content -
▪ He needs a partner as he suffers from Award 1 mark for any 2 relevant points
Award ½ mark for any one point
L

palpitations and is always getting upset.


Expression –
YA

1 mark when both aspects included


Answer organised effectively
O

usage of words for listing/ order (primarily/


To begin with/ apart from this/ In addition
G

to this/ Also/ secondly etc.)


½ mark when either aspect is missing
Deduct ½ mark from the overall score if the
error density is high (more than a total of 2
spellings and grammatical errors).

26 Score Plus CBSE Sample Question Paper with MTPs in English Language & Literature for Class 10 (Term II)
GOYAL A DIGITAL LEARNING APP
BROTHERS
PRAKASHAN
Learn @ ` 1 Per Day

(vi) Briefly state how Matilda invited ‘a dreadful life of necessity’ into her family. (2 marks)

VALuE PoiNTs GuidANCE


▪ Matilda’s extreme self-indulgence--dreams The question does not require the summary
of a luxurious life, riches and jewellery of the story but needs an answer only with
▪ Doesn’t pay heed to the advice of wearing relevant reference to Matilda’s extreme self-
natural flowers-- borrows the necklace-- indulgence and loss of the necklace.
loses it Content -

N
▪ In paying for the necklace invited a horrible Award 1 mark for full explanation of the two
life of necessity and deprivation/ a life that strands

HA
resulted in a hand-to-mouth existence OR Award ½ mark for partial explanation.
any other relevant point
Expression –
1 mark when both aspects included

AS
Answer organised effectively
usage of words for elaboration and cause-

AK
effect
½ mark when either aspect is missing
PR
Deduct ½ mark from the overall score if the
error density is high (more than a total of 2
spellings and grammatical errors).
RS

(vii) The hack driver successfully trapped the narrator in his web of words. Comment. (2 marks)

VALuE PoiNTs GuidANCE


HE

▪ Hack driver – a conversationalist, cheerful, The answer needs to showcase the hack
friendly, open driver’s clever and crafty words with evidence
OT

▪ befriends the narrator who trusts him for of the narrator being conned/tricked
everything he says and does Content -
▪ asks the narrator to stay behind, himself Award 1 mark for full explanation of the two
BR

makes inquiries, offers home-made lunch, strands


mints money Award ½ mark for partial explanation.
▪ befools and outwits the narrator in the guise Expression –
L
YA

of help
1 mark when both aspects included
Answer organised effectively
O

usage of words for description and


substantiation
G

½ mark when either aspect is missing


Deduct ½ mark from the overall score if the
error density is high (more than a total of 2
spellings and grammatical errors).

Score Plus CBSE Sample Question Paper with MTPs in English Language & Literature for Class 10 (Term II) 27
GOYAL A DIGITAL LEARNING APP
BROTHERS
PRAKASHAN
Learn @ ` 1 Per Day

7. LoNG QuEsTioNs 4*2


ANY 2 oF 3
GuidANCE – Content 2 marks; Expression & Accuracy 2 marks
Note-
Use the given descriptors to mark the LQs for CONTENT (refer to value points) and EXPRESSION
If the response does not justify all points of a level, the response is marked down.

dEsCRiPToRs FoR CoNTENT MARK

N
• Sustained, clear, well-developed personal response to the task 2
• Well-developed and justified arguments/evidence for the characters

HA
• Largely, a reasonably well-developed personal response to the task 1½
• Clear justification with arguments/evidence for the characters

AS
• Fairly competent personal response to the task 1
• Clear justification with restricted arguments/evidence for the characters
• Limited awareness of the task ½

AK
• Limited justification or relevant arguments/evidence for the characters

dEsCRiPToRs FoR EXPREssioN (Coherence & Cohesion) MARK


PR
• Carefully structured content with a beginning, middle and end with highly 2
relevant ideas presented cohesively.
• Highly effective vocabulary usage, relevant and appropriate sentences for
RS

conveying the ideas precisely and effectively.


• Spelling, punctuation and grammar are almost always accurate
HE

• Ideas generally well sequenced and related to the given topic maintaining overall 1½
cohesion of ideas.
• Range of vocabulary suffices in large parts to convey the overall idea and meaning
OT

• Spelling, punctuation and grammar mostly accurate, with occasional minor


errors but does not impede communication
• Ideas sequenced fairly well and related to the given topic, sometimes maintaining 1
BR

cohesion of ideas.
• Range of vocabulary is limited and conveys a basic idea of the overall meaning
• Spelling, punctuation and grammar fairly accurate, with occasional minor errors
L

but does not impede communication


YA

• Poor sequencing of ideas; though related to the given topic, expressed in a ½


disjointed manner exhibiting a lack of coherence of ideas.
• Very limited expected/ topical vocabulary as per question asked
O

• A lot of errors in spelling, punctuation and grammar that impede communication.


G

(i) Parents play a crucial role in the upbringing of their children. Critically examine the parents of
Bholi and Ebright, highlighting their impact on their children’s lives. (4 marks)
VALuE PoiNTs
students may draw upon the following:
▪ introductory sentence:
crucial and indelible role played by parents

28 Score Plus CBSE Sample Question Paper with MTPs in English Language & Literature for Class 10 (Term II)
GOYAL A DIGITAL LEARNING APP
BROTHERS
PRAKASHAN
Learn @ ` 1 Per Day

parents influence their children both implicitly (indirectly) and explicitly (directly)
are role models, first teachers, friends, companions, guides
▪ Critical examination:
Richard H. Ebright’s mother- loving, caring, understanding, a friend, a companion
filled the vacuum in Ebright’s life in his father’s absence
invested time and energy in his upbringing which made him see heights of success
Bholi’s parents – uncaring, indifferent, biased, insensitive, had a patriarchal mindset

N
traditional outlook- did not believe in the education of girls, totally indifferent to Bholi

HA
and her needs, neglected her
did not bother to groom her (oil her hair, give her good clothes to wear)
sent her to school not to educate her but to save their own face

AS
despite their prosperity, they left Bholi to her own misery, worst was when they decided
to marry her off to Bishamber

AK
(Accept any other relevant content point that lends itself to critical examination)
▪ Concluding thought:
PR
The contribution and companionship of parents cannot be denied. Love, care, and
guidance from parents makes a difference, sculpts children and shapes their future.
(ii) Pranjol and Rajvir discuss their next vacation destination. They shortlist Coorg and Goa. Rajvir
RS
is keen on Coorg and tries to convince Pranjol. Develop a conversation between the two, based
on your understanding of Glimpses of India. (4 marks)
specimen answer to assist content points
HE

Rajvir: Hey Pranjol! I think we should be visiting Coorg. It is a beautiful place with coffee
plantations. I can smell the aroma already!
OT

Pranjol: We visited a tea plantation last year, in Assam; I want to…


Rajvir: There is a lot more to do in Coorg than smelling the coffee! The place has rainforests,
so the megafauna will be worth watching. Not just this, Coorg provides opportunities to indulge
BR

in adventure sports like river rafting, rappelling, mountain biking, to name a few.
Pranjol: That sounds interesting, but I would prefer some serene moments too, away from this
post-pandemic hustle-bustle.
L

Rajvir: Oh! The answer is Coorg again!


YA

Pranjol: Oh, come on! You can’t be serious…


Rajvir: Believe me, I am. Coorg is the place. It has beautiful natural walking trails and the
Brahmagiri hills offer a panoramic view. I read that the place has the largest Tibetan settlement,
O

so the environment will reflect peace and spirituality, I’m sure.


G

Pranjol: Have to say, you’ve presented a fine case in favour of Coorg and convinced me. Let’s
plan to leave for Coorg next Wednesday!
▪ The dialogues should be based on understanding of the chapter. There should be arguments
from Pranjol showing he requires more to get convinced. Rajvir, being an explorer must be
able to give an interesting account about Coorg to convince Pranjol. The persuasive element
using examples from text descriptions need to be showcased.

Score Plus CBSE Sample Question Paper with MTPs in English Language & Literature for Class 10 (Term II) 29
GOYAL A DIGITAL LEARNING APP
BROTHERS
PRAKASHAN
Learn @ ` 1 Per Day

Apart from the above the following could be added:


• Coorg is culturally rich, the people there are brave and their hospitality is great.
• Some people in Coorg are the only ones allowed to keep firearms without license. It will
be fun meeting such bravehearts.
• Rainforests can be visited for flora and fauna.
• There flows the river Kaveri and sitting by the river could be peaceful.
(iii) Farce is a kind of comedy in which the situations and dialogues are ridiculous, exaggerated and

N
even absurd. Evaluate the play ‘The Proposal’ as a farce. (4 marks)
VALuE PoiNTs

HA
introductory sentence:
• farcical characters, their ridiculous and odd behaviour, unlikely and exaggerated situations

AS
(profuse in the play)
Evaluation:
• humor in the play, improbable situations, childish behavior of the characters (making a

AK
mountain out of a molehill), arguments and quarrels
• hurling of accusations and insults without a second thought
PR
• resolving the differences and fighting again over another topic
• Lomov’s nerve problems and other ailments, Chubukov’s theatrical statements, Natalaya’s
impulsive and belligerent remarks and other relevant evidence.
RS

Concluding statement:
• the manner in which the final proposal is made amidst all the chaos, makes the play a
HE

farce
OT
BR
L
YA
O
G

30 Score Plus CBSE Sample Question Paper with MTPs in English Language & Literature for Class 10 (Term II)
GOYAL A DIGITAL LEARNING APP
BROTHERS
PRAKASHAN
Learn @ ` 1 Per Day

N
HA
AS
AK
PR
RS
HE
OT
BR
L
YA
O
G

27
BROTHERS
PRAKASHAN
GOYAL A DIGITAL
Learn @LEARNING
` 1 Per DayAPP
BROTHERS
PRAKASHAN
Learn @ ` 1 Per Day

N N
HA HA
AS AS
AK AK
PR PR
RS RS
HE HE
OT OT
BR BR
L L
YA YA
O O
G G

To view Sample or Purchase of Books


WWW.goyal-books.com/mtpcx
To view Sample or Purchase of Books
7R YLHZ 6DPSOH RU 3XUFKDVH RI %RRNV 8
www.goyal-books.com/mtpcx
To view Sample or Purchase of Books
:::JR\DOERRNVFRPPWSF[
www.goyal-books.com/mtpcx
28

To view Sample or Purchase of Books
G

WWW.goyal-books.com/mtpcx
To view Sample or Purchase of Books
7R YLHZ 6DPSOH RU 3XUFKDVH RI %RRNV 8
www.goyal-books.com/mtpcx
To view Sample or Purchase of Books
:::JR\DOERRNVFRPPWSF[
www.goyal-books.com/mtpcx

28
GOYAL A DIGITAL LEARNING APP
BROTHERS
PRAKASHAN
Learn @ ` 1 Per Day

MODEL TEST PAPER–1 (Solved)


(Based on the latest CBsE sample Paper)

Time allowed: 2 Hours Maximum Marks: 40

General Instructions:
1. The Question Paper contains THREE sections-READING, WRITING & GRAMMAR and

N
LITERATURE.
2. Attempt questions based on specific instructions for each part.

HA
sECTioN A - REAdiNG (10 marks)

AS
1. Read the passage given below.
The difference between effectively and ineffectively pursuing happiness may all be in how
we go about it. Research suggests that people who strive to feel happy all of the time may

AK
suffer disappointment, and people who pursue happiness, as if it were the only thing that
matters, may, ironically, chase happiness away. But these are not the only ways you can go
about pursuing happiness. Another approach involves what I call “prioritizing positivity”:
PR
deliberately organizing your day-to-day life so that it contains situations that naturally give
rise to positive emotions. This way of pursuing happiness involves carving out time in
your daily routine to do things that you genuinely love, whether it be writing, gardening
RS

or connecting with loved ones. This way of pursuing happiness means proactively putting
yourself in contexts that spontaneously trigger positive emotions.
HE

One reason is that prioritizing positivity involves monitoring one’s daily itinerary, not one’s
moment-to-moment emotional experience. As research has revealed, the mere act of monitoring
one’s happiness from one second to the next may get in the way of positive emotions. In
OT

contrast, we thought a more effective strategy would be “situation selection,” which involves
approaching (or avoiding) situations that naturally trigger certain emotions. Going for a walk
with your partner after dinner is one way you might use situation selection to experience a
BR

feeling of tranquillity. The tendency to prioritize positivity draws upon this strategy.
We also predicted that prioritizing positivity would predict greater happiness and fewer
symptoms of depression because of its relevance to daily life. The “highs” we get from
L

one-time events like going on a vacation or winning a prize wear off over time. As a
YA

result, pursuing happiness may require regular engagement in behaviours that promote
happiness. By its nature, prioritizing positivity increases the chance that we will weave
these positive behaviours into our daily lives rather than just maintaining a general desire
O

for happiness or expecting it to come from a few isolated events.


The science on the deliberate pursuit of happiness is young, so any prescriptions for happiness
G

must be offered with the caveat that the research is still evolving and conclusions might be
subject to change. I do have some speculations, based on my research, about how people
might more effectively pursue happiness. First, let go of extreme ways of relating to your
happiness. Don’t set the unrealistic goal of feeling positive emotions all—or even most—of
the time. Second, reflect on the activities that give you joy or contentment. This thought
experiment should be highly personalized. Finally, once you think of a couple of activities,
schedule them into your upcoming week.
Score Plus CBSE Sample Question Paper with MTPs in English Language & Literature for Class 10 (Term II) 31
GOYAL A DIGITAL LEARNING APP
BROTHERS
PRAKASHAN
Learn @ ` 1 Per Day

The pursuit of happiness is not easy. If people attempt it with unrealistic expectations and too
much attention, they risk sabotaging it. But this doesn’t mean you should give up on trying
to be happy. It may be more effective to adjust your daily routine so that it includes activities
that naturally spark interest or contentment. Seeking happiness, although a delicate art, may
still be a worthwhile pursuit.
Based on your understanding of the passage, answer ANY FiVE questions from the six
given below. 1*5
(i) Why might some people chase away happiness? 1

N
Ans. Some people might chase away happiness because their ultimate goal is to find happiness

HA
in every thing.
(ii) Rewrite the following sentence by replacing the underlined phrase with a word that
means the same from para 2. 1

AS
Lack of sleep or too much sleep can lead to migraine.
Ans. Lack of sleep or too much sleep can trigger migraine.

AK
(iii) State two ways to pursue happiness. 1
Ans. The two ways to pursue happiness are, first, don't strive to be happy; rather, feel happy.
PR
The second reason is "prioritizing positivity", that is, deliberately organise your day-to-
day life so that it contains situations that naturally give rise to positive emotions.
(iv) What does the author mean when he says, “the highs we get”? 1
RS

Ans. By saying “the highs we get”, the author refers to the happiness and excitement we get
from one-time events of our life like going on a vacation.
(v) Write the author's speculations about how people can pursue happiness more effectively. 1
HE

Ans. As per the author's speculations, a person must let go of extreme ways that relate to
happiness. He/she shouldn't set any unrealistic goal of feeling positive emotions. Also,
OT

one must reflect on the activities that give joy or contentment.


(vi) How does the author explain the pursuit of happiness? 1
Ans. The author explains the pursuit of happiness through the concept of prioritizing positivity.
BR

2. Read the passage given below.


Globalization is the way to open businesses, improve technological growth, economy, etc. at
L

the international level for all countries. It is the way in which manufacturers and producers
YA

of the products or goods sell their products globally without any restriction. It provides huge
profits to the businessmen as they get low cost labour in poor countries easily. It provides a
big opportunity to the companies to deal with the worldwide market.
O

Globalization helps to consider the whole world as a single market. Traders are extending
their areas of business by treating the world as a global village. Earlier, till the 1990s, there
G

was a restriction on importing certain products which were already manufactured in India
like agricultural products, engineering goods, food items and toiletries. However, during the
1990s, there was a pressure from the rich countries on the poor and developing countries to
allow them to spread their businesses by opening their markets. In India, the globalization
and liberalization have been a bonanza for the consumers, however, a loss to the small-scale
Indian producers.
32 Score Plus CBSE Sample Question Paper with MTPs in English Language & Literature for Class 10 (Term II)
GOYAL A DIGITAL LEARNING APP
BROTHERS
PRAKASHAN
Learn @ ` 1 Per Day

Main players for international trade in goods, 2018


(billion EUR)

2,500

2,000

N
1,500

HA
1,000

AS
500

AK
0
EU-28 (1) China (2) United Japan South Mexico Canada India Singapore Russia Switzer- Brazil
States Korea land (3)
 Exports    Imports

(1) External trade flows with extra EU-28.


(2) Excluding Hong Kong, estimated data.
PR
(3) Including Liechtenstein.
Source: Eurostat (online data code: ext_lt_introle, Comtrade and UNCTAD)
RS

Globalization has had some very positive effects on the Indian consumer in all sectors of society.
HE

It has affected the Indian students and education sector to a great extent by making study books
and a lot of information available over the internet. Collaboration of foreign universities with the
Indian universities has brought about a huge change in the field of education
OT

Globalization of trade in the agricultural sector has brought varieties of quality seeds which
have disease resistance property. However, it is not good for the poor Indian farmers because
BR

the seeds and agricultural technologies are costly. It has brought about a huge revolution in
the employment sector by the spread of businesses like handloom, carpet, artisan carving,
ceramic, jewellery and glassware, etc.
With globalization’s strong momentum running up against powerful headwinds, it is important
L

to recognize that market integration is still limited in absolute terms. The foreign operations
YA

of multinational firms around the world generate only about 9% of global output. Exports
of goods and services add up to 29% of world GDP, but even that figure comes down to
about 20% if we adjust for output that crosses borders more than once. Managers surveyed
O

across six countries in 2017 estimated these international production and trade metrics at 37%
and 41%, respectively.
G

We cannot predict with confidence whether the coming year will bring a higher or a lower
level of globalization. But we can safely say that international flows and the constraints
that borders and distance impose upon them will both continue to matter. So, the biggest
winners regardless of whether globalization goes up or down, are likely to be companies that
embrace globalization’s complexity rather than purely local or global visions of their business
environments.

Score Plus CBSE Sample Question Paper with MTPs in English Language & Literature for Class 10 (Term II) 33
GOYAL A DIGITAL LEARNING APP
BROTHERS
PRAKASHAN
Learn @ ` 1 Per Day

on the basis of your understanding of the passage, answer ANY FiVE questions from
the six given below. 1*5
(i) What do the observations state about market integration? 1
Ans. The observations about market integration state that the export of goods and services
accounts for 29% of world GDP.
(ii) Comment on the inherent traits of globalization with reference to para 1. 1
Ans. Globalization helps in improving technological growth and the economy at an
international level. It led to the way in which manufacturers and producers of products or

N
goods sell their products globally without any restrictions.

HA
(iii) Who doesn’t benefit from globalization in India in reference to para 2? 1
Ans. In India, small-scale producers don’t get much benefit from globalization.
(iv) To which fact does the ‘global village’ concept of globalization point? 1

AS
Ans. The ‘global village’ concept of globalization points to the fact that the information sector
has been boosted.

AK
(v) According to the graphical representation, who are the biggest and the smallest players in
import in international trade in 2018? 1
Ans. The graphical representation clearly shows that the biggest player in import in international
PR
trade in 2018 was the United States of America, and the smallest player was Brazil.
(vi) What does the impact of globalization on the agricultural sector indicate? 1
Ans. The impact of globalization on the agricultural sector indicates that the quality of seeds
RS

has improved, but affordability has decreased.

sECTioN B - WRiTiNG ANd GRAMMAR (10 marks)


HE

3. Attempt ANY oNE from (i) and (ii).


(i) The pie chart below shows the spending
OT

Digital 5% 4% Radio
on advertising in India through different Out-of-home 6%
media. The accompanying extract talks about
Print
BR

advertising as an industry. Write a paragraph 39%


commenting upon the situation.
Advertising was initially meant to make people
aware of the goods available in the market. It TV 46%
L

was as simple as announcing what you have


YA

in your store or the services you offer on your


premises. Over the years, advertising has evolved into a major industry that goes beyond
informing to persuading and influencing. It is a form of brainwashing consumers. 5
O

Ans. Gone are the days when advertising was meant just to announce a product. We now have
G

specially designed advertising campaigns by the fully evolved and established advertising
industry. The job of the advertising industry is not only to inform consumers about the
launch of a new product but also to overwhelm them with stimuli that would persuade
them to buy the product. TV and print media have a major role to play with the annual
spend on TV advertisements, amounting to 46 per cent and print to 36 per cent. The
spending on advertisements on the radio and digital media are as minor as 4 and 5 per
cent respectively. Out-of-home advertising has a minuscule share of 6 per cent.

34 Score Plus CBSE Sample Question Paper with MTPs in English Language & Literature for Class 10 (Term II)
GOYAL A DIGITAL LEARNING APP
BROTHERS
PRAKASHAN
Learn @ ` 1 Per Day

(ii) You have just graduated in Commerce (B.Com.) from Bangalore University. You want
to do a high-level computer course at a reputed institute. Write a letter making inquiries
about admission fees and courses offered. 5
Ans. B-9, Battery Lane
Avenue Road
Bengaluru-560002

10 July 20XX

N
The Director

HA
Alps Computer Institute
MG Road
Bengaluru-560001

AS
Dear Sir

AK
Subject: Seeking admission to a Computer Course

I have just received my degree in B.Com from Bangalore University. I'm a meritorious
PR
student with a keen interest in computers. Recently, I heard about your institute and
the vast number of computer courses it offers. Therefore, I am keen to do a high-level
computer course from your institute. Please let me know the criteria for admission,
courses offered, their level and duration, and fees to be charged. I will be grateful if
RS

I get a detailed reply from you as soon as possible.

Yours faithfully
HE

Nagarath Karman
4. The following paragraph has not been edited. There is one error in each line. Identify the
OT

error and write its correction against the correct blank number. Remember to underline the
correction. The first one has been done for you. 1*3
Error Correction
BR

Only those who work hard gets glory and success in life. E.g. gets get
Nothing great could be achieved without effort. (a)
A student can never comes out with flying colours unless he (b)
L

has really worked hard throughout a year. (c)


YA

Ans. Error Correction


(a) could can
O

(b) comes come


(c) a the
G

5. Read the conversation between Raghav and Ravi and complete the passage that follows. 1*2
Raghav: Ravi, Varun and I are going to watch the cricket match tomorrow. Would you like to
join us?
Ravi: I would love to, but I have to take my father’s permission first.
Raghav: You must let me know by this evening because I have to arrange for the passes.
Ravi: I will do so as soon as I get permission.

Score Plus CBSE Sample Question Paper with MTPs in English Language & Literature for Class 10 (Term II) 35
GOYAL A DIGITAL LEARNING APP
BROTHERS
PRAKASHAN
Learn @ ` 1 Per Day

Raghav informed Ravi that Varun and he were going to watch the cricket match the next
day. He asked Ravi (a) ________________. Ravi replied that he would love to, but he had to
take his father’s permission first. Raghav told him (b) ________________ because he had to
arrange for the passes. Ravi said that he would do so as soon as he would get permission.
Ans. (a) whether he would like to join them
(b) to let him know by that evening

sECTioN C - LiTERATuRE (20 marks)

N
6. Answer ANY siX questions in 30-40 words each. 2*6

HA
(i) “Not enough can be said to show how important a baker can be for a village.” How
were the services of the baker required on various occasions? 2
Ans. The services of the baker were, undoubtedly, required on various occasions, making

AS
him a very important person for a village. Mothers had to prepare sandwiches for
their daughter’s engagement. Cakes and bolinhas were a must for Christmas and other
festivals. Thus, the services of the baker in the village are important.

AK
(ii) Amanda was longing for wishful desires. List them. 2
Ans. Initially, she imagined herself as a drifting mermaid in the green emerald sea. She also
PR
imagined herself as an orphan and a fictional character, Rapunzel, to live aloof and enjoy
her freedom.
(iii) What is the turning point in the poet’s life in the poem ‘Animals’? 2
RS

Ans. The poet expresses his desire to live with animals instead of humans. He believes
humans do not possess many qualities that animals do, such as kindness, love, respect,
and truthfulness.
HE

(iv) Belinda’s pets, except Custard, were a boastful lot. Explain with reference to the poem. 2
Ans. Belinda and her three pets, excluding Custard, were very proud of their bravery, but they
OT

were not brave. Only Custard dared to face the pirate, so he was fearless. The other pets
soon made excuses for their cowardice and said that they would have been twice and
thrice as brave as Custard.
BR

(v) Give exemplars from the story to show that Valli was a painstaking and meticulous
organizer. 2
Ans. Valli was a meticulous planner. She listened carefully to her neighbours' conversations
L

and people who regularly used the bus and asked them discreet questions. She picked up
YA

the various small details about the bus journey and then planned it.
(vi) Matilda rarely met her rich friends. Why? 2
Ans. Matilda was unhappy due to her poverty and lack of comforts in her life. She lived in
O

a shabby house and remained ill-tempered over her poor situation. This was the only
reason why she rarely met her friends.
G

(vii) What do you know about the character of Lutkins? Was he served summons? 2
Ans. Oliver Lutkins was a hack driver who showed himself as Bill before the lawyer to
deceive him.
The lawyer was sent to New Mullion to serve him notice. Yes, the summons was served
to Lutkins.

36 Score Plus CBSE Sample Question Paper with MTPs in English Language & Literature for Class 10 (Term II)
GOYAL A DIGITAL LEARNING APP
BROTHERS
PRAKASHAN
Learn @ ` 1 Per Day

7. Answer ANY TWo of the following in about 120 words each. 4*2
(i) “Put the fear out of your heart, and you will be able to speak like anyone else.” These
words of encouragement from the teacher highlight that a change of social attitude and
encouragement can help a child like Bholi to become confident and face the world
bravely.
Taking help from the lesson ‘Bholi’, write how the social attitude towards Bholi made
her an introvert. What should be done to help such children face the world bravely? 4
Ans. Bholi’s mind was weakened due to a childhood mishap (falling from her cot). She began

N
to stammer while speaking as well. She then became unattractive due to pock-marks

HA
on her face and a diseased body caused by smallpox. As a result, her family and other
children mistreated her, causing her to become an introvert.
We must treat such youngsters with love and affection and urge them to enter

AS
mainstream society to help them face the world boldly. Instead of mocking their
limitations, we should motivate and encourage them, giving them hope that they can be
as good as the other children.

AK
(ii) The Buddha said, ‘The world is afflicted with death and decay; therefore, the wise do
not grieve, knowing the terms of the world.’ Write your views in the context of the
above statement.
PR
Ans. In his first sermon, Gautama Buddha preached that human life is very short and it
4

is full of sorrow and pains. He cleared that the mortals’ brief life is full of troubles
RS

and pains. Everyone on this earth has to one day face death. Human beings are led
to death as an ox taken for killing. We cannot console ourselves by weeping and
grieving. By following these methods out, the pain will be greater. Our bodies will
HE

suffer and become weak. The wise and intelligent persons never weep on death and
grieve on sorrow. They know that these are the realities and truths of our lives.
OT

Those who are born are to die certainly. No one can escape from death. So we
should not grieve on deaths.
(iii) ‘Disagreements and quarrels occasionally solve a problem.’ Discuss in brief the various
BR

disputes between Natalya and Lomov. 4


Ans. Lomov and Chubukovs were neighbours. Lomov came to Chubukov’s house to propose
Natalya. But instead of proposing, Lomov and Natalya started fighting over petty
L

issues like Oxen Meadows and the qualities of their dogs. This is not a quality of good
neighbours. Neighbours should be like friends. There should be a positive relationship
YA

between two neighbours. There should be understanding and patience among neighbours.
The fight between Lomov and Natalya could have been avoided if handled carefully.
The issue of the Meadows could be resolved calmly and quietly by a good conversation.
O

There must have been some documents showing the ownership of the Meadows. Both
the parties could be convinced by making them understand that the Meadows would
G

belong to both of them if they got married.


Similarly, the issues of dogs could be solved. Instead of counting the negative points
of each other’s dogs, they could have discussed their positive features. Thus, the issues
could be resolved.

Score Plus CBSE Sample Question Paper with MTPs in English Language & Literature for Class 10 (Term II) 37
GOYAL A DIGITAL LEARNING APP
BROTHERS
PRAKASHAN
Learn @ ` 1 Per Day

N
HA
AS
AK
PR
RS
HE
OT
BR
L
YA
O
G

27
BROTHERS
PRAKASHAN
GOYAL A DIGITAL
Learn @LEARNING
` 1 Per DayAPP
BROTHERS
PRAKASHAN
Learn @ ` 1 Per Day

N N
HA HA
AS AS
AK AK
PR PR
RS RS
HE HE
OT OT
BR BR
L L
YA YA
O O
G G

To view Sample or Purchase of Books


WWW.goyal-books.com/mtpcx
To view Sample or Purchase of Books
7R YLHZ 6DPSOH RU 3XUFKDVH RI %RRNV 8
www.goyal-books.com/mtpcx
To view Sample or Purchase of Books
:::JR\DOERRNVFRPPWSF[
www.goyal-books.com/mtpcx
28

To view Sample or Purchase of Books
G

WWW.goyal-books.com/mtpcx
To view Sample or Purchase of Books
7R YLHZ 6DPSOH RU 3XUFKDVH RI %RRNV 8
www.goyal-books.com/mtpcx
To view Sample or Purchase of Books
:::JR\DOERRNVFRPPWSF[
www.goyal-books.com/mtpcx

28
GOYAL A DIGITAL LEARNING APP
BROTHERS
PRAKASHAN
Learn @ ` 1 Per Day

MODEL TEST PAPER–2 (Solved)


(Based on the latest CBsE sample Paper)

Time allowed: 2 Hours Maximum Marks: 40

General Instructions:
1. The Question Paper contains THREE sections-READING, WRITING & GRAMMAR and

N
LITERATURE.
2. Attempt questions based on specific instructions for each part.

HA
sECTioN A - REAdiNG (10 marks)

AS
1. Read the passage given below.
In these trying times, when buying ordinary foodstuff can burn a hole in our pockets, comes

AK
the news that can actually help us save some hard cash when we go out to shop the next
time. According to a Stanford University study, the first of its kind in the world, there is
no evidence to suggest that there are more nutritional benefits from expensive organic food
than those grown by conventional methods. The researchers add that there is no difference in
PR
protein and fat content between organic and conventional milk and the vitamin count is similar
in both types. The only benefit is that organic foods are not contaminated with pesticides, but
then, before you chew on that plate of organic okra with roti made from organic wheat, they
RS

are not 100% pesticide-free either. In India, organic food has been growing at 20–22% and the
export market is valued at 1,000 crores. Obviously, the study is not good news for that sector
and for people who are big on organic food.
HE

In India, eating organic food is more of a style statement than due to health worries because
the stuff is expensive. But people who can do indulge in not only organic vegetables but even
OT

organic eggs laid by ‘happy hens’, who are allowed to roam around freely, whereas ‘unhappy
hens’ are kept in coops. Then there are companies that have installed music channels in
their cowsheds and the milk from those sheds is sold at a marked-up price since it has more
BR

nutritional value because the animals are happy thanks to lilting 24 × 7 music. We don’t know
yet of any farmer using music to improve his crop quality, but then you never know: plants
are known to respond to music.
L

Why such pickiness about food? These days, the huge number of TV shows and articles that
YA

we see and read on food provide bread and butter for the specialists. But instead of decoding
food, its sources and what has gone into growing it, isn’t it much better to enjoy what’s on
the plate?
O

Based on your understanding of the passage, answer ANY FiVE questions from the six
given below. 1*5
G

(i) What is the real reason behind the popularity of organic products? 1
Ans. The reason behind the popularity of organic products is it has become a fashion. It is
more of a style statement than people's health issues.
(ii) Rewrite the following sentence by replacing the underlined phrase with a word that
means the same from para 1. 1

38 Score Plus CBSE Sample Question Paper with MTPs in English Language & Literature for Class 10 (Term II)
GOYAL A DIGITAL LEARNING APP
BROTHERS
PRAKASHAN
Learn @ ` 1 Per Day

Some of the ordinary and traditional practices for growing grains are still prevalent.
Ans. Some of the conventional practices for growing grains are still prevalent.
(iii) State one factor that favours the organic food manufacturers. 1
Ans. The only factor is that organic foods are not contaminated with pesticides.
(iv) What does the author mean when he uses the idiom ‘burn a hole in our pockets’? 1
Ans. By using the idiom ‘burn a hole our pockets’, the author means that people spend money
lavishly.

N
(v) “Eating organic food is more of a style statement than health worries for those who can

HA
afford it.” List one more thing these people indulge in as a style statement. 1
Ans. These people, who can afford the expensive organic food, also indulge in organic eggs
laid by ‘happy hens’, who are allowed to roam around freely.

AS
(vi) What is the author’s attitude towards people who eat food sourced from happy
animals? 1
Ans. The author is unhappy with people who eat food sourced from happy animals.

AK
2. Read the passage given below.
PR
In 2015, there were 4.2 million deaths globally because of pollution, caused by fine particulate
matter with a diameter less than 2.5 micrometer, according to environmental research
organisation Health Effects Institute. India accounts for one in four of these deaths—1.09
RS
million where research shows the youth are largely ignorant about environmental issues.

The Hindustan Times-MaRS Monitoring and Research Systems Youth Survey 2017, reveals
that more than 74% of people between the ages of 18 and 25 are not aware of how
HE

greenhouse gases cause global warming. Renewable energy is a mystery to 70.9% of the
respondents and 63.4% cannot explain why bio-degradable waste is important. About 68.4%
OT

of youngsters cannot identify eco-friendly products and about 80.2% youngsters have never
participated in cleanliness drives.
BR

Sustainable practices are limited to switching off lights at home and avoiding use of “plastic
bags as far as possible.”

Experts feel, the youth will be forced to act soon as they will most likely be left with no
L

choice since the next few decades will see a more pronounced impact of environmental
YA

inaction. The pollution, the landfills and the food shall become more synthetic and will force
the youth to take steps in order to adapt to the future world that we are creating.

Studies related to sustainable development and climate change should be built into the
O

education curriculum. Children should be taught how waste is segregated; they should
understand why the waste problem exists.
G

The youth should build a consensus on a future they deeply desire. What is it that they wish
to have, say, in 2040? Do they see a world that is more crowded than today, more polluted
than today, more expensive, poorer, more polarized, more synthetic? If they can create a
shared vision for themselves, the youth can achieve a lot.

Score Plus CBSE Sample Question Paper with MTPs in English Language & Literature for Class 10 (Term II) 39
GOYAL A DIGITAL LEARNING APP
BROTHERS
PRAKASHAN
Learn @ ` 1 Per Day

N
HA
AS
AK
PR
on the basis of your understanding of the passage, answer ANY FiVE questions from
RS

the six given below. 1*5


(i) What did Monitoring and Research Systems Youth Survey 2017 reveal? 1
HE

Ans. Monitoring and Research Systems Youth Survey 2017 revealed that only a minor section
of the Indian youth is aware of environmental issues.
OT

(ii) Comment on the practices followed by the youth for a sustainable environment with
reference to para 2. 1
Ans. The youth, for a sustainable environment, is limited to switching off lights at home and
BR

avoiding the use of plastic bags as far as possible.


(iii) Justify the writer's recommendation to include studies related to sustainable development
and climate change in the education curriculum. 1
L

Ans. Including studies related to sustainable development and climate change in the education
YA

curriculum is necessary, as children should be taught how waste is segregated. Children


should understand why the problem of waste exists.
(iv) Why do the experts in the field of environmental studies feel the younger generation will
O

be forced to retrieve their reckless attitude towards the environment? 1


Ans. The experts in the field of environmental studies felt that because the younger generation
G

would, in the next few decades, see a more pronounced impact of environmental inaction.
(v) According to the survey, only a very small percentage of Indian youth is aware of
the greenhouse effect. How does the evidence from the graphical data support this
statement? 1
Ans. The graphic data clearly shows that only 25.7% of the Indian youth is aware of the greenhouse
effect.
40 Score Plus CBSE Sample Question Paper with MTPs in English Language & Literature for Class 10 (Term II)
GOYAL A DIGITAL LEARNING APP
BROTHERS
PRAKASHAN
Learn @ ` 1 Per Day

(vi) What purpose does the "Don't Know How Earth Can Be Saved" part serve in the graph? 1
Ans. The “Don't Know How Earth Can Be Saved” part shows that a majority of youth in the
age group of 18-25 years is unaware of renewable energy.

sECTioN B - WRiTiNG ANd GRAMMAR (10 marks)


3. Attempt ANY oNE from (i) and (ii).
(i) The chart below shows the book sales for five different types of fiction books from 2006
5

N
to 2010. Write a paragraph analyzing the given data.

HA
Fiction Book Sales

$120,000.00

AS
$100,000.00

$80,000.00
Gross Earnings

Young Adult

AK
$60,000.00 Classics
Mystery
$40,000.00 Romance

$20,000.00
PR Sci-Fi & Fantasy

$0.00
2006 2007 2008 2009 2010
RS

Year

Ans. In the given bar graph, fiction book sales are grouped from 2006 to 2010. Five
HE

categories of fiction books are compared according to income. It is clear from the chart
that the romantic fiction books are bestsellers, collecting the highest revenue. Romance
novels started selling off for around $80,000 in 2006 and then were sold for a major
$110,000 in 2007. The sales dipped the next year reasonably and ranged from $70,000
OT

to just above the $80,000 mark in the graph by 2010. Mystery novels were the second-
highest selling books, with a high of $80,000 in 2007, remaining between $60,000 to
$45,000 for the rest of the years. Young adult novels steadily sold for up to $30,000
BR

every year. Classics sold for an average of around $19,000 for all the years, the best
year being 2007 ($25,000) and the worst, 2010 (around $10,000). The least sold novels
were sci-fi and fantasy books that reached a height of just $20,000 in 2007, otherwise
remaining below that mark. Overall, romance and mystery books have been leading in all
L

the years, and 2007 is the most profitable year for book sales in all 5 categories.
YA

(ii) A new construction company has advertised a new colony, ready to be completed by
January 20XX. Write a letter of inquiry asking for all the details — the size of the
O

flats, number of storeys, amenities, security offered, the date of complete possession
and the total cost of owning a flat. Ask them to set a date for a meeting after
G

sending the details. 5


Ans. B–24/1A
24, Vidhan Sabha Road
Delhi–110054

Score Plus CBSE Sample Question Paper with MTPs in English Language & Literature for Class 10 (Term II) 41
GOYAL A DIGITAL LEARNING APP
BROTHERS
PRAKASHAN
Learn @ ` 1 Per Day

8 January 20XX

The Director
The Modern Housing Company
Sector 92
Gurugram–122505

N
Dear Sir
Subject: Details of a new Housing Complex

HA
I have just read your advertisement in all the leading papers for a new Housing
Complex—the Rosary, almost completed by you. You still have some flats to sell. Please

AS
let me know at the earliest the size, floor, amenities, bathrooms, bedrooms, etc., available
in your flats. What other securities are you offering—water, electricity, gardens, play sites

AK
for children and above all, personal security?
I would also like to know how to approach the colony, transport available and its
distance from the main section of Gurugram. I would also like to know the complete
PR
cost of a three-bedroom (baths attached) flat on the 3rd or 4th floor.
I am hoping for a quick reply.
RS

Yours faithfully
D. Khosla
HE

(Retd. Judge of Delhi High Court)

4. The following paragraph has not been edited. There is one error in each line. Identify the
OT

error and write its correction against the correct blank number. Remember to underline the
correction. The first one has been done for you. 1*3
Error Correction
BR

In today's world, parents needed to be well-educated. E.g. needed need


Educated mothers knows that today's children demand (a)
logical reasoning in everything. They couldn’t be fooled (b)
L

by narrating many fairy tale or animal story. (c)


YA

Ans. Error Correction


(a) knows know
O

(b) couldn’t can’t


G

(c) many some


5. Read the given conversation and complete the passage that follows. 1*2
"Is Mr Modi at home?” asked the postman.
“No, he isn’t,” said Rita, ‘‘but I am his daughter. Why don’t you give that letter to me?’’

42 Score Plus CBSE Sample Question Paper with MTPs in English Language & Literature for Class 10 (Term II)
GOYAL A DIGITAL LEARNING APP
BROTHERS
PRAKASHAN
Learn @ ` 1 Per Day

The postman inquired (a) ______________________ home. Rita answered that he was not but
added that she was his daughter. She asked (b) _________________ the letter to her.
Ans. (a) whether Mr Modi was
(b) why he didn’t give

sECTioN C - LiTERATuRE (20 marks)


6. Answer ANY siX questions in 30-40 words each. 2*6

N
(i) The Coorgis are known to be very brave people. Comment. 2

HA
Ans. Coorg is inhabited by a proud race of martial men and beautiful women. The Coorg
Regiment is one of the most decorated in the Indian Army, and the first Chief of the
Indian Army, General Cariappa, was a Coorgi.

AS
(ii) Describe the glorious beauty of the tea plantations with reference to the chapter ‘Tea
from Assam’. 2

AK
Ans. Rajvir seemed to enjoy the exquisite beauty of tea plantations. It was greenery
everywhere. Rajvir was overjoyed to grasp countless rows of tiny tea bushes to the
extent that his eye could see. Doll-like figures, the women tea pluckers were stirring
PR
firmly in those tea bushes. There were groups of people who were plucking tea leaves
from the bushes.
(iii) Is Amanda an orphan? Why does she say so? 2
RS

Ans. No, she is not an orphan. She just imagines herself to be one so that she can enjoy the
freedom and avoid scolding of her mother.
HE

(iv) In the poem ‘Animals’, how true do you consider the poet and his beliefs? 2
Ans. In this poem, the poet compares the selfish world of humans with the selfless world
OT

of animals. I agree with the poet, as today’s world looks like an unrealistic world of
confusion and contradictions.
(v) Who cried for a nice safe cage, and why? 2
BR

Ans. Custard, the dragon, cried for a nice safe cage because he thought that the little white
house Belinda and her other friends lived in was unsafe, which was why he was called a
coward dragon.
L

(vi) Why did Ebright say, “Mr Weiherer opened my mind to new ideas”? 2
YA

Ans. Mr Weiherer was the most admirable person for Richard H. Ebright. He opened his mind
to great thoughts and novel ideas. He would always add great efforts besides research.
O

He had always inspired him with new thoughts and new experiments. He was a perfect
person in his life.
G

(vii) What led to a change in the lawyer’s attitude? 2


Ans. The lawyer thought that the people living in the countryside were simple, humble and
rustic. After meeting with the hack driver, his view changed. He was befooled by the
hack driver, who himself was Oliver Lutkins.

Score Plus CBSE Sample Question Paper with MTPs in English Language & Literature for Class 10 (Term II) 43
GOYAL A DIGITAL LEARNING APP
BROTHERS
PRAKASHAN
Learn @ ` 1 Per Day

7. Answer ANY TWo of the following in about 120 words each. 4*2
(i) Life is full of trials and tribulations. Kisa Gotami also passes through a period of grief in
her life.
How does she behave in those circumstances? What lesson does a reader learn from
the story of her life? Give two points about how you would like to act during adverse
circumstances. 4
Ans. Kisa Gotami’s only son had died. She went to Buddha to get a medicine that could cure

N
her son. The Buddha asked her to obtain a handful of mustard seeds from a house where
no relatives or friends had died. Kisa Gotami went from house to house. But there was

HA
not a single house where someone had not died. Thus, she was unable to fulfil what
Buddha had asked her to do. She had become selfish in wanting her son back, but later,
she realised that man is immortal.

AS
The lesson we learn from her story is that peace of mind cannot be obtained by grieving.
I would try to overcome my sorrow and accept that human life is mortal when faced

AK
with such a situation.
(ii) Age is not a barrier to doing something different and great. Which characteristics of Valli
help her achieve the wonder of visiting the town at such a tender age? 4
PR
Ans. Valli had made careful, painstaking and elaborate plans to visit the town. She had saved
every coin she could. She overcame the temptation to buy peppermints, toys, balloons
and like. Finally, she had saved a total of 60 paise. She suppressed her strong desire to
RS

ride on a merry-go-round at the village fair. Her next problem was slipping out of the
house without her mother’s knowledge. But she managed this without much difficulty.
HE

Every day after lunch, her mother would nap from about one to four or so. She would
visit the town during this period.
(iii) A little confession would have changed the life of Matilda. Should we confess our
OT

mistakes courageously? Do you agree about the above statement? 4


Ans. Yes, even a tiny blunder can cast a shadow over our lives and make them dismal and
dark. More often than not, open-minded and truthful people claim to feel happier.
BR

Matilda's life was flipped upside down by a seemingly insignificant error. If Matilda had
confessed to losing Madame Forestier's necklace, the course of her life would have been
very different.
L

Confession could have been able to bring some relief in this situation. Matilda would
YA

have picked up on the fact that the jewellery she was wearing was fake almost
immediately. Matilda's arduous efforts in repaying the jewellery she had borrowed could
have been saved. We have all received the message that we should not be scared to
O

acknowledge our mistakes and learn from them.


G

44 Score Plus CBSE Sample Question Paper with MTPs in English Language & Literature for Class 10 (Term II)
GOYAL A DIGITAL LEARNING APP
BROTHERS
PRAKASHAN
Learn @ ` 1 Per Day

N
HA
AS
AK
PR
RS
HE
OT
BR
L
YA
O
G

27
BROTHERS
PRAKASHAN
GOYAL A DIGITAL
Learn @LEARNING
` 1 Per DayAPP
BROTHERS
PRAKASHAN
Learn @ ` 1 Per Day

N N
HA HA
AS AS
AK AK
PR PR
RS RS
HE HE
OT OT
BR BR
L L
YA YA
O O
G G

To view Sample or Purchase of Books


WWW.goyal-books.com/mtpcx
To view Sample or Purchase of Books
7R YLHZ 6DPSOH RU 3XUFKDVH RI %RRNV 8
www.goyal-books.com/mtpcx
To view Sample or Purchase of Books
:::JR\DOERRNVFRPPWSF[
www.goyal-books.com/mtpcx
28

To view Sample or Purchase of Books
G

WWW.goyal-books.com/mtpcx
To view Sample or Purchase of Books
7R YLHZ 6DPSOH RU 3XUFKDVH RI %RRNV 8
www.goyal-books.com/mtpcx
To view Sample or Purchase of Books
:::JR\DOERRNVFRPPWSF[
www.goyal-books.com/mtpcx

28
GOYAL A DIGITAL LEARNING APP
BROTHERS
PRAKASHAN
Learn @ ` 1 Per Day

MODEL TEST PAPER–3 (Solved)


(Based on the latest CBsE sample Paper)

Time allowed: 2 Hours Maximum Marks: 40

General Instructions:
1. The Question Paper contains THREE sections-READING, WRITING & GRAMMAR and

N
LITERATURE.
2. Attempt questions based on specific instructions for each part.

HA
sECTioN A - REAdiNG (10 marks)

AS
1. Read the passage given below.
1. Ever since I can remember, I always felt I was stupid, really stupid. I lost my father
when I was three. Since I was a sick child, a polio victim, I lived with my grandmother

AK
in Mumbai so I could avail of the metro’s advanced medical facilities. I remember being
on the streets all day, playing or lazing around.
PR
2. When I turned ten, my mother and my sister came to live in Mumbai too. My sister
was a bright student, always securing high grades. She and my mother encouraged me
to study, but I just could not bring myself to concentrate on any subject. I would get
RS
bored and feel sleepy when forced to open my books. After innumerable lectures on the
importance of studying had failed, mother tried threats as well, but nothing worked, I
truly believed I was stupid; no matter how much I worked, it wouldn’t help.
HE

3. I had failed sixth grade and had to repeat the year; Aditi had topped the class and was
the brightest student of the seventh grade too. I flinched at little Aditi as I saw her walk
OT

towards me. ‘Hi,’ she said. ‘Hi,’ I replied, but not with much enthusiasm; I wanted her
to leave. I continued to eat my chips. Ignoring my coldness, she sat down next to me.
For five minutes neither of us spoke and then she asked, ‘Is Devika your real sister?’
BR

‘Yes, she is,’ I answered.


4. ‘So how come she doesn’t teach you so that you too can do well, she asked. I peered
at her, wondering if she was making fun of me, but all I could see on her face was
L

earnestness. No, she is not trying to tease me, I decided. This time, I answered a little
YA

more warmly, ‘She tries to, but I don’t like studying.’ ‘Why not? I’m sure you can get
good grades too,’ she said emphatically. ‘No, I can’t. I have no brains. God forgot to
give me brains, health or beauty. He gave me nothing.’ With the gentlest tone that she
O

could muster, she said, ‘I can prove it to you that you are not stupid. Give me one
G

chance, I will teach you to study.’


5. Though I was softening from inside, I still held on to my low self-esteem. ‘You’ll be
wasting your time. I cannot study, I am neither as bright as you are nor as lucky as
my sister,’ I said. ‘Let me try and help you, please. I will show you the right way to
study. You just have to cooperate with me. Will you try?’ Her belief in me proved more
powerful than my doubts about myself. She won and I relented.

Score Plus CBSE Sample Question Paper with MTPs in English Language & Literature for Class 10 (Term II) 45
GOYAL A DIGITAL LEARNING APP
BROTHERS
PRAKASHAN
Learn @ ` 1 Per Day

Based on your understanding of the passage, answer ANY FiVE questions from the six
given below. 1*5

(i) What does the narrator mean by the term “held on to my low self-esteem”? 1
Ans. By the term “held on to my low self-esteem”, the narrator means the feeling of
incompetence.

(ii) Rewrite the following sentence by replacing the underlined phrase with a word that
means the same from para 3. 1

N
He did not have a feeling of intense enjoyment and eagerness for sports.

HA
Ans. He did not have a feeling of enthusiasm for sports.

(iii) Mention any two things that the narrator felt God had not given her. 1

AS
Ans. The narrator felt that God had forgotten to give her brains, health, or beauty.

(iv) What did the narrator mean by saying, “She won, and I relented”? 1

AK
Ans. The narrator meant that Aditi was able to prevail upon her.

(v) List one impact of the narrator's thought about herself being a stupid person. 1
Ans. The narrator felt that she was stupid. As a result, she failed in sixth grade and repeated
PR
the year.

(vi) How can you say that Aditi was empathetic and responsible towards the narrator? 1
RS

Ans. Aditi was empathetic and responsible towards the narrator because she wanted to help
her and show her the right way to study.
HE

2. Read the passage given below.

The tertiary degree structure of India’s education system is largely patterned after the British
OT

system. At its core, it’s a three-tiered structure comprising bachelor’s degrees, master’s
degrees, and pre-doctoral (master of philosophy) and doctoral programmes. The third level
institutions train specialists who give form and shape to the development process.
BR

The overwhelming majority of India’s students—79.4 per cent—are enrolled in undergraduate


programmes. Enrolments in postgraduate programs are rising fast but so far makeup only
a small fraction of overall enrolments. HRD Ministry releases All India Higher Education
L

Survey. The Gross Enrolment Ratio (GER) in higher education of Indian has registered an
YA

increase from 24.5% in 2015-16 to 25.2% in 2016-17, according to the latest All India Higher
Education Survey (AIHES) released by HRD Ministry.
O

Over the past two decades, the tertiary student population increased sixfold, from 5.7 million
in 1996 to an estimated 36.6 million in 2017-18. The number of universities, likewise, grew
G

from 190 in 1990-91 to 903 in 2017-18, while the number of colleges exploded: 18,000 new
colleges were established between 2008 and 2016 alone—that’s more than six new colleges a
day. HRD Ministry releases All India Higher Education Survey.

46 Score Plus CBSE Sample Question Paper with MTPs in English Language & Literature for Class 10 (Term II)
GOYAL A DIGITAL LEARNING APP
BROTHERS
PRAKASHAN
Learn @ ` 1 Per Day

Basic Structure of Tertiary Education in India

Doctoral Programmes
2–5 Years

M. Phil
1–2 Years
Bachelor’s Degree
Bachelor’s Degree in Medicine
Master’s Degree
5.5 years

N
1–3 Years in Engineering.
Pharmacy,
or Agriculture Dentistry

HA
Bachelor’s Degree 4 years 5 years
3 Years

AS
Entrance Examination (not required for all programmes)

Upper-Secondary Education

AK
12 Years

Types of Higher Education institutions PR


There are five types of institutions with degree-granting authority:
Central universities (Union Universities)— These are established, overseen and funded
RS
mostly through UGC grants. There are presently, 40 central universities directly under the
MHRD.
state universities— There were 370 state universities as of 2017- eligible for UGC grants,
HE

but not all of them receive such grants; instead they derive funding from state governments,
tuition fees, and other sources.
Private universities—These are privately owned institutions, established by federal or state
OT

legislation and referred to as “state private universities.” Private universities are said to operate
like de facto for-profit institutions.
BR

deemed-to-be-universities (124 institutions)—Institutions of high quality, public or private,


federal government grants equal standing with universities via executive order based on the
advice of the UGC.
institutions of National importance (iNis) (91)—“These institutions serve as a pivotal
L

player in developing highly skilled personnel within the specified region of the country/state.”
YA

INIs are specialized, highly selective elite institutions that receive special central government
funding.
O

on the basis of your understanding of the passage, answer ANY FiVE questions from
the six given below. 1*5
G

(i) What does it mean by, “serve as a pivotal player” in education? 1


Ans. It means higher education institutions play an important role in education.
(ii) Based on your understanding of the passage, list the inherent qualities of tertiary
education. 1
Ans. Tertiary education promotes development, closing the urban-rural divide and creates
skilled and trained professionals.
Score Plus CBSE Sample Question Paper with MTPs in English Language & Literature for Class 10 (Term II) 47
GOYAL A DIGITAL LEARNING APP
BROTHERS
PRAKASHAN
Learn @ ` 1 Per Day

(iii) What does the phrase “to operate as de facto-for profit institutions” suggest? 1
Ans. It suggests and describes the practices that exist in reality, even though they are not
officially recognized by law.
(iv) What has the MHRD observed in its survey? 1
Ans. The MHRD has observed in its survey that over the past two decades, the tertiary
student population has increased sixfold.
(v) According to the survey, the enrolments in postgraduate programs are rising fast, but so

N
far make up only a small fraction of overall enrolments. Why can this be so? 1
Ans. The enrolments in postgraduate programs are rising fast, but so far made up only a small

HA
fraction of overall enrolments because higher education is expensive.
(vi) According to the basic structure of tertiary education in India, how long does a student
1

AS
require to complete the Bachelor's Degree in Medicine?
Ans. A student requires 5.5 years to complete the Bachelor's Degree in Medicine.

AK
sECTioN B - WRiTiNG ANd GRAMMAR (10 marks)
3. Attempt ANY oNE from (i) and (ii). PR
(i) A new coaching centre has been opened in your city, and it has made a name for a
very high standard of teaching in a short time. Write a letter inquiring about the courses
taught for Class X, timings, venue and the cost of one-year coaching. You are Bipin
5
RS

Chandra, a resident of Old Ring Road, Hissar, Haryana.


Ans. Sector–24, Old Ring Road
Hissar, Haryana
HE

March 8, 20XX
OT

The Oxford Coaching Institute


Gandhi Road,
BR

Hissar, Haryana

Sir
Subject: Course Inquiry Letter
L
YA

This is regarding the courses offered for Class-Xth. I am a student of Class-Xth and
would like to take coaching in maths, science and English from your reputed institute.
It is known that your institute has made a name for itself in a short span of time with
O

more and more students enrolling on various subjects.


G

I would thus, like to enquire about the courses being taught at your institute, timings,
venue and the cost of one-year coaching.
Please send me the information at the address listed above, so that I can also be a part
of this institute and do well in my exams.

Yours sincerely
Bipin Chandra
48 Score Plus CBSE Sample Question Paper with MTPs in English Language & Literature for Class 10 (Term II)
GOYAL A DIGITAL LEARNING APP
BROTHERS
PRAKASHAN
Learn @ ` 1 Per Day

(ii) Read the given cues that show the number of students who had enrolled in games, that
they termed ‘favourite games’, during the academic year 2019-20 in St Martin’s School,
Manali. Write a paragraph analyzing the following. 5

Cues : turnout of students enrolled—participate—in games of their choice—for


academic year at St Martin’s School, Manali—reveals a larger interest in badminton
and basketball—surprising—abysmal turnout for cricket—most considered game—a hot
favourite among children—seems to appeal—only about 1/8 of the student community—
similar case—attitude towards the national game—hockey—football too seems to fall—

N
into the category—favourites—the percentage—is the same as basketball and badminton

HA
Ans. The given cues represent the turnout of students enrolled to participate in the games of
their choice for this academic year at St Martin’s School, Manali. It is quite clear that
a larger number of students have an interest in the game of badminton and basketball.

AS
What is surprising is an abysmal turnout for cricket! This sport, that is considered a
hot favourite among children these days, seem to have appealed to just about 1/8 of the

AK
student community. The same is the attitude towards the national game of India, Hockey.
Football too seems to fall into the category of favourites as the percentage here is the
same as basketball and badminton.
PR
4. The following paragraph has not been edited. There is one error in each line. Identify the
error and write its correction against the correct blank number. Remember to underline the
correction. The first one has been done for you. 1*3
RS

Error Correction
In urban India, not only is boys much taller than E.g. is are
there grandfathers, even girls have broken the five-foot-nothing (a)
HE

barrier. Though the new generation should enjoy better growth, (b)
its unhealthy lifestyle has a cause for worry. (c)
OT

Ans. Error Correction


(a) there their
BR

(b) should may


(c) has is
5. Read the conversation between Ritu and Reena and complete the passage that follows. 1*2
L

Ritu: Hello, Reena! Do you know that the school trip to Mussoorie has been cancelled?
YA

Reena: No, I didn’t know that. Why has the trip been cancelled?
Ritu: The principal is a little worried about our safety.
O
G

Ritu greeted Reena and asked her if (a) _______________ that the school trip to Mussoorie
had been cancelled. Reena replied that she had not known that. She asked Ritu why the trip
(b) _______________. Ritu told her that the principal was a little worried about their safety.
Ans. (a) she knew
(b) had been cancelled

Score Plus CBSE Sample Question Paper with MTPs in English Language & Literature for Class 10 (Term II) 49
GOYAL A DIGITAL LEARNING APP
BROTHERS
PRAKASHAN
Learn @ ` 1 Per Day

sECTioN C - LiTERATuRE (20 marks)


6. Answer ANY siX questions in 30-40 words each. 2*6
(i) Baking was considered essential in a traditional Goan village. What reasons does the
writer give to support his point? 2
Ans. Baking was considered essential because marriage gifts were meaningless without bol.
Mothers had to prepare sandwiches for their daughter’s engagement. Cakes and bolinhas
were a must for Christmas and other festivals.

N
(ii) What was Valli’s deepest desire? How did she acquire it? 2
Ans. Valli’s deepest desire was to ride on a bus that went past her village. She had acquired it

HA
by standing in her house’s front door and seeing the bus pass through the street.
(iii) What could Amanda do if she were a mermaid? What can you say about the nature of

AS
Amanda? 2
Ans. Amanda imagined herself to be a mermaid in water. She would have ecstatically floated
and enjoyed in the green sea.

AK
She hated to be always poked and instructed by her mother, and she craved freedom and
lived life in her own way. PR
(iv) Why does the poet like animals? 2
Ans. The poet likes animals for their self-contained and quiet nature. The fact that animals are
not like human beings and are satisfied with their lives appeals to the poet a lot.
RS

(v) How did everyone react after Custard ate the pirate? 2
Ans. The dragon hit the pirate hard with his forceful tail and gobbled every bit of him.
HE

Everyone regretted making fun of him and obliged Custard to save their lives. But after
some time, they again started boasting their bravery.
(vi) Do you think Mr Weiherer paid a glowing tribute to Richard? 2
OT

Ans. Mr Weiherer was Ebright’s social studies teacher. He praised him for his brilliant mind,
curiosity, and will to win for the right reason. He also admired Richard for his spirit to
BR

do his very best all the time.


(vii) Why did Matilda feel unfortunate? 2
Ans. Matilda felt unfortunate as she was fond of all the good things in her life, but she could
L

not buy herself any of the luxuries of life. Married to a poor, petty clerk, she always
YA

complained and felt unfortunate.


7. Answer ANY TWo of the following in about 120 words each. 4*2
(i) What was the reason for Matilda’s loss? What changes came into the life of the Loisels
O

after borrowing money? 4


G

Ans. Matilda’s downfall was caused by her unhappiness with whatever life offered her. She
was always in a bad mood and felt as if she had been born for all the luxuries and
delicacies. She had lost her friend’s necklace, and her life became miserable.
Loisels had to borrow a huge amount of money from usurers and the whole race of
lenders to buy a new necklace for Mme Forestier. Loisels had to work day and night to
earn extra income to repay debts. Moreover, Mme Loisel had to sacrifice all the comforts

50 Score Plus CBSE Sample Question Paper with MTPs in English Language & Literature for Class 10 (Term II)
GOYAL A DIGITAL LEARNING APP
BROTHERS
PRAKASHAN
Learn @ ` 1 Per Day

of her life, and M. Loisel had to perform several odd jobs to earn more money. They
sent away the maid, changed their lodging and rented some rooms in the attic.
(ii) Neighbours must have an affectionate relationship which Lomov and Natalya do not
have. Describe the first fight between them. 4
Ans. Lomov and Natalya meet, and instead of proposing to her, he wanders away. He tells her
about how he inherited his estate and admired her father. He also informs Natalya that
his Oxen Meadows are in contact with her birchwoods.

N
When Natalya hears this, she declares that the Oxen Meadows are her family’s property.
They begin to argue over it, and Lomov clarifies that he refers to the Oxen Meadows,

HA
which is located between her birchwoods and Burnt Marsh. Natalya emphasises once
more that they are theirs, but Lomov refuses. He tells her that his aunt’s grandmother
gifted those Meadows to her father’s grandfather. After 40 years of use, the peasants

AS
began to see the land as their own. Natalya’s father arrives, and he fights with Lomov.
They get into a heated disagreement and start calling one other names. Lomov eventually
quits their house.

AK
(iii) How does the Buddha make human beings realize that death is common to all? 4
Ans. According to the Buddha, the life of mortals in this world is troubled and brief. It is
PR
combined with pain. Nobody can avoid dying. After reaching old age, there is death.
Such is the nature of human beings; just as ripe fruits are in danger of falling, mortals
are in danger of death. As all earthen vessels after a certain period break, so is the
RS
life of mortals. All have to die. Only he can get peace of mind, who does not lament,
complain and grieve. Who has overcome sorrow will be free from sorrow and be
blessed.
HE
OT
BR
L
YA
O
G

Score Plus CBSE Sample Question Paper with MTPs in English Language & Literature for Class 10 (Term II) 51
GOYAL A DIGITAL LEARNING APP
BROTHERS
PRAKASHAN
Learn @ ` 1 Per Day

N
HA
AS
AK
PR
RS
HE
OT
BR
L
YA
O
G

27
BROTHERS
PRAKASHAN
GOYAL A DIGITAL
Learn @LEARNING
` 1 Per DayAPP
BROTHERS
PRAKASHAN
Learn @ ` 1 Per Day

N N
HA HA
AS AS
AK AK
PR PR
RS RS
HE HE
OT OT
BR BR
L L
YA YA
O O
G G

To view Sample or Purchase of Books


WWW.goyal-books.com/mtpcx
To view Sample or Purchase of Books
7R YLHZ 6DPSOH RU 3XUFKDVH RI %RRNV 8
www.goyal-books.com/mtpcx
To view Sample or Purchase of Books
:::JR\DOERRNVFRPPWSF[
www.goyal-books.com/mtpcx
28

To view Sample or Purchase of Books
G

WWW.goyal-books.com/mtpcx
To view Sample or Purchase of Books
7R YLHZ 6DPSOH RU 3XUFKDVH RI %RRNV 8
www.goyal-books.com/mtpcx
To view Sample or Purchase of Books
:::JR\DOERRNVFRPPWSF[
www.goyal-books.com/mtpcx

28
GOYAL A DIGITAL LEARNING APP
BROTHERS
PRAKASHAN
Learn @ ` 1 Per Day

MODEL TEST PAPER–4 (Solved)


(Based on the latest CBsE sample Paper)

Time allowed: 2 Hours Maximum Marks: 40

General Instructions:
1. The Question Paper contains THREE sections-READING, WRITING & GRAMMAR and

N
LITERATURE.
2. Attempt questions based on specific instructions for each part.

HA
sECTioN A - REAdiNG (10 marks)

AS
1. Read the passage given below.
1. The main difference between man and animal is the difference in their power of
concentration. All success in any field of work is the result of concentration. Everybody

AK
knows something about concentration. We see its results every day. High achievements
in art, music, etc. are the results of concentration. Those who have trained animals find
much difficulty in the fact that they are constantly forgetting what is told to them. They
PR
cannot concentrate their mind on anything for long at a time. Herein is the difference
between man and animal – the man has the greater power of concentration and also
constitutes the difference between man and man.
2. Everybody’s mind becomes concentrated at times. We all concentrate upon those things
RS

we love, and we love those things upon which we concentrate our minds. Which mother
does not love the face of her homeliest child? That face is to her the most beautiful
HE

in the world. She loves it because she concentrates her mind on it, and if everyone
concentrates their mind on that same face, everyone would love it. It would be to all the
most beautiful face.
OT

3. We all concentrate our minds upon those things we love. When we hear beautiful music,
our mind starts concentrating faster upon it and we cannot stop it. Those who concentrate
their minds on classical music does not like common music, and vice-versa. Music in
BR

which the notes follow each other in rapid succession holds the mind readily. A child
loves lively music because the rapidity of the notes gives the mind no chance to wander.
A man who likes common music dislikes classical music because it is more complicated
and requires a greater degree of concentration to follow it.
L

4. The great trouble with such concentration is that we do not control the mind; it controls
YA

us. Something outside of ourselves, as it was, draws the mind into it and holds it as long
as it chooses. We hear melodious tones or see a beautiful painting, and the mind is held
fast; we cannot take it away.
5. If I speak to you well upon a subject you like, your mind becomes concentrated upon
O

what I’m saying. I draw your mind away from yourself and hold it upon the subject in
spite of yourself. Thus our attention is held, our minds are concentrated upon various
G

things, in spite of ourselves. We cannot help it.


Based on your understanding of the passage, answer ANY FiVE questions from the six
given below. 1*5
(i) What does the writer mean when he says "in spite of ourselves"? 1
Ans. When the writer says, "in spite of ourselves", he means we should try to control our minds.

52 Score Plus CBSE Sample Question Paper with MTPs in English Language & Literature for Class 10 (Term II)
GOYAL A DIGITAL LEARNING APP
BROTHERS
PRAKASHAN
Learn @ ` 1 Per Day

(ii) Rewrite the following sentence by replacing the underlined phrase with a word that
means the same from para 1. 1
In the last week of her course, she focussed completely on her writing and teaching skills.
Ans. In the last week of her course, she concentrated on her writing and teaching skills.
(iii) Describe two characteristics that are apt about the human mind. 1
Ans. Two characteristics that are apt about the human mind are that first, it is a great master
if untamed, and second, it can lead to high achievement.
(iv) What is the actual difference between a man and an animal? 1

N
Ans. The actual difference between man and animal is the degree of concentration.

HA
(v) What does the writer try to explain through the passage? 1
Ans. The writer tries to explain the importance of concentration in our lives.

AS
(vi) How does the writer justify the statement "main difference between man and animal is
the difference in their power of concentration"? 1
Ans. The writer has given several examples to justify the statement like "All success in any

AK
field of work is the result of concentration." He also states that high achievements in art,
music, etc. are the results of concentration.
2. Read the passage given below. PR
India is approaching the ninth month of the coronavirus pandemic with more than five million
confirmed cases— the second-highest in the world after the US— and more than 80,000
reported deaths.
RS

Infection is surging through the country in a “step-ladder spiral”, a government scientist told
me. The only “consolation” is a death rate—currently 1.63%—that’s lower than many
HE

countries with a high caseload. The increase in reported cases has partly to do with increased
testing—but the speed at which the virus is spreading is worrying experts.
OT

Here’s why, it took 170 days for India to


reach the first million cases. The last million
cases took only 11 days. Average daily cases
BR

have shot up from 62 in April to more than


87,000 in September. In the past week, India
has recorded more than 90,000 cases and
1,000 deaths every day. Seven states are worst
L

affected—accounting for about 48% of India’s


YA

population, according to a report released by


Indian Health Ministry.
But even as infections soar, India is opening
O

up - workplaces, public transport, eateries,


gyms – to try to repair a battered economy
G

suffering its worst slump in decades.


The world’s most draconian lockdown forced
people to stay at home, shut businesses and
triggered an exodus of millions of informal
workers who lost their jobs in the cities and
returned home on foot, buses and trains.

Score Plus CBSE Sample Question Paper with MTPs in English Language & Literature for Class 10 (Term II) 53
GOYAL A DIGITAL LEARNING APP
BROTHERS
PRAKASHAN
Learn @ ` 1 Per Day

But the resumption of economic activity, even as cases spiral, suggests “lockdown fatigue”,
the Nomura India Business Resumption Index says.
More than 50 million Indians have been tested so far for the virus, and more than a million
samples are being tested daily. But the country still has one of the lowest testing rates in the
world. So epidemiologists suggest that India’s real infection rates are far higher.
The government’s own antibody tests on a random sample of people nationwide estimate 6.4
million infections in early May, as compared to the recorded case count of 52,000 around that
time.

N
Bhramar Mukherjee, a professor of biostatistics and epidemiology at the University of

HA
Michigan, who has been closely tracking the pandemic, says her models point to about 100
million infections in India now. “I think India has taken a path of cruising towards herd
immunity. I am not sure whether everyone is following preventive measures like wearing

AS
masks and keeping social distancing seriously,” she told me. Herd immunity is achieved
when enough people become immune to a virus to stop its spread. “This could be due to
habituation, desensitization [to the disease], fatigue, denial, fatalism or a combination of both.

AK
It feels like a thousand deaths a day have become normal.”
(courtesy by BBC, https://www.bbc.com/news/world-asia-india-54176375)
on the basis of your understanding of the passage, answer ANY FiVE questions from
PR
the six given below. 1*5
(i) Why has the expression “draconian lockdown” been used? 1
Ans. The expression “draconian lockdown” has been used to refer to the excessively harsh and
RS

severe conditions caused by the lockdown.


(ii) Comment on the significance of Bhramar Mukherjee’s, a professor of biostatistics and
HE

epidemiology at the University of Michigan, models. 1


Ans. Bhramar Mukherjee’s models pointed out that Indians are becoming immune to the virus.
According to her, India had taken the path of cruising towards herd immunity.
OT

(iii) Justify Professor Mukherjee’s observation—“It feels like a thousand deaths a day have
become normal” with reference to the last paragraph of the report. 1
BR

Ans. According to Professor Mukherjee’s observation, “Herd immunity is achieved when


enough people become immune to a virus to stop its spread.” Therefore, the spread will
automatically lead to a thousand deaths a day, which seemed normal then.
(iv) In comparison to the other countries, what could be a suitable reason for the increased
L

number of cases? 1
YA

Ans. In India, the lowest testing rates, as compared to the other countries, could be a suitable
reason for the increased number of cases.
(v) “India has become the second-highest in the world after the US.” How does the evidence
O

from the survey report support this statement? 1


Ans. Seven Indian states were the worst affected, accounting for about 48% of India’s population,
G

according to a report released by the Indian Health Ministry.


(vi) What did the Indian Health Ministry, in a report, share about Tamil Nadu’s condition
regarding the spread of the coronavirus? 1
Ans. The Indian Health Ministry, in a report, said that Tamil Nadu was about to touch 10 lakh
cases.

54 Score Plus CBSE Sample Question Paper with MTPs in English Language & Literature for Class 10 (Term II)
GOYAL A DIGITAL LEARNING APP
BROTHERS
PRAKASHAN
Learn @ ` 1 Per Day

sECTioN B - WRiTiNG ANd GRAMMAR (10 marks)


3. Attempt ANY oNE from (i) and (ii).
(i) Read the given cues that show how youngsters prefer to access the internet at home and
other places. Analyze the cues to write an analytical paragraph. 5
Cues : survey—conducted on groups of youngsters—in various cities—regarding use of
the internet—over their preferred devices—question referred to going online—home and
in other places—four devices—majorly found—in answers—smartphone, tablet, laptop

N
and desktop computer—clear—majority of youth prefers—laptop and smartphone—
few people—access internet on PC and tablet—only a small number of people use

HA
internet—other than four devices—concluded—portable devices—more popular among
youth—as can be used—outside homes also—least popular—desktop—as need to stay
indoors for accessing the internet

AS
Ans. A survey was conducted on groups of youngsters in various cities regarding the use of
internet over their preferred devices. The question referred to going online at home and in
other places. Four devices were majorly found in their answers: smartphone, tablet, laptop

AK
and desktop computer. It can be clearly seen that the majority of the youth prefers to use
laptop and smartphone. There are few people who access the internet on PC and tablet. Only
a small number of people use the internet other than these four devices. It can be concluded
PR
that portable devices are more popular in the youth, as they can use it outside their homes
also. The least popular is desktop as you need to stay indoors for accessing the internet.
(ii) You are Poorvi/Prem, residing at 52, Moti Nagar, Lucknow. You need accommodation for
RS
yourself. Write a letter to the Manager of paying guest service, 29, Aashiana Complex,
Maharani Bagh, Delhi, enquiring about the details such as accommodation type, monthly
charges, facilities provided, etc. 5
HE

Ans. 52, Moti Nagar


Lucknow, U.P.
OT

January 12, 20XX

The Manager
BR

Paying Guest Service


29, Aashiana Complex
Maharani Bagh
L

Delhi
YA

Sir
Subject: Enquiring about Accommodation and Other Facilities
O

I am writing to enquire about the availability of accommodation for students in your


complex. I am a student of B.Tech at Jaypee Institute of Technology. I am looking for
G

a suitable place in and around the Maharani Bagh area to relocate there. Kindly provide
me with the information and charges of the following:
1. Rent
2. Meals
3. Transportation facility
I shall be highly grateful if you respond promptly as currently, I live with some distant
relative.

Score Plus CBSE Sample Question Paper with MTPs in English Language & Literature for Class 10 (Term II) 55
GOYAL A DIGITAL LEARNING APP
BROTHERS
PRAKASHAN
Learn @ ` 1 Per Day

I am looking forward to hearing from you soon.

Yours truly
Poorvi
4. The following paragraph has not been edited. There is one error in each line. Identify the
error and write its correction against the correct blank number. Remember to underline the
correction. The first one has been done for you. 1*3
Error Correction

N
The earth is one of the eight planet that E.g. planet planets
go around the sun. In ancient times, the people (a)

HA
who study the stars noticed that while certain (b)
heavenly bodies seemed fixed in a sky, others (c)
seemed to move around.

AS
Ans. Error Correction
(a) go goes

AK
(b) study studied
(c) a the
5. Read the conversation between a father and his son and complete the passage that follows.1*2
Father: Is tomorrow a holiday?
PR
son: Yes, it is Sunday, so it is a holiday.
Father asked the son (a) ______________. The son replied (b) ______________ and added
RS

that being Sunday, it was a holiday.


Ans. (a) if the next day was a holiday
(b) in affirmation
HE

sECTioN C - LiTERATuRE (20 marks)


OT

6. Answer ANY siX questions in 30-40 words each. 2*6


(i) What is the story of the Arabs about the descent of Kodavu people? 2
Ans. The theory of Arab origin draws support from the long, black coat with an embroidered
BR

waist-belt worn by the Kodavus. Known as kuppia, it resembles the kuffia worn by the
Arabs and the Kurds.
(ii) How did Rajvir describe the view from the train? 2
L

Ans. Rajvir discusses the breathtaking scenery from the train window. It was a sea of tea
YA

bushes flitting against the densely forested hillsides. Tall shade trees emerged at random
intervals, and women tea-pluckers, who appeared to be doll-like creatures, could be seen
collecting tea leaves.
O

(iii) Do you find the title ‘Amanda’ appropriate? Give a reason. 2


Ans. Amanda is the protagonist of this poet’s narration. I believe that the title is quite
G

appropriate as the poem’s theme revolves around the central character, Amanda.
(iv) Why do animals not weep for their sins? 2
Ans. Animals do not weep for their sins because they do not need to do so. They are innocent
creatures that commit no sins. It is humans who weep yet commit sins.
(v) Pen down the appearance of the pirate. Was the pirate killed? 2
Ans. The pirate looked very terrible. He had two pistols, one in each hand. He had a black beard,

56 Score Plus CBSE Sample Question Paper with MTPs in English Language & Literature for Class 10 (Term II)
GOYAL A DIGITAL LEARNING APP
BROTHERS
PRAKASHAN
Learn @ ` 1 Per Day

and one of his legs was made of wood. Indeed, amid his teeth, he was holding a cutlass.
Yes, the pirate was killed by Custard, who pounced upon the pirate and ate him at once.
(vi) Why did Matilda throw the invitation spitefully? 2
Ans. When Matilda’s husband handed her the invitation, she was irritated. She felt humiliated
and refused the invitation because she lacked something lovely to wear to a fancy party.
(vii) Was the lawyer unhappy about his job? Why? 2
Ans. After his graduation, the lawyer joined the law firm as a junior assistant clerk. He was
unhappy about the job as sometimes he was beaten up, and he was often given a job of

N
serving the summons.

HA
7. Answer ANY TWo of the following in about 120 words each. 4*2
(i) The age-old wisdom ‘forgive and forget’ always comes in handy in maintaining cordiality
in relationships.

AS
Imagine yourself to be the playwright of this play. Based on your understanding of
Lomov and Natalya's character traits, write how the play ‘The Proposal’ brings out the
truth in the statement. 4

AK
Ans. Initially, the neighbours in the play, Lomov, Natalya and Chubukov, fought over petty issues.
They put essential concerns aside and spend their time bickering about trivial issues.
These disagreements eventually turned into fights, and they began screaming insults at
PR
one another, which strained their already tense relationship. When Natalya learned that
Lomov had come to propose to her, she immediately abandoned the subject of their
previous disagreement (Oxen Meadows). She also appealed for Lomov's forgiveness and
RS

pleaded with him to get to the heart of the matter.


As a result, if one wants to maintain a friendly connection with others, it is necessary to
"forgive and forget."
HE

(ii) In life, people who easily trust others are sometimes made to look foolish. One should
not be too trusting. Describe how Oliver Lutkins made a fool of the young lawyer. 4
Ans. During his first visit to the village, Oliver Lutkins made a fool of the young lawyer.
OT

He identified himself as Bill at the railway station and told the lawyer that they would
search for Lutkins together. He told the lawyer that he was familiar with most of
BR

Lutkins’ hangout spots. He escorted the narrator to Fritz, the barber’s business, Gray’s
shop, and finally Lutkins’ mother, whom he described as awful. He cheated the lawyer
throughout and made money by driving him around. As a result, he was able to fool
the lawyer by taking him all over the village without success because the lawyer didn’t
L

recognise Lutkins and thus didn’t realise Lutkins himself was following him.
YA

(iii) Bholi was believed to be a ‘dumb cow’. Do you think so? How did she transform herself
into a bold and confident girl? 4
Ans. Bholi was seen as a dumb cow. She was sent to school solely to carry out Tehsildar’s
O

commands.
Her teacher was quite fond of her when she first started school and encouraged her to
G

succeed, which helped her build confidence. Her teacher was effective in transforming
her name from Bholi to Sulekha. She had grown into a self-assured, educated young
lady, but no one recognised the change in her demeanour because she was no longer a
member of her family. Because of her disfigured features, her parents wanted to marry
her off to a man twice her age as she grew older. She agreed in the way that an obedient
daughter would. She protested against the selfish groom’s demand for a large dowry
because of her education, which had given her free will, wisdom, and maturity.
Score Plus CBSE Sample Question Paper with MTPs in English Language & Literature for Class 10 (Term II) 57
GOYAL A DIGITAL LEARNING APP
BROTHERS
PRAKASHAN
Learn @ ` 1 Per Day

N
HA
AS
AK
PR
RS
HE
OT
BR
L
YA
O
G

27
BROTHERS
PRAKASHAN
GOYAL A DIGITAL
Learn @LEARNING
` 1 Per DayAPP
BROTHERS
PRAKASHAN
Learn @ ` 1 Per Day

N N
HA HA
AS AS
AK AK
PR PR
RS RS
HE HE
OT OT
BR BR
L L
YA YA
O O
G G

To view Sample or Purchase of Books


WWW.goyal-books.com/mtpcx
To view Sample or Purchase of Books
7R YLHZ 6DPSOH RU 3XUFKDVH RI %RRNV 8
www.goyal-books.com/mtpcx
To view Sample or Purchase of Books
:::JR\DOERRNVFRPPWSF[
www.goyal-books.com/mtpcx
28

To view Sample or Purchase of Books
G

WWW.goyal-books.com/mtpcx
To view Sample or Purchase of Books
7R YLHZ 6DPSOH RU 3XUFKDVH RI %RRNV 8
www.goyal-books.com/mtpcx
To view Sample or Purchase of Books
:::JR\DOERRNVFRPPWSF[
www.goyal-books.com/mtpcx

28
GOYAL A DIGITAL LEARNING APP
BROTHERS
PRAKASHAN
Learn @ ` 1 Per Day

MODEL TEST PAPER–5 (Solved)


(Based on the latest CBsE sample Paper)

Time allowed: 2 Hours Maximum Marks: 40

General Instructions:
1. The Question Paper contains THREE sections-READING, WRITING & GRAMMAR and

N
LITERATURE.
2. Attempt questions based on specific instructions for each part.

HA
sECTioN A - REAdiNG (10 marks)

AS
1. Read the passage given below.

The colour of animals is by no means a matter of chance; it depends on many considerations, but

AK
in the majority of cases tends to protect the animal from danger by rendering it less conspicuous.
Perhaps it may be said that if colouring is mainly protective, there ought to be but few brightly
coloured animals. There are, however, not a few cases in which vivid colours are themselves
PR
protective. The kingfisher itself, though so brightly coloured, is by no means easy to see. The
blue harmonizes with the water, and the bird as it darts along the stream looks almost like a
flash of sunlight.
RS

Desert animals are generally the colour of the desert. Thus, for instance, the lion, the antelope,
and the wild donkey are all sand-coloured. “Indeed,” says Canon Tristram, “in the desert,
HE

where neither trees, brushwood, nor even undulation of the surface affords the slightest
protection to its foes, a modification of colour assimilated to that of the surrounding country
is absolutely necessary. Hence, without exception, the upper plumage of every bird, and
OT

also the fur of all the smaller mammals and the skin of all the snakes and lizards, is of one
uniform sand colour.”
BR

The next point is the colour of the mature caterpillars, some of which are brown. This
probably makes the caterpillar even more conspicuous among the green leaves than would
otherwise be the case. Let us see, then, whether the habits of the insect will throw any
light upon the riddle. What would you do if you were a big caterpillar? Why, like most
L

other defenceless creatures, you would feed by night, and lie concealed by day. So do these
YA

caterpillars. When the morning light comes, they creep down the stem of the food plant and
lie concealed among the thick herbage and dry sticks and leaves, near the ground, and it is
obvious that under such circumstances the brown colour really becomes protection. It might
O

indeed be argued that the caterpillars, having become brown, concealed themselves on the
G

ground and that we were reversing the state of things. But this is not so, because, while we
may say as a general rule that large caterpillars feed by night and lie concealed by day, it
is by no means always the case that they are brown; some of them still retaining the green
colour. We may then conclude that the habit of concealing themselves by day came first and
that the brown colour is a later adaptation.

58 Score Plus CBSE Sample Question Paper with MTPs in English Language & Literature for Class 10 (Term II)
GOYAL A DIGITAL LEARNING APP
BROTHERS
PRAKASHAN
Learn @ ` 1 Per Day

Based on your understanding of the passage, answer ANY FiVE questions


from the six given below. 1*5
(i) Which creatures does the author refer to by using the phrase “other defenceless
creatures”? 1
Ans. The author refers to creatures that can be attacked and harmed easily.
(ii) Rewrite the following sentence by replacing the underlined phrase with a word that
means the same from para 1. 1

N
Most people don’t want to be too easily noticeable or seen.
Ans. Most people don’t want to be too conspicuous.

HA
(iii) Describe two special features of caterpillars that save themselves from being prey. 1
Ans. The two special features of caterpillars that save themselves from being prey are that

AS
they feed by night and lie concealed by day.
(iv) Why is the colour of animals a type of boon for them? 1

AK
Ans. The colour of animals is a type of boon for them because it helps them to protect
themselves.
(v) List one factor that shows that kingfishers cannot be seen easily.
PR 1
Ans. One of the factors that shows that kingfishers cannot be seen easily is that their blue
colour harmonizes with the water.
(vi) What does the author mean when he says, “a matter of chance”? 1
RS

Ans. The author means something that happens unpredictably and without obvious intention.
2. Read the passage given below.
HE

ACC Limited is India’s foremost manufacturer of cement and ready mixed concrete with a
countrywide network of factories and sales offices. Established in 1936, ACC is acknowledged
OT

as a pioneer and trendsetter in cement and concrete technology. The quality of its products
and customer services make ACC the most preferred brand in the Indian cement industry.
ACC Limited is part of the worldwide Holcim Group.
BR

Table: disclosure of Human Resource Variables in ACC Limited


HR Variable 2009 2010 2011 2012 2013 Total %
1. Age wise distribution 0 0 0 0 0 0 0 .00
L

2. Awards and Rewards given for good 1 1 1 1 1 5 100.00


YA

performance
3. Brand Valuation 1 1 1 1 1 5 100.00
4. Capital Employed 1 1 1 1 1 5 100.00
O

5. Discount rate used 0 0 0 0 1 1 20.00


G

6. Economic Value Added (EVA) 0 1 1 1 1 4 80.00


7. Employee Cost 0 0 0 0 0 0 0.00
8. Employee Remuneration benefit 0 0 0 0 0 0 0.00
9. Gender wise distribution 0 0 0 0 0 0 0.00
10. Group wise distribution 0 0 0 0 0 0 0.00
11. Human capital in balance sheet 0 0 0 0 0 0 0.00

Score Plus CBSE Sample Question Paper with MTPs in English Language & Literature for Class 10 (Term II) 59
GOYAL A DIGITAL LEARNING APP
BROTHERS
PRAKASHAN
Learn @ ` 1 Per Day

12. Human Resources 0 0 0 0 0 0 0.00


Development Fund
13. Man power in numbers 1 1 1 1 1 5 100.00
14. Qualification of the employees 1 1 1 1 1 5 100.00
15. Recruitment cost 0 0 0 0 0 0 0.00
16. Retirement Benefits 1 1 1 1 1 5 100.00
17. Separate HRA statement 0 0 0 0 0 0 0.00

N
18. Ten years at a glance 1 1 1 1 1 5 100.00
19. Training and development expense 1 1 1 1 1 5 100.00

HA
20. Turnover per employee 0 0 0 0 0 0 0.00
21. Valuation Model use 0 0 0 0 0 0 0.00
22. Value Added 0 0 0 0 0 0 0.00

AS
23. Value Added per employee 0 0 0 0 0 0 0.00
24. Value of Human Resources 0 0 0 0 0 0 0.00

AK
Total 8 9 9 9 10 45 37.50
Table shows discloser of human resource variables in the company’s annual reports from 2009
to 2013. Table shows disclosure of particular variable in particular variable and 0 shows non-
PR
disclosure of that variable in that particular year.
The first variable was Age wise distribution. From year 2009 to years 2013 ACC limited never
disclosed this variable, hence in five years this variable’s discloser rate was 0%.
RS

The next HR variable was Award and reward given for good performance. ACC Ltd. disclosed
this variable in all the five years of study i.e. from 2009 to 2013 and its discloser rate was 100%.
HE

The other variables can be discussed in the same way as the above two variables are discussed.
on the basis of your understanding of the passage, answer ANY FiVE questions from
the six given below. 1*5
OT

(i) Why has ACC Limited become a part of the Holcim Group? 1
Ans. ACC Limited has become a part of the Holcim Group because it has proved itself for its
BR

quality and customer satisfaction.


(ii) Comment on the personal benefits of the employees as per your understanding of the
report. 1
L

Ans. The personal benefits of the employees are employee remuneration, training and
YA

development benefits, and retirement benefits.


(iii) Justify how, in five years, the age-wise distribution variables' discloser rate was 0%. 1
Ans. From the year 2009 to 2013, ACC Limited never disclosed this variable. Hence, this
O

variable's discloser rate was 0%.


G

(iv) Which factor is the most powerful tool to keep a check on the employees’ consistency
and performance? 1
Ans. The most powerful tool to keep a check on the employees’ consistency and performance
is Training and Development.
(v) What does the report show about the performance of ACC Limited? 1
Ans. The report clearly shows that the overall performance of ACC Limited is good.

60 Score Plus CBSE Sample Question Paper with MTPs in English Language & Literature for Class 10 (Term II)
GOYAL A DIGITAL LEARNING APP
BROTHERS
PRAKASHAN
Learn @ ` 1 Per Day

(vi) According to the given report, what was the five-year average rate of human resource
variable disclosure? 1
Ans. The five-year average rate of human resource variable disclosure was 37.50%.

sECTioN B - WRiTiNG ANd GRAMMAR (10 marks)


3. Attempt ANY oNE from (i) and (ii).
(i) Read the following excerpt from an article that appeared in the magazine section of a

N
local daily:
According to the NCRB data, the total crimes on the register were 2.97 million, while the

HA
crime rate figured to be 379 crimes per lakh population. The year 2018 has witnessed
some of the most horrific cases of crime. The rape and murder incident of Kathua just
petrified the entire nation. Similarly, the Unnao incident also sent shivers up everyone’s

AS
spine.
Write a paragraph to analyze the given argument.

AK
You could think about what alternative explanations might weaken the given conclusion
and include rationale/evidence that would strengthen/counter the given argument. 5
Ans. Wherever we go, we hear the cases of crimes in every nook and corner of society. Not
PR
only this but the newspapers and TV are also full of crimes cases. According to the
recent survey by NCRB, the total crimes registered were 2.97 million, while the crime
rate figured to be 379 crimes per lakh population. This data has made everyone shocked
RS
and created an environment of terror.
The year 2018 has been witnessed as one of the most horrible years as many crimes
have been witnessed this year. The cases of rape, murder, theft, robbery, killing and
HE

many more have been registered in considerable numbers. The nation needs protection
and safety to safeguard the citizens of the country. The rape and the murder incidents
happened in Kathua petrified the entire nation with terror and concern and, similarly
OT

the cases of Unnao. The incidents registered in Unnao also froze everyone’s spine. The
government needs to take serious steps against these issues as one of the reasons for
the rising crime is unemployment and the increasing cases. Only the government can
BR

lend his helping hand by reducing poverty and providing employment chances to all the
needy, which will ultimately help in reducing the cases of crimes.
(ii) You are Abhay/Anubhuti of 160, Daryaganj, Delhi. You wish to do a short term course
L

on personality development. Write a letter in 100-120 words to the Director, Personal


YA

Care, Lajpat Nagar, Delhi, enquiring about the course details. 5


Ans. 160, Daryaganj
O

Delhi
G

April 25, 20XX

The Director
Personal Care
Lajpat Nagar
Delhi—110024

Score Plus CBSE Sample Question Paper with MTPs in English Language & Literature for Class 10 (Term II) 61
GOYAL A DIGITAL LEARNING APP
BROTHERS
PRAKASHAN
Learn @ ` 1 Per Day

Sir
Subject: Enquiring about Personality Development Course
I am planning to go abroad for pursuing my MBA. I am well aware that fluency in
reading, writing, listening, and, most importantly, speaking is a prerequisite for applying
to any university abroad.
Through various sources, I have come to know that yours is a very reputed institute for
grooming personality and public speaking. I am keen to join your institute as soon as my

N
classes begin in August this year. Kindly furnish me with the following details:

HA
1. Fee structure
2. Timing and duration of the course
3. Demo classes

AS
4. Online assistance provided by the institute
5. Morning and evening batches

AK
I am looking forward to a prompt reply.

Yours truly
PR
Abhay
4. The following paragraph has not been edited. There is one error in each line. Identify the
RS

error and write its correction against the correct blank number. Remember to underline the
correction. The first one has been done for you. 1*3
Error Correction
HE

The earthquake begin with side movements which knocked E.g. begin begins
down all person who was standing. Then, there (a)
OT

were up and down movement, a vast rumbling and reverberating (b)


noises as of an artillery bombardment or a hundred aeroplanes. (c)
Ans. Error Correction
BR

(a) all every


(b) were was
L

(c) noises noise


YA

5. Read the conversation between two friends and complete the passage that follows. 1*2
I said to my friend, “Don’t talk of the past. Is there nothing else to talk about? Remember,
O

there is so much to learn and so much to forget.”


I advised my friend (a) ______________ and asked if there was nothing else to talk about. I
G

further reminded him (b) ________________ and so much to forget.


Ans. (a) not to talk of the past
(b) that there was so much to learn

62 Score Plus CBSE Sample Question Paper with MTPs in English Language & Literature for Class 10 (Term II)
GOYAL A DIGITAL LEARNING APP
BROTHERS
PRAKASHAN
Learn @ ` 1 Per Day

sECTioN C - LiTERATuRE (20 marks)


6. Answer ANY siX questions in 30-40 words each. 2*6
(i) How did Valli gather all the information about the bus? 2
Ans. Over many days and months, Valli listened carefully to conversations between her
neighbours and people who regularly used the bus. She also asked a few discreet
questions here and there.
(ii) Amanda doesn’t like to be controlled and instructed on what to do or not. Comment. 2

N
Ans. Amanda is a child, and she does not like to be controlled. Whenever we try to control a

HA
child, he/she feels that his/her freedom is being curtailed. A child can learn much more
when freedom is given to him/her.
(iii) What does the poet mean by, “Not one is respectable or unhappy over the whole earth”? 2

AS
Ans. The poet means that animals do not pray to God or ancestors, and all of them are equal.
Hence, no one is more respectable than the other. The good values of animals and the
lack of social conventions make them happy.

AK
(iv) Do you think that one should be made fun of because of their preferences and choices in
life? Explain in the context of Custard, the dragon. 2
PR
Ans. It is not right to make fun of anyone based on their lifestyle and choices. Custard, the
dragon, always wanted comfort and safety, and always cried for a nice safe cage. Belinda
and other pets made fun of him because they thought he was a coward. Custard proved
RS
that he is not a coward just because he likes comfort. He was the only one who dared to
face the pirate and killed him.
(v) How can one attain peace? Does the lesson teach you something? Explain in your own
HE

words the theme of the lesson ‘The Sermon at Benaras’. 2


Ans. One can attain peace by renunciation, i.e., giving up all material gains. One who can
OT

overcome one’s greed, grief and lust can obtain peace.


This lesson teaches us through the sermon of Lord Buddha. The sermon teaches us that
we all are mortals. So we will die one day.
BR

(vi) Identify four values that Richard Ebright projected as a man of substance. 2
Ans. Ebright was a sharp observer as well as a brilliant scientist. He was a great debater and
public speaker. He was a professional photographer who specialised in woo and scientific
L

shows.
YA

(vii) Why was Sulekha nicknamed Bholi? 2


Ans. When Sulekha was a 10-month-old infant, she had fallen off the cot. Perhaps, it had
O

damaged some part of her brain. This incident made her different and a backward child
from the rest of her siblings. Thus, she was nicknamed Bholi, the simpleton.
G

7. Answer ANY TWo of the following in about 120 words each. 4*2
(i) “Don’t you worry, Pitaji! In your old age, I will serve you and mother.” The narrator
wants to highlight the moral values Bholi was imbued with through the statement. Based
on the reading of the lesson, what made Bholi aware of her rights, and how did she use
them? 4

Score Plus CBSE Sample Question Paper with MTPs in English Language & Literature for Class 10 (Term II) 63
GOYAL A DIGITAL LEARNING APP
BROTHERS
PRAKASHAN
Learn @ ` 1 Per Day

Ans. The story of ‘Bholi’ highlights social prejudice against women, in which women are
viewed as a liability, a weight to be carried until they are eventually married off.
Ramlal’s daughter Bholi was a naive young woman. Her father fixed her marriage with
Bishamber Nath, an elderly man with grown children. Bishamber demanded a huge
dowry, and Bholi showed courage by refusing to marry such a cruel and greedy man.
Bholi promised to look after her parents as they grew older.
She grew mature and confident as a result of her education and her teacher’s efforts. She
was made aware of her legal rights. Bholi’s education made her aware of her rights to

N
speak out against dowry.

HA
(ii) "And that blind hen, yes, that turnip ghost has the confounded cheek to make a proposal,
and so on!"
Who is the speaker? About whom is the above line said? Why? How does Natalya react

AS
when she comes to know about the proposal? 4
Ans. Chubukov is the speaker.

AK
Chubukov says the above line about his neighbour, Lomov.
Lomov goes to Chubukov's place to ask Natalya for her hand in marriage.
However, he quickly becomes engaged in a dispute with Natalya and later Chubukov
PR
over the ownership of Oxen Meadows. As they abuse each other, the quarrel becomes
terrible. Lomov leaves Chubukov's house at a given point in time. Chubukov then says
those words.
RS

When Natalya learns that Lomov has come to their house to propose to her, she is taken
aback and immediately sends his father to bring Lomov back to their house, as she is
HE

anxious to marry Lomov.


(iii) The people and surroundings are a great book to learn. In the lesson ‘Madam Rides the
Bus’, Valli learns a lot from others. Mention the traits of her character that help her learn
OT

from her surroundings. 4


Ans. Valli had both pleasant and unpleasant experiences during her first bus journey. She had
some new experiences during this journey. At the railroad crossing, she saw a train at
BR

some distance. It grew bigger as it came near. While passing through a street, she saw
big and bright shops. They displayed clothes and other goods. However, Valli was greatly
amused when she saw a young cow running in front of the bus. When the driver blew
L

the horn, the cow started running faster. This made Valli laugh. The most unpleasant
YA

thing for Valli was the sight of the same cow lying dead by the roadside. It made her
very sad.
O
G

64 Score Plus CBSE Sample Question Paper with MTPs in English Language & Literature for Class 10 (Term II)
GOYAL A DIGITAL LEARNING APP
BROTHERS
PRAKASHAN
Learn @ ` 1 Per Day

N
HA
AS
AK
PR
RS
HE
OT
BR
L
YA
O
G

27
BROTHERS
PRAKASHAN
GOYAL A DIGITAL
Learn @LEARNING
` 1 Per DayAPP
BROTHERS
PRAKASHAN
Learn @ ` 1 Per Day

N N
HA HA
AS AS
AK AK
PR PR
RS RS
HE HE
OT OT
BR BR
L L
YA YA
O O
G G

To view Sample or Purchase of Books


WWW.goyal-books.com/mtpcx
To view Sample or Purchase of Books
7R YLHZ 6DPSOH RU 3XUFKDVH RI %RRNV 8
www.goyal-books.com/mtpcx
To view Sample or Purchase of Books
:::JR\DOERRNVFRPPWSF[
www.goyal-books.com/mtpcx
28

To view Sample or Purchase of Books
G

WWW.goyal-books.com/mtpcx
To view Sample or Purchase of Books
7R YLHZ 6DPSOH RU 3XUFKDVH RI %RRNV 8
www.goyal-books.com/mtpcx
To view Sample or Purchase of Books
:::JR\DOERRNVFRPPWSF[
www.goyal-books.com/mtpcx

28
GOYAL A DIGITAL LEARNING APP
BROTHERS
PRAKASHAN
Learn @ ` 1 Per Day

MODEL TEST PAPER–6 (Solved)


(Based on the latest CBsE sample Paper)

Time allowed: 2 Hours Maximum Marks: 40

General Instructions:
1. The Question Paper contains THREE sections-READING, WRITING & GRAMMAR and

N
LITERATURE.
2. Attempt questions based on specific instructions for each part.

HA
sECTioN A - REAdiNG (10 marks)

AS
1. Read the passage given below.
I know poverty and misery and I quite appreciate, by personal experience, what it is to be

AK
poor, what it is to have no clothes, what it is to have no books, what it is to struggle through
life, what it is to walk through the streets without an umbrella, without conveyance along
miles on dusty roads. I have been through it all and I can understand the difficulties that most
PR
of you graduates have to face today. I am speaking from a long experience of 60 years. Please
do not imagine that all the 60 years are milk and roses. To be able to accomplish something, I
want to tell you that you have to go through such experiences.
RS

I admit success in life is not always to be intelligent or be strong, and it is to some extent
a bit of a gamble, but those who got their minds right and those who know their job sooner
or later, will sooner perhaps than later make their way in life. But they should not be
HE

disappointed if they do not, they have to face up life and take it as they find it.
What I say is that the great things in life are not really great. What I love is to enjoy the
OT

common things of life. I am happy that I am still able to sleep at night provided I have three
miles walk in the evening. I am still able to enjoy a good lunch or a good dinner. I am still
able to look at the blue sky and like it. I still like to walk in the open fields and like the
BR

smell of the ragi or the jowar.


We think that happiness consists of going to theatres and seeing thrilling films and dramas.
Not at all, the great things in life are the God-given things which cost nothing. What you need
L

is the desire to appreciate them. If you have your minds and hearts open, you have around you
YA

things which give you joy. There is the butterfly jumping about in flourishing colours on all
sides. Look at the wonderful thing that God has given for our enjoyment.
O

We have to love nature and appreciate nature and appreciate her wonderful gifts, of nature’s
marvellous ingenuity, its resourcefulness, and infinite variety. It is the same thing that has
G

inspired me all my life.


Based on your understanding of the passage, answer ANY FiVE questions from the six
given below. 1*5
(i) What does the writer mean by the phrase “nature’s marvellous ingenuity”? 1
Ans. The writer here refers to nature’s amazing skill at solving our problems.

Score Plus CBSE Sample Question Paper with MTPs in English Language & Literature for Class 10 (Term II) 65
GOYAL A DIGITAL LEARNING APP
BROTHERS
PRAKASHAN
Learn @ ` 1 Per Day

(ii) Rewrite the following sentence by replacing the underlined phrase with a word that
means the same from para 3. 1
Shaan will buy the medicines on the condition that he has enough money.
Ans. Shaan will buy the medicines provided he has enough money.
(iii) Describe two aspects of nature that the writer wants us to appreciate and love. 1
Ans. Two factors that the writer wants us to appreciate and love about nature are its
resourcefulness and its infinite variety.

N
(iv) Why is life called a gamble? 1

HA
Ans. Life is called a gamble because even an intelligent or strong person is unsure of success.
(v) List one fact through which the writer wants us to understand that happiness doesn’t lie in
materialistic things. 1

AS
Ans. The fact through which the writer wants us to understand that happiness doesn’t lie
in materialistic things is that all the great things in life are God-given things that cost
nothing.

AK
(vi) How does the writer justify the words “What you need is the desire to appreciate God-
given things”? 1
PR
Ans. The writer here takes the example of a butterfly, jumping about in flourishing colours on
all sides. He/she wants us to love and appreciate the wonderful gifts of nature.
2. Read the passage given below.
RS

The nationwide 21-day lockdown from March 25, 2020 suspended domestic flights, trains
and inter-state buses, and instructed all Indians to stay at home. The government had already
HE

restricted foreign arrivals earlier in March 2020, first suspending tourist visas and visa-
free travel for those holding Overseas Citizen of India cards, then prohibiting arrivals from
COVID-19 hotspots, and finally cancelling all international flight landings from March 22,
OT

2020, about 700 a week.


The government called for suspension of all hospitality services with exceptions only for
BR

those who were accommodating tourists and people stranded due to the lockdown, and those
designated as quarantine facilities.
Thus, foreign tourist arrivals in India, which saw a 9% decline in February 2020 (1.02
L

million) over January 2020 (1.12 million), and a 7% fall compared to February 2019 (1.09
million), have plummeted in March 2020. The Indian tourism and travel industry is likely to
YA

suffer a loss of 70% of its total workforce, said an April 2020 KPMG report.
Likewise, occupancies across hotels in India have crashed during March 2020, by over 40%
O

in many cases; and cancellations are at an all-time high, said a March 23, 2020, report.
G

The United States and the United Kingdom together accounted for 24% of India’s foreign
tourist arrivals in February 2020. These two countries are among the worst affected by
COVID-19, reporting over 600,000 cases (April 12, 2020), according to data from Johns
Hopkins University.
The Indian tourism and hospitality industry is staring at a potential job loss of around 38
million, which is 70% of the total workforce due to COVID-19. While every industry faces
uncertainty over its future course with the unabated global advance of COVID-19, which has
66 Score Plus CBSE Sample Question Paper with MTPs in English Language & Literature for Class 10 (Term II)
GOYAL A DIGITAL LEARNING APP
BROTHERS
PRAKASHAN
Learn @ ` 1 Per Day

so far caused outbreaks in 185 countries, for this one the fears are more visceral. As those in
the industry point out, information technology, banking/financial services and pharmaceuticals
have no doubt felt the sting of the virus, but they are not as dependent on the physical
mobility of people, and their comfort with the idea of travel, both of which have been
casualties of the current crisis. Will it take nothing short of a vaccine, they wonder, for people
to feel confident enough to undertake non-essential travel again?
Foreign Tourist Arrivals in India
1,200,000

N
1,103,380 1,087,694 1,118,150
1,000,000 1,015,632

HA
800,000

AS
600,000

400,000

AK
200,000

0
January 2019 February 2019 PR
January 2020 February 2020

on the basis of your understanding of the passage, answer ANY FiVE questions from
the six given below. 1*5
RS

(i) Why does the writer say that the Indian travel and tourism industry “plummeted in March
2020”? 1
Ans. The writer says that the Indian travel and tourism industry “plummeted in March 2020
HE

because the foreign tourist arrival in India fell from its peak.
(ii) Comment on the fact that there was a fall in the number of foreign tourist arrivals in
OT

India before the announcement of the lockdown. 1


Ans. This fact indicates that even before the announcement of the lockdown was made,
people’s discomfort in travelling, unless essential, could easily be seen.
BR

(iii) Justify the statement—“As per the KPMG report, 70% of the total workforce of the
Indian travel and tourism industry is likely to lose their jobs”–with reference to the last
paragraph of the report. 1
L

Ans. This statement reveals that around 38 million people, which is 70% of the total
YA

workforce in the Indian tourism and hospitality industry, suffered a potential job loss.
(iv) What was the only exception made by the government with regard to the suspension of
the hospitality industry? 1
O

Ans. The only exception made by the government with regard to the suspension of the
G

hospitality industry was the hotels catering to stranded people and designated as
quarantine facilities.
(v) The number of foreign tourists visiting India in January 2020 increased. How does the
evidence from the graph support this statement? 1
Ans. The graph clearly shows that the number of foreign tourists visiting India in January 2020 was
1,118,150 tourists, and in January 2019, it was around 1,103,380 people which are lesser than
January 2020.
Score Plus CBSE Sample Question Paper with MTPs in English Language & Literature for Class 10 (Term II) 67
GOYAL A DIGITAL LEARNING APP
BROTHERS
PRAKASHAN
Learn @ ` 1 Per Day

(vi) Which month, according to the graph, faced an acute decline in foreign tourists’ arrival in
India? 1
Ans. The month that faced an acute decline in foreign tourists’ arrival in India was February
2020.

sECTioN B - WRiTiNG ANd GRAMMAR (10 marks)


3. Attempt ANY oNE from (i) and (ii).
(i) Read the following excerpt from an article that appeared in an online newsletter:

N
A study published in the journal Current Science was definitive and claimed that odd-

HA
even might have actually been associated with an increase in pollution. The researchers
argue that odd-even did not decrease emissions because more commuters travelled before
odd-even hours kicked in or used other vehicles such as taxis and three-wheelers.

AS
Write a paragraph to analyse this argument. You could think about what alternative
explanations might weaken or strengthen the given conclusion and include rationale/evidence
that might do so. 5

AK
Ans. The odd-even system in Delhi has proven to be largely effective in curbing the problem
of pollution in Delhi. But on analyzing the data and looking at the previous odd-even
experiments, Delhi’s residents should realize that odd-even may provide some relief, but
PR
much more is needed.
The odd-even policy seeks to reduce pollution by halving vehicle usage on Delhi’s roads.
Other cities have often seen a sharp spike in PM2.5 levels following Diwali, though
RS

the spike was not as dramatic as in the case of Delhi. Results from Delhi’s odd-
even experiment could potentially influence similar policies in these cities. Odd-even
HE

also hopes to decrease pollution by clamping down on transport emissions. Cars and
bikes spew out toxic particles which pollute the air, but the extent of this pollution is
widely debated. The Central Pollution Control Board’s study, based on an analysis of
OT

pollution levels before and after odd-even, was ambiguous. It suggested that pollution
might have reduced, but this could not be attributed to a single factor. Out of the two
more optimistic studies, one study analysed air pollution data to conclude that odd-
BR

even decreased PM2.5 levels by 4% to 5% across Delhi. And these were the findings
that Chief Minister Arvind Kejriwal cited in the latest odd-even policy announcement.
Though studies differ in their verdicts of odd-even, they are unanimous in their overall
conclusion: Delhi needs a long-term holistic approach to tackle air pollution.
L

(ii) As the Sports Secretary of your school, ask the manager of BKL Sports Ltd., the best
YA

sports company in your city, to provide the latest equipment needed for a newly built
gym in your school. 5
Ans. St. John’s Public School
O

Brigade Road
G

Bengaluru–560001

June 1, 20XX

The Manager
BKL Sports Ltd.

68 Score Plus CBSE Sample Question Paper with MTPs in English Language & Literature for Class 10 (Term II)
GOYAL A DIGITAL LEARNING APP
BROTHERS
PRAKASHAN
Learn @ ` 1 Per Day

Central Mall
Bengaluru–560011

Dear Sir
Subject: A complete new sports equipment for a gym
As the Sports Secretary of my school, St. John’s Public School, I would like to place
an order for complete gym equipment for our newly built gym. We want the latest and
modern items of the highest standard.

N
Please let us know how soon you can provide the items, a list of prices for each item

HA
and the quality of items provided. You can send some knowledgeable person with a
complete brochure to meet our Principal.
The gym will be inaugurated in July. So, do the needful at the earliest possible.

AS
Yours faithfully
D. Gurunathan

AK
(Sports Secretary)
St. John’s Public School
PR
4. The following paragraph has not been edited. There is one error in each line. Identify the
error and write its correction against the correct blank number. Remember to underline the
correction. The first one has been done for you. 1*3
Error Correction
RS

A good business letter is one that get results. E.g. get gets
The best way to get results is to develop some letter that, (a)
HE

in there appearance, conveys information efficiently. (b)


To perform that function, a business letter should be (c)
OT

concise, clear and courteous.


Ans. Error Correction
(a) some a
BR

(b) there its


(c) that this
L

5. Read the given conversation and complete the passage that follows. 1*2
YA

The king said to the messenger, “Tell the General that I shall send no help. If he can manage
the attack, the glory shall be his own.”
O

The king ordered the messenger (a) _______________ and told that if he could manage the
G

attack, (b) _______________.


Ans. (a) to tell the General that he would send no help
(b) the glory would be his own

Score Plus CBSE Sample Question Paper with MTPs in English Language & Literature for Class 10 (Term II) 69
GOYAL A DIGITAL LEARNING APP
BROTHERS
PRAKASHAN
Learn @ ` 1 Per Day

sECTioN C - LiTERATuRE (20 marks)


6. Answer ANY siX questions in 30-40 words each. 2*6
(i) What is the Chinese legend regarding tea? 2
Ans. A Chinese legend says that one Chinese emperor always boiled water before drinking it.
One day, a few leaves of the twigs burning under the pot fell into the water, giving it a
delicious flavour. It is said they were tea leaves.
(ii) Why is Gautama known as Buddha? 2

N
Ans. ‘Buddha’ means ‘The Awakened’ or ‘The Enlightened’. After seven years of wandering,

HA
Gautama attained wisdom. When he started sharing his new understandings with the
suffering people, he was called ‘Buddha’.
(iii) Do you know the story of Rapunzel? Why does Amanda want to be Rapunzel? 2

AS
Ans. Yes, of course, ‘Rapunzel’ is a beautiful fairy tale of a girl with golden hair. She was
scolded a lot by her annoying mother. To stay away from her anguishes, she lived alone
in a tower. Assuming herself into the same ugly phase, Amanda wants to be aloof and

AK
parted from her mother in a lonely place.
(iv) Why does the poet show a preference for living with the animals? 2
Ans. The poet says that it is better to live with animals as they show a token of goodness.
PR
They are peaceful, self-contained and happy, and show innate goodness, lacking in
human beings. Animals do not grumble about their lives nor cry over their sins.
(v) The dragon, Custard, was showcased as a coward. But the humble dragon took no
RS

time to prove it wrong. Sometimes, situations demand prompt actions like bravery and
courage. Express your views about it. 2
HE

Ans. Bravery and courage are indeed situational and spontaneous. Custard, the dragon, does not
boast of his bravery as other pets of Belinda do. They boasted of their bravery and made
fun of the dragon’s cowardice. But when real danger came, none of them could face the
OT

danger. Only the dragon dared to face the frightening pirate and gobbled him up.
(vi) How did M. Loisel react to his wife’s aspiration for a new dress at the ball? What did
2
BR

he suggest to her?
Ans. When Matilda told her husband that she wanted a new dress to wear, he asked her to
wear whatever she possessed. When she got angry and refused to go to the party at all,
he finally agreed to let her buy a new one.
L

(vii) Who was Lutkins? What was the comical end of the chapter? 2
YA

Ans. Lutkins was a 40-year-old man. He was a cheerful, red-faced person who behaved very
friendly.
O

The comical end of the chapter was when the lawyer was stupefied to know from his
companion that Bill was no other man than Lutkins himself.
G

7. Answer ANY TWo of the following in about 120 words each. 4*2
(i) Pranjol invited Rajvir, and they visited Pranjol’s house in Assam by train.
Imagine yourself as the writer of the story. Based on your understanding of the
personality traits of Pranjol and Rajvir, write a dialogue based on an imaginary event,
showcasing a discussion between them over the beauty of Assam. 4

70 Score Plus CBSE Sample Question Paper with MTPs in English Language & Literature for Class 10 (Term II)
GOYAL A DIGITAL LEARNING APP
BROTHERS
PRAKASHAN
Learn @ ` 1 Per Day

Ans. Rajvir: Pranjol, look outside. It’s very beautiful.


Pranjol: Yes, it is green everywhere, so it looks beautiful.
Rajvir: Pranjol, I haven’t seen such greenery before.
Pranjol: It’s because you live in Delhi, and that is a populated area.
Rajvir: You are right, Pranjol. I am delighted to see these soft green paddy fields. It is a
magnificent view.
Pranjol: You can see a sea of tea bushes stretched as far as the eye can see.

N
Rajvir: Wow! I am enthralled to see such natural beauty of Assam.
Pranjol: Do you know Assam has the largest concentration of plantations in the world?

HA
You will see enough gardens to last you a lifetime!
Rajvir: Pranjol, please let me know more about Assam.
Pranjol: Assam is surrounded by hills, major rivers such as Brahmaputra and Barak and

AS
its tributaries, thick forests, tea gardens which enhances the scenic beauty of Assam.
Rajvir: Now, I want to see all the beautiful places in Assam.

AK
Pranjol: Why not? We have plenty of time as we are going to stay here for one week.
(ii) In such a fast-moving world, when we hear so many incidents happening with people, do
you think Valli did right by not telling her mother about the bus journey?
PR 4
Ans. No, I don’t think that Valli did the right thing. She shouldn’t have travelled alone to the
town without informing her mother. Her innocence might have made it dangerous for
her, as she put herself in a vulnerable position. She could have easily been a victim of
RS

child abuse had she encountered the wrong people on the way. In fact, she was lucky to
return home safely.
HE

Children are easy targets for criminals. So, they should always travel along with their
parents or elders. We hear a lot of instances of crime committed against children these
days, like kidnapping, child abuse, molestation, rape, etc. Valli could have expressed or
OT

at least shared her desire of travelling on the bus with her mother.
(iii) ‘Richard Ebright was a successful scientist who gave a new theory of cells to the
4
BR

scientific world.’ Illuminate with reference to the lesson.


Ans. Richard H. Ebright, at the age of 22, gave a new scientific theory on how cells work.
He published it in a science journal along with his college roommate, James. It was one
of his rarest and finest achievements. He loved butterflies and collected them. By the
L

time he was in grade two, he had collected 25 species of butterflies found around his
YA

hometown. Ebright’s mother helped him like his best friend. She had always encouraged
him for his habits and activities. She took him on trips and bought him telescopes,
microscopes, cameras and other useful instruments. In grade eight, he tried another
O

project to find out the cause of a viral disease that killed nearly all monarch caterpillars
every year. He had also carried successful experiments to prove that hormone produced
G

by a pupa’s gold spots is responsible for the growth of butterfly wings. Hence, we can
say that he was an expert scientist who loved experimenting with new things now and
then.

Score Plus CBSE Sample Question Paper with MTPs in English Language & Literature for Class 10 (Term II) 71
GOYAL A DIGITAL LEARNING APP
BROTHERS
PRAKASHAN
Learn @ ` 1 Per Day

N
HA
AS
AK
PR
RS
HE
OT
BR
L
YA
O
G

27
BROTHERS
PRAKASHAN
GOYAL A DIGITAL
Learn @LEARNING
` 1 Per DayAPP
BROTHERS
PRAKASHAN
Learn @ ` 1 Per Day

N N
HA HA
AS AS
AK AK
PR PR
RS RS
HE HE
OT OT
BR BR
L L
YA YA
O O
G G

To view Sample or Purchase of Books


WWW.goyal-books.com/mtpcx
To view Sample or Purchase of Books
7R YLHZ 6DPSOH RU 3XUFKDVH RI %RRNV 8
www.goyal-books.com/mtpcx
To view Sample or Purchase of Books
:::JR\DOERRNVFRPPWSF[
www.goyal-books.com/mtpcx
28

To view Sample or Purchase of Books
G

WWW.goyal-books.com/mtpcx
To view Sample or Purchase of Books
7R YLHZ 6DPSOH RU 3XUFKDVH RI %RRNV 8
www.goyal-books.com/mtpcx
To view Sample or Purchase of Books
:::JR\DOERRNVFRPPWSF[
www.goyal-books.com/mtpcx

28
GOYAL A DIGITAL LEARNING APP
BROTHERS
PRAKASHAN
Learn @ ` 1 Per Day

MODEL TEST PAPER–7 (Solved)


(Based on the latest CBsE sample Paper)

Time allowed: 2 Hours Maximum Marks: 40

General Instructions:
1. The Question Paper contains THREE sections-READING, WRITING & GRAMMAR and

N
LITERATURE.
2. Attempt questions based on specific instructions for each part.

HA
sECTioN A - REAdiNG (10 marks)

AS
1. Read the passage given below.
Everyone procrastinates. “It’s part of the human condition,” says procrastination researcher

AK
Tim Pychyl, a Psychology professor at Carleton University, Canada. “One of the ways we
cope is avoidance, and that’s what procrastination comes down to: We want to feel good now.
The way to do that is avoid the task.” PR
“One that almost everyone procrastinates on is their wills,” says Steel. “Seventy to eighty
per cent of people, when they pass away, have no will or an out-of-date or incomplete
will.” But death isn’t actually the ultimate avoidance trigger. Research has shown that the
RS

top tasks people delay are cleaning, advancing their careers, taking care of their health
and planning their finances. One big factor for them is fear of failure, of not living up
to expectations. Kelli Saginak, a 57-year-old functional health coach from Wisconsin,
HE

procrastinated about looking for a new job for years. That inability to take action only
confirmed her belief that she would never do any better. “It’s totally fear of judgement,”
OT

says Saginak. “If I don’t take the risk, decide or commit, I don’t have to face the judgment.
Yet it’s simply me judging myself.”
Whatever the motivation (or lack thereof), procrastination is usually not going to be good
BR

for you. “Procrastinators experience higher levels of stress, both from leaving things to the
last minute and from their own negative and self-critical feelings about their procrastination,”
says Fuschia Sirois, a Psychology lecturer at the University of Sheffield, England. “Some
L

of the research has shown that this stress increases their vulnerability for catching colds and
YA

experiencing digestion issues, insomnia, migraines and muscle tension.”


One of the most commonly procrastinated activities: going to bed. But bedtime procrastination
can lead to sleep deprivation, which in turn can bring about obesity, heart disease and many
O

other health issues. There’s one bit of good news: people tend to procrastinate less as they
age. “As you grow older, the illusion of our immortality does get stripped away,” Steel says.
G

“It becomes clearer: How many summers do we have left? 10? 15? They’re really finite. What
are you going to do with each of those summers?”
The best advice might be to think small. “We used to believe behaviours follow attitude, but
if you can prime the pump with a little bit of progress, that motivates you,” Pychyl says. So if
you have been procrastinating on starting an exercise routine, just put on your walking shoes.

72 Score Plus CBSE Sample Question Paper with MTPs in English Language & Literature for Class 10 (Term II)
GOYAL A DIGITAL LEARNING APP
BROTHERS
PRAKASHAN
Learn @ ` 1 Per Day

If you’ve been putting off a work project, re-read your boss’ notes about it. Next, try sending
yourself signals to prompt action.
For instance, you might put your lights on a timed dimmer switch to encourage a consistent
bedtime. Finally, reward yourself for each step you take towards your goal. After you’ve gone
to the gym, take a relaxing bath. After you’ve paid the bills, watch your favourite TV show.
(But don’t try to convince yourself it will work the other way around!)
Based on your understanding of the passage, answer ANY FiVE questions from the six
given below. 1*5

N
(i) What does the writer mean by saying “prime the pump”? 1

HA
Ans. The writer meant encouraging the growth of something that would motivate you towards
success.

AS
(ii) Rewrite the following sentence by replacing the underlined phrase with a word that
means the same from para 2. 1
People often delay or postpone things when they face something they do not want to

AK
do.
Ans. People often procrastinate when they face something they do not want to do.
PR
(iii) State any two steps that would motivate you if you’ve been procrastinating work.
Ans. Two steps that would motivate us if we’d been procrastinating work are, first, we must
1

re-read our boss’ notes about the project, and second, we must try sending signals to
RS

ourselves to prompt action.


(iv) What could be a possible reason for increased vulnerability to diseases like cold and
HE

indigestion? 1
Ans. The possible reason for increased vulnerability to diseases like cold and indigestion could
be stress from procrastination.
OT

(v) List one likely impact of bedtime procrastination. 1


Ans. One likely impact of bedtime procrastination can be sleep deprivation, which can bring
BR

about obesity, heart disease and many other health issues.


(vi) How does the writer justify that procrastination is usually not good for anyone? 1
Ans. The writer justifies it by quoting Fuschia Sirois’ words, “Procrastinators experience
L

higher levels of stress, both from leaving things to the last minute and from their own
YA

negative and self-critical feelings about their procrastination.”


2. Read the passage given below.
O

India Bottled Water Market value US$ XX Mn in 2019 is expected to reach US$ XX Mn by
2026, at a 15% CAGR during a forecast period.
G

The report includes an analysis of the impact of COVID-19 lockdown on the revenue of
market leaders, followers, and disruptors. Since lockdown was implemented differently in
different regions, the impact of the same is also different by regions and segments. The report
has covered the current short term and long term impact on the market, the same will help
decision-makers to prepare the outline for short term and long term strategies for companies
by region.

Score Plus CBSE Sample Question Paper with MTPs in English Language & Literature for Class 10 (Term II) 73
GOYAL A DIGITAL LEARNING APP
BROTHERS
PRAKASHAN
Learn @ ` 1 Per Day

India bottled water market


by Type:, 2019-2026

N
HA
2019 2020 2021 2022 2023 2024 2025 2026
PET Glass

AS
The report contains a detailed list of factors that will drive and restrain the growth of the
India Bottled Water Market. Disposable incomes and awareness about clean drinking water
is rising. The packaged water industry is growing at about 15 % every year. Increase in per

AK
capita spending, rapid urbanization and growing awareness regarding bottled water’s benefits
coupled with increasing preference for an active lifestyle is driving India’s bottled water
market. India is a country with a majority of sunny areas. Bottled water market varies with
PR
season. Installed purifier in homes restrains India’s bottled water market.
People spend longer outdoors and prefer to purchase 20-litre bulk packages of water for home
rather than installing water purifiers. People don’t carry water as it is available everywhere,
RS

even in the smallest shops. There are 5,735 licensed bottlers for packaged drinking water
across India, along with uncountable unbranded ones. The government is unable to provide
HE

clean water, hence leading is taking advantage. Owing to all these factors, bottled water
accounts for about 90 % market share.
PET packaging constitutes 98% of the India bottled water market. PET packaging offers up to
OT

90% weight reduction, compared to glass and therefore, more economical transportation. Better
mechanical qualities, high transparency, pressure resistance, shiny surface, stability, good
barrier properties make PET ideal for water packaging. The reports cover key developments
BR

in the bottled water market as organic and inorganic growth strategies. Various companies
are focusing on organic growth strategies such as product launches, product approvals even
patents and events. The objective of the report is to present a comprehensive analysis of India
L

Bottled Water Market, past and current status of the industry with forecasted market size and
YA

trends and analysis of complicated data in simple language. External as well as internal factors
(like big and small industry players) affecting the business positively or negatively have been
analyzed to give a clear, futuristic view of the industry to the decision-makers.
O

on the basis of your understanding of the passage, answer ANY FiVE questions from
the six given below. 1*5
G

(i) What is the advantage of using bottled water? 1


Ans. The advantage of using bottled water is that it is safe, as it is clean.
(ii) Comment on the impact of COVID-19 lockdown on the revenue of market leaders with
reference to para 2. 1
Ans. The impact of COVID-19 lockdown is different in different regions and segments.

74 Score Plus CBSE Sample Question Paper with MTPs in English Language & Literature for Class 10 (Term II)
GOYAL A DIGITAL LEARNING APP
BROTHERS
PRAKASHAN
Learn @ ` 1 Per Day

(iii) What should not be included as the objective of this report? 1


Ans. Analyzing the quality of bottled water must not be included as the objective of this report.
(iv) For whom will the current short term and long term impacts on the market help? 1
Ans. The current short term and long term impact on the market will help the decision-makers
to prepare the outline for short term and long term strategies for companies by region.
(v) Which year holds an equal percentage of PET packaging and glass packaging? 1
Ans. The year that holds an equal percentage of PET packaging and glass packaging is 2019.

N
(vi) What does the graph show about the PET packaging and glass packaging of the bottled
water market in India after the year 2021? 1

HA
Ans. The graph shows that both the PET packaging and glass packaging of the bottled water
market in India grew, but PET packaging always took the lead.

AS
sECTioN B - WRiTiNG ANd GRAMMAR (10 marks)
3. Attempt ANY oNE from (i) and (ii).

AK
(i) The following data in the form of the histogram shows that death due to violence has
increased considerably during recent years. Write its interpretation in the form of a
paragraph in 100-120 words on how educated youth can play a major role in establishing
peace in society.
PR 5

8
RS
7

6
HE

4
OT

2
BR

0
1990 1995 2000 2005 2010
L

In this,
YA

0-8 indicated– Number of deaths in lakhs


1990-2010 indicates- Years in which death happened
Ans. In society, youth and peace are deeply intertwined. In establishing harmony in society,
O

youth can play an important role. In a study conducted, it was found that since the
G

nineties, the number of deaths due to violence has risen exponentially.


In 1990, the mortality rate in the 1000s was 2.5. Yet it rose to 7.5 in 2010. Indeed, it’s really
sad. It must be inspected. In monitoring this crime index, our youth can play a significant
role. They can lead in many respects to the establishment of stability. They are able to
awaken people to the need for reconciliation. They can help the police identify suspects by
catching them. Through teaching them, they can get offenders into the mainstream where
they can do much more.

Score Plus CBSE Sample Question Paper with MTPs in English Language & Literature for Class 10 (Term II) 75
GOYAL A DIGITAL LEARNING APP
BROTHERS
PRAKASHAN
Learn @ ` 1 Per Day

(ii) Your school, Mahila Vidyalaya, Sector-29, Noida is renovating the school library. Write a
letter to Om Book Shop, Noida, to send you the required books. You are the librarian of
the school. Give details of the books required by your school. 5
Ans. Mahila Vidyalaya
Sector–29
Noida–201301

N
10 June 20XX

HA
Om Book Shop

AS
Great India Place
Sector 18
Noida–201301

AK
Dear Sir
PR
Subject: Order for Books
Mahila Vidyalaya, Sector 29, Noida, is renovating its library and needs a lot of reference
RS

books, dictionaries, and storybooks. Please send by June 20, 20XX, the books listed
below. Payment will be made on receipt of books.
BooKs REQuiREd
HE

1. Collins - English Dictionary


2. Collins Thesaurus
OT

3. Advanced Learner’s Dictionary


4. Essential English Grammar and Composition
BR

5. Senior English Grammar by NK Aggarwala


6. The Canterville Ghost
L

7. The Diary of a Young Girl (Anne Frank)


YA

8. The Story of My Life (Helen Keller)


9. Three Men in a Boat (Jerome K. Jerome)
O

10. Brihat Hindi Kosh, published by Gyan Mandal Ltd., Varanasi


11. Please look into the safe delivery of the books.
G

Yours faithfully
Ms Ragini Devi
(Librarian)

76 Score Plus CBSE Sample Question Paper with MTPs in English Language & Literature for Class 10 (Term II)
GOYAL A DIGITAL LEARNING APP
BROTHERS
PRAKASHAN
Learn @ ` 1 Per Day

4. The following paragraph has not been edited. There is one error in each line. Identify the
error and write its correction against the correct blank number. Remember to underline the
correction. The first one has been done for you. 1*3
Error Correction
At the age of seventeen, Galileo knew that he has E.g. has had
discovered an important law—the law of a pendulum.
He saw that if he can check the time of a swing of (a)
a pendulum by means of his pulse beats, he could (b)

N
just as well check his pulse by mean of a pendulum. (c)

HA
Ans. Error Correction
(a) can could

AS
(b) a the
(c) mean means
5. Read the given conversation between Raj and Sanjay and complete the passage that follows. 1*2

AK
Raj: Our school is celebrating its Annual Day tomorrow.
sanjay: Who is your chief guest? PR
Raj: Our chief guest is the honourable President himself.

Raj told Sanjay that their school was celebrating (a) _________________. Sanjay asked Raj
RS

who their chief guest was. Raj (b) _________________ the honourable President himself.
Ans. (a) its Annual Day the next day
HE

(b) replied that their chief guest was


OT

sECTioN C - LiTERATuRE (20 marks)


6. Answer ANY siX questions in 30-40 words each. 2*6
BR

(i) Even today, any person with a jackfruit-like physical appearance is easily compared to a
baker. Explain. 2
Ans. Earlier, bakers had a jackfruit-like plump physique because baking was a profitable
L

profession. Baker’s family and servants never starved and were prosperous. So even
today, people with such appearance are easily compared to a baker.
YA

(ii) Valli was different from children of her age as she was not interested in playing any
games. Elaborate. 2
O

Ans. Valli was unlike other kids her age because she was not interested in playing elaborate
games. The bus that ran between her village and the neighbouring town was the most
G

exciting view for her. It came and went through her street every hour, once going to
town and once returning.
(iii) What does the girl yearn for? What does this poem tell you about Amanda? 2
Ans. During teenage, usually, children find their parents nagging and irritating them. Amanda
feels the same and yearns for freedom and isolation where she cannot listen to her

Score Plus CBSE Sample Question Paper with MTPs in English Language & Literature for Class 10 (Term II) 77
GOYAL A DIGITAL LEARNING APP
BROTHERS
PRAKASHAN
Learn @ ` 1 Per Day

mother’s scolding. She hates to be unnecessarily poked and instructed by her mother, and
she craves freedom and wants to live life in her own way.
(iv) Trace the contrast between animals and humans according to your perception. 2
Ans. Human beings are rarely found to be satisfied with their situations. On the contrary,
animals are never found worrying themselves about their conditions. Undoubtedly, there
is a vibrant contrast between humans and animals. Men remain offended and unhappy,
but animals don’t. Humans are possessive about their materialistic needs, but animals
aren’t.

N
(v) The use of similes in the poem depicts the characteristics of Custard. Expand with

HA
reference to the poem. 2
Ans. The poet uses similes as poetic devices when he compares the dragon’s mouth with a
fireplace or compares Belinda with a barrel of bears. The other similes used in the poem

AS
are: sharp as mustard, as brave as a tiger in a rage, snorting like an engine, went at the
pirate like a robin at a worm.

AK
(vi) How did the Loisels react when they comprehended that the necklace had been lost?
Why did they feel hopeless? 2
Ans. When the Loisels realised the necklace was missing, they were upset. They were certain
PR
that losing the necklace would make their lives a living nightmare, for M. Loisel was
only a lowly clerk, and replacing the diamond necklace would be extremely difficult for
him.
RS

(vii) Lutkins was really ‘a hard fellow to catch’. How? 2


Ans. Lutkins was a crook. He impressed the narrator with his open, friendly and affectionate
HE

nature. He disguised himself many times. He had a good relationship with all in his
village. So, it was difficult to catch him.
7. Answer ANY TWo of the following in about 120 words each. 4*2
OT

(i) ‘Coorg’ highlights various historical perspectives. What is your opinion on sharing such
historical knowledge essential for us? 4
BR

Ans. The author of the lesson ‘Coorg’ claims that the Coorgi people are descended from either
Greeks or Arabs. They could be of Greek origin, he claims, because there is a belief
that a part of Alexander’s army travelled south and settled here. According to another
L

narrative, they were said to be of Arab ancestry because of their costumes.


YA

General Cariappa, the first General of the Indian Army, was a Coorgi.
We need to understand our country’s deep cultural roots to recognise how our culture has
always embraced and absorbed people from other cultures.
O

This conveys the message of tolerance, which is important in today’s world when wars
rip the world apart over ethnic differences.
G

(ii) Read the line taken from 'The Proposal'—"I'm dying! fetch him!"
Who is dying? Why is the speaker dying? What was to be done to prevent the speaker's
death? 4
Ans. Natalya Stepanovna is stated to be dying.

78 Score Plus CBSE Sample Question Paper with MTPs in English Language & Literature for Class 10 (Term II)
GOYAL A DIGITAL LEARNING APP
BROTHERS
PRAKASHAN
Learn @ ` 1 Per Day

When Natalya hears that Lomov comes to their house with the proposal to marry her
and has driven him out of their house for the quarrels over some petty matters, she feels
very sorry and guilty for this. In utter despair and frustration, Natalya declares that she is
dying and directs Chubukov to fetch Lomov back to their house.
Natalya was desperately looking for a wealthy groom to get herself married, and she
considered Lomov as her life partner. Hearing from Chubukov that Lomov himself
proposed to her, she was ashamed and felt sick. She directed her father to bring Lomov
back to prevent her death.

N
(iii) “Where there is a will, there is a way.” Ebright had a will to become a scientist but did

HA
not have enough resources. How do will and curiosity help us become successful people
in our lives? Elaborate with reference to Ebright. 4
Ans. “Where there is a will, there is a way.” Ebright aspired to be a scientist but lacked the

AS
financial means to do so. His mother catered to his desires and piqued his interest until
he completed schooling. He put in a lot of effort on numerous projects and models and
received a lot of awards. He continued his advanced research on the monarch pupa as a

AK
high school student. His efforts were rewarded, and he received a prize.
During the summer, he got a chance to work in another well-equipped modern army
laboratory. He had another opportunity to work further in the army laboratory during
PR
his senior year. As a result, he was able to carry out numerous experiments. His desire
to work and curiosity enabled him to overcome his lack of resources and become a
successful scientist. Resources are automatically generated when we are willing to do
RS
anything. We receive assistance from all sides.
HE
OT
BR
L
YA
O
G

Score Plus CBSE Sample Question Paper with MTPs in English Language & Literature for Class 10 (Term II) 79
GOYAL A DIGITAL LEARNING APP
BROTHERS
PRAKASHAN
Learn @ ` 1 Per Day

N
HA
AS
AK
PR
RS
HE
OT
BR
L
YA
O
G

27
BROTHERS
PRAKASHAN
GOYAL A DIGITAL
Learn @LEARNING
` 1 Per DayAPP
BROTHERS
PRAKASHAN
Learn @ ` 1 Per Day

N N
HA HA
AS AS
AK AK
PR PR
RS RS
HE HE
OT OT
BR BR
L L
YA YA
O O
G G

To view Sample or Purchase of Books


WWW.goyal-books.com/mtpcx
To view Sample or Purchase of Books
7R YLHZ 6DPSOH RU 3XUFKDVH RI %RRNV 8
www.goyal-books.com/mtpcx
To view Sample or Purchase of Books
:::JR\DOERRNVFRPPWSF[
www.goyal-books.com/mtpcx
28

To view Sample or Purchase of Books
G

WWW.goyal-books.com/mtpcx
To view Sample or Purchase of Books
7R YLHZ 6DPSOH RU 3XUFKDVH RI %RRNV 8
www.goyal-books.com/mtpcx
To view Sample or Purchase of Books
:::JR\DOERRNVFRPPWSF[
www.goyal-books.com/mtpcx

28
GOYAL A DIGITAL LEARNING APP
BROTHERS
PRAKASHAN
Learn @ ` 1 Per Day

MODEL TEST PAPER–8 (Solved)


(Based on the latest CBsE sample Paper)

Time allowed: 2 Hours Maximum Marks: 40

General Instructions:
1. The Question Paper contains THREE sections-READING, WRITING & GRAMMAR and

N
LITERATURE.
2. Attempt questions based on specific instructions for each part.

HA
sECTioN A - REAdiNG (10 marks)

AS
1. Read the passage given below.
1. The other day I received an unusual and very gratifying gift: It was a tree or rather, I
was given half-a-dozen trees which would be planted on my behalf. I had been invited

AK
to give a talk to an organization. After such events, the speaker is usually given a token
gift. Sometimes, the gift is that of a pen, or something useful. Often, the gift is in the
form of a plaque or a similar token. However well-meant, such gifts are destined to
PR
gather dust in forgotten corners. Which is why I was agreeably surprised to be given
a scroll which attested that on a designated plantation established for the purpose,
trees would be added in my name as part of the ‘green’ movement sponsored by the
RS

organization.
2. In an increasingly environment conscious world, the gift of a living tree or a plant makes
HE

for a perfect present. The tradition of giving and receiving gifts has increasingly become
a highly evolved marketing exercise. The giver feels pleasure in giving rather than the
one who gets. Apart from festivals like Diwali, Holi, Christmas, Eid and others, a whole
OT

new calendar of celebration events has been created to promote the giving of gifts:
Mother’s Day, Father’s Day, Teacher’s Day, Valentine’s Day and so on. Sometimes, we
give gifts to friends with whom we hang out.
BR

3. What do you give to people–friends, relatives, spouses, children, parents, employees, and
clients, well wishers who more or less have everything or at least everything that you
could afford to give them as a gift?
L

4. Another box of chocolates..., another bottle of scent or after-shave..., another shirt or


YA

a kurta...? Thinking of unusual and pleasing presents which are also affordable is a
full-time job like wedding planners and planners of theme parties, present planners–
professionals who select and make up gift packages for you to give on occasions like
O

marriages and corporate events—and doing increasingly good business.


G

5. However the best planned gifts of mine go often awry. How often particularly during
the so called ‘festive seasons’, when gift giving and gift receiving reach epidemic
proportions—have you had the embarrassing experience of getting back as a present a
gift you yourself had given to someone who, having no use for it and not realizing that
it was you who had gifted it, to begin with, had unwittingly returned your gift to you?
Like musical chairs, musical gifts only too often go round and round.

80 Score Plus CBSE Sample Question Paper with MTPs in English Language & Literature for Class 10 (Term II)
GOYAL A DIGITAL LEARNING APP
BROTHERS
PRAKASHAN
Learn @ ` 1 Per Day

Based on your understanding of the passage, answer ANY FiVE questions from the six
given below. 1*5
(i) What does the writer mean by the phrase “forgotten corners”? 1
Ans. The phrase, “forgotten corners” refers to those places that are not in regular use.
(ii) Rewrite the following sentence by replacing the underlined phrase with a word that
means the same from para 1. 1
I will not provide funds for the yearly Fashion Gala; it would not be beneficial to me.

N
Ans. I will not sponsor the yearly Fashion Gala; it would not be beneficial to me.

HA
(iii) Write two things or gifts usually given as a token gift to respect the guest of honour of
an event. 1
Ans. Often, the gift is in the form of a plaque or other similar token. Sometimes, the gift is a

AS
pen or something useful.
(iv) Why was the author surprised to be a part of the green movement? 1

AK
Ans. The author was surprised to be a part of the green movement because, on the designated
plantation established for the purpose, trees would be added in her name.
(v) In today’s environment-conscious world, what one thing can make for a perfect present? 1
PR
Ans. In today’s increasingly environment-conscious world, the gift of a living tree or plant
makes for a perfect present.
(vi) How does the author justify the importance of planning the gifts in our lives? 1
RS

Ans. The author explains the importance of planning the gifts in our lives by exemplifying the
sending and receiving of the same gift during the so-called ‘festive seasons’.
HE

2. Read the passage given below.


Of the total Commonwealth Games in Delhi, 2010— related spending on improving Delhi’s
OT

infrastructure, ` 5,700 crores was allocated for flyovers and bridges alone, ` 650 crore for
stadiums, ` 16,887 crores for the Delhi Metro expansion and ` 35,000 crores for new power
plants.
BR

According to records of parliamentary proceedings, when the Games were approved in 2002, the
budget estimate had been ` 617.5 crore. Back then, the Atal Bihari Vajpayee government had
agreed to fund any future shortfalls. By March 2003, the figure had risen to ` 1,895.3 crore.
L

Two years later, the cost had escalated even more and by 2008, an estimated figure of ` 7,000
YA

crore was floating around. A year later, the comptroller and auditor general quoted the figure of
` 13,000 crore— 20 times the original cost estimate. This figure did not include the spending
by various agencies involved in the Games.
O

If you were running a company, such cost overruns would, in most cases, be seen as management
G

failure. But this is only half the story. The figure of ` 66,500 crore was stumbled upon during
a lazy Sunday afternoon lunch with a senior Delhi government official.
Trying to dig deeper into why this figure had remained “invisible”, it can be found out that the
Game’s budgets submitted in the Parliament between 2003 and 2008 had listed a little over `1,300
crore for building infrastructure under the Delhi government’s head. That was the amount the
Delhi government had asked the Planning Commission for. This was the paltry figure reflected

Score Plus CBSE Sample Question Paper with MTPs in English Language & Literature for Class 10 (Term II) 81
GOYAL A DIGITAL LEARNING APP
BROTHERS
PRAKASHAN
Learn @ ` 1 Per Day

in budget estimates presented by sports ministers in the Parliament. The Delhi government and
other agencies were spending far more from their own coffers, but this spending has escaped all
other reporting.

FACTs ANd FiGuREs oF CoMMoNWEALTH GAMEs


delhi Government’s CWG Related infrastructure spends
Amount directly spend on commonwealth ` 670 crore on stadium
Flyovers and Bridges, including Baraphulla Nallah ` 5,700 crore

N
ROB, RUB *at Indira Gandhi International Terminal Network ` 450 crore

HA
BRTS from Ambedkar Nagar to Delhi Gate ` 215 crore
Augmenatation of DTC fleet ` 1800 crore
Construction of Bus depot ` 900 crore

AS
Strengthening and resurfacing of roads ` 650 crore
Street-scaping ` 525 crore

AK
Road signages ` 150 crore
Metro Connectivity PR ` 3000 crore
Total amount spent ` 16,560 crore
‘ROB (road overbridge) ‘RUB (road underbridge)
RS

We all know how the Commonwealth Games in 2010 in New Delhi turned out. With serious
corruption and delays in finishing off construction work in the venues, along with compromising
on the quality of the infrastructure, the mishandling of the CWG led to massive international
HE

media coverage. Economists slammed India for hosting the event, with Miloon Kothari, a leading
expert in socio-economic development, stating that the Games created a negative financial legacy
for the country. “When one in three Indians lives below the poverty line and 40% of the
OT

hungry live in India, when 46% of India’s children and 55% of women are malnourished,
does spending billions of dollars on a 12-day sports event build national pride or is it
a matter of national shame?”
BR

If there is something to feel even worse about, consider this: Melbourne, which hosted the Games
in 2006, only went 0.6 per cent over its estimated spending. Too bad, Delhi didn’t opt to take
a leaf out of the previous host’s book.
L

on the basis of your understanding of the passage, answer ANY FiVE questions from
YA

the six given below. 1*5


(i) What does the writer mean by “management failure”? 1
O

Ans. The writer has exemplified a company with such cost overruns, referring to the
Commonwealth Games, which would be declared in a state of management failure.
G

(ii) Comment on the statement, “The CWG was criticised on almost every front–from
infrastructure to the actual sporting events and was most badly hurt by corruption
charges.” 1
Ans. The whopping escalation of more than twenty times the estimated budget in the
infrastructure expenditure, serious corruption charges and delays in finishing off
construction work in the venues led to CWG’s acute criticism.

82 Score Plus CBSE Sample Question Paper with MTPs in English Language & Literature for Class 10 (Term II)
GOYAL A DIGITAL LEARNING APP
BROTHERS
PRAKASHAN
Learn @ ` 1 Per Day

(iii) Justify “20 times the original cost estimate” in regards to the escalated cost in the budget
estimate of the Games. 1
Ans. When the Games were approved in 2002, the budget estimate had been `617.5 crores.
But in 2009, the comptroller and auditor general quoted a figure of `13,000 crore, which
was actually 20 times the original cost estimate.
(iv) What does Miloon Kothari’s observation about the negative legacy of the games on the
country indicate? 1
Ans. Miloon Kothari’s observation about the negative legacy of the games on the country

N
indicates that there has been an overt expenditure on the Games when it should have

HA
been used for exercising the rights of the poor.
(v) The least infrastructure expenditure was spent on road signage. How does the evidence
from the table support this statement? 1

AS
Ans. The study of the table clearly shows that an amount of `150 crores was spent on road
signages.

AK
(vi) How much amount was spent on metro connectivity as mentioned in the table? 1
Ans. The expenditure on metro connectivity was `3000 crore.

PR
sECTioN B - WRiTiNG ANd GRAMMAR (10 marks)
3. Attempt ANY oNE from (i) and (ii).
(i) Read the following excerpt from an article.
RS

There is a growing lack of sensitivity and respect of our fellow creatures. This attitude is
being drilled into a child by social forces, which can only be countered by environmental
HE

education. Yet, sadly, in most cases, this is not done. What is done is to talk about the
food web and the energy cycles and ecological balance and how the removal of any
elements disrupts the whole system, and how this can affect human beings too.
OT

Write a paragraph to comment upon the given argument.


You could think about what alternative explanations might weaken the given conclusion
BR

and include rationale/evidence that would strengthen/counter the given argument. 5


Ans. From an early age, we begin to tell children how harming the environment will
eventually harm human beings. Are we cultivating the feeling of being the centre of the
L

universe in them? The need of the hour is to sensitize children that all aspects of the
YA

environment and all forms of life are as important as human life. We must not harm any
form of life because that might lead to the destruction of the balance that exists in nature.
We have to encourage children to establish a connection with nature and preserve it.
O

(ii) As the President of the Resident Welfare Association, Maharani Nagar, Jhansi, order a
G

complete set of furniture for the newly built Community Hall in the colony. You are
A. Chaturvedi. 5
Ans. Resident Welfare Association
Maharani Nagar
Jhansi–284128

Score Plus CBSE Sample Question Paper with MTPs in English Language & Literature for Class 10 (Term II) 83
GOYAL A DIGITAL LEARNING APP
BROTHERS
PRAKASHAN
Learn @ ` 1 Per Day

10 October 20XX

D.D. Furniture
Civil Lines
Jhansi–284001

Dear Sir

N
Subject: New furniture required for Community Hall
The Community Hall of our colony requires new furniture. We want to place an order, as

HA
you are one of the best furniture shops in our city. We need the following sets:
1. 3 piece sofa sets in brown and cream colours

AS
2. 24 garden chairs which can be folded and kept
3. one Table Tennis Table

AK
4. 2 dining tables
5. 3 garden umbrellas
6. 6 armchairs for the elderly
PR
7. 2 shelves for keeping books
Please send someone with a brochure for us to select and a list of the cost of each item
as soon as possible. Do let us know the date by which you can supply the items.
RS

Yours faithfully
HE

A. Chaturvedi
(President)
OT

4. The following paragraph has not been edited. There is one error in each line. Identify the
error and write its correction against the correct blank number. Remember to underline the
correction. The first one has been done for you. 1*3
BR

Error Correction
The camel was a domestic animal. It is being used for E.g. was is
thousands of years by their desert people of Africa and Asia (a)
L

to meet their needs. In fertile areas of the desert, camels pulls (b)
YA

a plough and were used for other similar work. (c)


Ans. Error Correction
O

(a) their the


(b) pulls pull
G

(c) were are


5. Read the given conversation between Hobbes and Calvin and complete the passage that
follows. 1*2
Hobbes: Why are you digging a hole?
Calvin: I’m looking for buried treasure!

84 Score Plus CBSE Sample Question Paper with MTPs in English Language & Literature for Class 10 (Term II)
GOYAL A DIGITAL LEARNING APP
BROTHERS
PRAKASHAN
Learn @ ` 1 Per Day

Hobbes: What have you found?


Calvin: A few dirty rocks, a weird root, and some disgusting grubs.

Hobbes asked Calvin (a) ________ a hole, to which Calvin replied that he was looking for
buried treasure. Hobbes again asked (b) ________. Calvin replied saying a few dirty rocks, a
weird root and some disgusting grubs.
Ans. (a) why he was digging

N
(b) what he had found

HA
sECTioN C - LiTERATuRE (20 marks)
6. Answer ANY siX questions in 30-40 words each. 2*6
(i) Comment on the significance of a baker in a traditional Goan village. 2

AS
Ans. A baker is an important member of the traditional Goan village because bread is essential for
a Goan meal. Besides, sweet bread ‘bol’ is a special delicacy, most important in marriages

AK
and served at festivals. Also, cakes and bolinhas are special and a must for Christmas.
(ii) How can you say that the conductor was a jolly fellow? Support your answer with
examples. PR 2
Ans. The conductor was a fun-loving, jolly fellow. For instance, when Valli refused his help to
board the bus, she said jokingly, “don’t be angry with me, my fine madam. Everybody
move aside, please—make way for madam.”
RS

(iii) “Children are guided by their elders at all times.” Do you agree? Put light in the poem’s
(Amanda!) context. 2
HE

Ans. Undoubtedly, the elders advise and guide children for their welfare and well being.
Indeed, children like Amanda find them cranky and nosy in their life that is not right and
acceptable. Elders share their experiences and are grounded with perfect knowledge.
OT

(iv) What makes the poet sick? Explain with reference to the poem ‘Animals’. 2
Ans. Humans tend to commit all kinds of sins intently in this idealistic world. They still talk
BR

over their duties to God; this thought makes the poet sick and curious.
(v) What did the pirate do when he gaped at Belinda’s dragon? 2
Ans. The pirate gaped at Belinda’s dragon because he thought Custard was a coward. The pirate
L

gulped some wine from the wine bottle in his pocket to prepare himself for the fight.
YA

(vi) Hobbies play an important role in one’s life. Elaborate this with reference to the lesson
‘The Making of a Scientist’. 2
Ans. Hobbies play an important role in one’s life. Ebright started collecting butterflies as a
O

hobby, which led him to research and discover many things about butterflies.
G

(vii) Do you think M. Loisel had a delightful evening at the ball? Cite examples from the
text. 2
Ans. M. Loisel was intoxicated with immense pleasure at the party. She danced with
enthusiasm and received the admiration of all the guests. Everyone praised her grace and
charm, and she was living her long-cherished dream.

Score Plus CBSE Sample Question Paper with MTPs in English Language & Literature for Class 10 (Term II) 85
GOYAL A DIGITAL LEARNING APP
BROTHERS
PRAKASHAN
Learn @ ` 1 Per Day

7. Answer ANY TWo of the following in about 120 words each. 4*2
(i) ‘The author shared a lot of information about the bakers in his hometown when he
was young. He appears to be a very observant child. Should children have such keen
observation powers? 4
Ans. Yes, children must be observant because being observant will help them be aware of
society. Therefore, such strong observation skills would be beneficial for children.
The author gives a comprehensive description of his childhood days in Goa in the story.

N
He tells us about the bakers that they held a significant place in the Goan society since
the time of the Portuguese. The bakers were friends, companions, and guides to the

HA
children and were known as ‘paders’. He recalled a baker who would visit the village
twice a day, and he would carry a bamboo stick which the baker used to hit the ground.
Being a keen observer, the author in his childhood knew even about the profits gained

AS
by the bakers. He even knew that the bakers maintained their monthly bills on the walls.
(ii) Justify the title 'The Proposal'. 4

AK
Ans. The play's title, 'The Proposal', is quite appropriate. Lomov, a young man, wants to
propose marriage to Natalya. For the same reason, he comes to her house. He doesn't
propose right away since he's perplexed. He rambles on and on. He says things that
PR
aren't relevant. Natalya begins quarrelling because she does not realise Lomov has come
to propose to her and believes he has come to claim Oxen Meadows as his own. As a
result, quarrels follow one another. Natalya learns from her father, Chubukov, that Lomov
RS

has come to propose to her. She completely forgets about the fight. The proposal is then
presented and accepted. They get engaged in the end.
(iii) Read the following quote by an American Comedian, John Belushi.
HE

“The trick is knowing what you want to do and then resolving to do everything you
have to do to get there.”
OT

How does it align with the storyline of ‘‘The Hack Driver”? Justify. 4
Ans. The lawyer was sent to New Mullion to serve the summons on Oliver Lutkins, who was
needed as a witness in a law case. A hack driver at the station, who called himself Bill
BR

Magnuson, befriended him. He told the lawyer that he knew Lutkins and would help
find him. Bill took him to all the places where Lutkins was known to frequent. However,
Oliver Lutkins was not found. A poor hack driver successfully tricks a qualified lawyer.
L

This was because the lawyer had not equipped himself adequately in his resolve to
complete his job effectively. He followed instructions blindly and left in search of
YA

someone of whom he did not have a clue. His disgruntlement and disinterest pushed him
backwards in his journey to succeed as a good lawyer.
O
G

86 Score Plus CBSE Sample Question Paper with MTPs in English Language & Literature for Class 10 (Term II)
GOYAL A DIGITAL LEARNING APP
BROTHERS
PRAKASHAN
Learn @ ` 1 Per Day

N
HA
AS
AK
PR
RS
HE
OT
BR
L
YA
O
G

27
BROTHERS
PRAKASHAN
GOYAL A DIGITAL
Learn @LEARNING
` 1 Per DayAPP
BROTHERS
PRAKASHAN
Learn @ ` 1 Per Day

N N
HA HA
AS AS
AK AK
PR PR
RS RS
HE HE
OT OT
BR BR
L L
YA YA
O O
G G

To view Sample or Purchase of Books


WWW.goyal-books.com/mtpcx
To view Sample or Purchase of Books
7R YLHZ 6DPSOH RU 3XUFKDVH RI %RRNV 8
www.goyal-books.com/mtpcx
To view Sample or Purchase of Books
:::JR\DOERRNVFRPPWSF[
www.goyal-books.com/mtpcx
28

To view Sample or Purchase of Books
G

WWW.goyal-books.com/mtpcx
To view Sample or Purchase of Books
7R YLHZ 6DPSOH RU 3XUFKDVH RI %RRNV 8
www.goyal-books.com/mtpcx
To view Sample or Purchase of Books
:::JR\DOERRNVFRPPWSF[
www.goyal-books.com/mtpcx

28
GOYAL A DIGITAL LEARNING APP
BROTHERS
PRAKASHAN
Learn @ ` 1 Per Day

MODEL TEST PAPER–9 (Solved)


(Based on the latest CBsE sample Paper)

Time allowed: 2 Hours Maximum Marks: 40

General Instructions:
1. The Question Paper contains THREE sections-READING, WRITING & GRAMMAR and

N
LITERATURE.
2. Attempt questions based on specific instructions for each part.

HA
sECTioN A - REAdiNG (10 marks)

AS
1. Read the passage given below.
Some people get emotional satisfaction from talking about how awful other people are. But

AK
research has found that when you talk negatively to a friend about how awful a colleague
is, the listening friend is more likely to associate the negativity to you rather than the person
you’re describing. Researchers call this ‘spontaneous trait transference’. So, it’s best to go
PR
easy on bad-mouthing others because it may backfire anyway.
It takes a big person to accept outwardly and inwardly that they screwed up. I’m not saying
we should never blame other people. Sometimes others are at fault and they need to know
RS
it and stand up. But being able to accept responsibility when that’s right means we actually
become less helpless and passive. If everything is someone else’s fault, then what part do I
play in my own life? Are my actions entirely without consequence? Am I that powerless? Or
HE

do all my actions only lead to good outcomes?


Knowing we can accept responsibility when things go wrong means we can also accept
OT

credit when things go well. We do, as individuals, have an effect on life; and that’s a
good thing. But we need to develop the capacity to be objective enough about ourselves to
avoid assuming we could never possibly have created problems ourselves. We also need to
BR

distinguish between accepting responsibility and punishing ourselves unduly.


If you are just used to dishing out the blame and not accepting your art, remember the
research that chose being able to apologize in relationships makes them much more likely to
L

last and thrive. Don’t always be too quick to blame yourself, but, just now and then, admit to
co-workers or partners or friend that yes, you too are human and you made a mistake, people
YA

will respect you for it. Ever noticed some people get more hung up on assigning blame than
actually fixing a problem? If people feel you blame them unfairly, they will resent you. People
instinctively hate injustice people can be shouted at, cursed at, and blamed.
O

Calling someone idiot or telling them they “always do everything wrong!” is not feedback;
G

it’s just abuse, no matter why you think you’re doing it. This kind of emotional incontinence
may make people anxious, but they’ll never respect you because it displays your weakness
so clearly. As the wise Roman empire, Marcus Aurelius said, “How much more grievous are
the consequences of anger than the causes of it.” Life is full of people who take emotional
shortcuts and blame other people unfairly or aggressively; for the sake of the human race,
don’t be one of them. But I wouldn’t blame you if you were.

Score Plus CBSE Sample Question Paper with MTPs in English Language & Literature for Class 10 (Term II) 87
GOYAL A DIGITAL LEARNING APP
BROTHERS
PRAKASHAN
Learn @ ` 1 Per Day

Based on your understanding of the passage, answer ANY FiVE questions from the six
given below. 1*5
(i) What does the writer mean by using the word 'bad-mouthing' in a conversation? 1
Ans. By using 'bad-mouthing' in a conversation, the writer meant to criticize in a very
unpleasant manner.
(ii) Rewrite the following sentence by replacing the underlined phrase with a word that
means the same from para 2. 1

N
Be alert. Your near and dear ones can also ruin you through stupidity.
Ans. Be alert. Your near and dear ones can also screw you up.

HA
(iii) Describe two ways in which we can develop the ability to be objective about ourselves. 1
Ans. The writer tells us to be objective by saying, "Knowing we can accept responsibility

AS
when things go wrong means we can also accept credit when things go well. We also
need to distinguish between accepting responsibility and punishing ourselves unduly."
(iv) What does the writer mean when he says, “instinctively hate injustice”? 1

AK
Ans. By saying “instinctively hate injustice”, the writer means the natural action of a person
not to adopt what is unfair. PR
(v) How does the writer explain the difference between feedback and abuse? 1
Ans. The writer explains the difference by saying that calling someone an idiot or telling them
they “always do everything wrong!” is not feedback; it’s just abuse, no matter why you
RS

think you’re doing it. This kind of emotional incontinence may make people anxious, but
they’ll never respect you because it displays your weakness so clearly.
HE

(vi) How does the writer explain the importance of self-examination in our lives? 1
Ans. The writer explains the importance of self-examination in our lives by giving the
example of the negativity of a person’s thoughts.
OT

2. Read the passage given below.


HIV infection in India was first detected in 1986 among female sex workers in Chennai.
BR

At the beginning of the epidemic, four southern states of India, Andhra Pradesh,
Karnataka, Maharashtra, Tamil Nadu, and the north-eastern states of Manipur and
Nagaland were classified as high-prevalence states. Infections in the early days were
L

mostly found in female sex workers and truck drivers, but soon spread to housewives and
the general population.
YA

Over 90% of those infected with the virus were not aware of their status. Most diagnoses
occurred at the late stages of the disease. While the number of voluntary counselling and
O

testing centres on the rise in both government and non-governmental settings in India,
facilities had been underused because of (a) inaccurate perceptions of personal risk (b) a
G

widespread belief that HIV is restricted to high-risk populations such as sex workers, drug
users, and truck drivers and (c) the persistence of stigma surrounding HIV. Some available
estimates of incidence have been carried out among the sex workers in Maharashtra (22.1 per
100 person-years) and drug users in Chennai (4.53%). But there had been a steady decline in
the number of PLHIV (People Living with HIV/AIDS) in India since the 2.23 million figure
reported in 2007.

88 Score Plus CBSE Sample Question Paper with MTPs in English Language & Literature for Class 10 (Term II)
GOYAL A DIGITAL LEARNING APP
BROTHERS
PRAKASHAN
Learn @ ` 1 Per Day

As per the estimates of the National Aids Control Organisation (NACO), there are 2.12
million persons infected with HIV in India (NACO 2012). The adult HIV prevalence at the
national level has continued its steady decline from an estimated peak of 0.38% in 2001–
2003 through 0.34% in 2007 and 0.28% in 2012 to 0.26% in 2015. Also, there has been an
encouraging decrease in the number of new infections since the peak of 3,00,000 per year in
1998 to less than 1,20,000 per year in 2012, a reduction of about 60%. Therefore, the HIV
epidemic in India clearly shows signs of decline.
3.00
0.21

0.23

N
0.24

2.50
New HIV Infections in Lakhs

0.25

HA
0.26
2.00
0.25

0.25

0.24

0.23
1.50

0.21
2.64

0.20
2.51

0.18

0.17
2.27

0.17

0.16

0.15
AS 0.12

0.10
2.00

1.00
1.78

1.58

1.41

1.29

1.17

1.06

0.96
0.50

0.88

0.85

0.82

0.80

0.79

0.77

0.76
AK
0.00
1998

1999

2000

2001

2002

2003

2004

2005

2006

2007

2008

2009

2010

2011

2012

2013

2014

2015
Adults (15 + yrs) PR
Children (0-14 yrs)

on the basis of your understanding of the passage, answer ANY FiVE questions from
the six given below. 1*5
RS

(i) Why does the writer use the term "high-prevalence states"? 1
Ans. The writer has used the term "high-prevalence states" to indicate the states where HIV
HE

infection was first detected among female sex workers and truck drivers.
(ii) Write what do some available estimates of incidence reveal with reference to para 2. 1
Ans. Some available estimates of incidence revealed that there had been a steady decline in
OT

the number of PLHIV in India since the 2.23 million figure reported in 2007.
(iii) According to the survey, what are the reasons behind the counselling centres being
BR

underused? 1
Ans. The counselling centres were underused because of inaccurate perceptions of personal
risk, the persistence of stigma surrounding HIV, and the belief that HIV is restricted to
high-risk populations only.
L

(iv) What do studies show about 90% of the infected people with the virus? 1
YA

Ans. Studies show that these people were unaware of their status.
(v) "The HIV epidemic in India clearly shows signs of decline." How does the evidence
O

from the graph support this statement? 1


Ans. The adult HIV prevalence at the national level has continued its steady decline from an
G

estimated peak of 0.38% in 2001–2003 through 0.34% in 2007 and 0.28% in 2012 to 0.26%
in 2015.
(vi) According to the survey, which year did not see a decline in the number of children affected
by HIV from the previous year from the years 1998-2002? 1
Ans. The year that did not see a decline in the number of children affected by HIV from the
previous year was 2001.
Score Plus CBSE Sample Question Paper with MTPs in English Language & Literature for Class 10 (Term II) 89
GOYAL A DIGITAL LEARNING APP
BROTHERS
PRAKASHAN
Learn @ ` 1 Per Day

sECTioN B - WRiTiNG ANd GRAMMAR (10 marks)


3. Attempt ANY oNE from (i) and (ii).
(i) The chart shows the division of household tasks done by male and female in Great
Britain. Write a paragraph analyzing the data and chart given. 5

Division of household tasks by Gender in Great Britain


80

N
70

HA
60
Minutes per person per day

5o
40

AS
30
20

AK
10
0
Cleaning Clothes,
Cooking Care of own
house Gardening,
PR Maintenance washing,
baking children
tidying pet care odd jobs ironing,
washing up and play
sewing

30 13 49.5 20 26 2
RS

males
females 74 58 21 45 9.5 25

Ans. The chart shows the number of minutes per day spent by British men and women
HE

in doing household tasks. On an average, the women spend about four hours doing
household tasks whereas the men spend less than two and half hours.
OT

The tasks on which women spend more time than men include cooking, cleaning the
house, taking care of the children and laundry. In doing all these tasks, women spend
approximately 200 minutes which is almost three times the time spent by men in these
BR

tasks. On the other hand, men spend twice the time than women in doing tasks like
gardening and maintenance of odd jobs in the house. Men spend the highest time, almost
50 minutes, in gardening and pet care. While the women spend only 30 minutes for
these tasks, the time spent by men in washing, ironing and sewing clothes is as low as 2
L

minutes per day as opposed to 25 minutes spent by women.


YA

In short, women spend far more time doing household chores than men whilst men
prefer to do gardening, pet care and maintaining odd jobs.
(ii) You are Vibha Gupta, hostel warden of Bloom Public School, Ooty. Write a letter to the
O

Manager of Ghansayam Electronics and Domestic Appliances, Delhi, placing an order for
G

a few geysers, ceiling fans, etc. that you wish to purchase for the hostel. Also, ask for
the discount permissible on the purchase. 5
Ans. Bloom Public School
Ooty, Tamil Nadu

November 15, 20XX

90 Score Plus CBSE Sample Question Paper with MTPs in English Language & Literature for Class 10 (Term II)
GOYAL A DIGITAL LEARNING APP
BROTHERS
PRAKASHAN
Learn @ ` 1 Per Day

The Sales Manager


Ghansayam Electronics and Domestic Appliances
Delhi

Sir
Subject: Bulk Purchase of Electronic Appliances

N
We want to place a bulk order of geysers, ceiling fans and microwave ovens for our
hostel at Ooty. It would be very kind of you to send your terms and conditions to

HA
supply the articles mentioned above. The particulars of the items and their numbers to be
supplied are given below:
1. Ceiling Fans — 40

AS
2. Geysers — 30
3. Microwave Ovens — 15

AK
Kindly let us know the discount you offer on bulk purchases. We would also like to
know the mode of payment.
We would like to hear from you at the earliest.

Yours faithfully
PR
Vibha Gupta
RS

Hostel Warden
4. The following paragraph has not been edited. There is one error in each line. Identify the
HE

error and write its correction against the correct blank number. Remember to underline the
correction. The first one has been done for you. 1*3
Error Correction
OT

At one time, consumers saw telemarketing as the necessary E.g. the a


evil and want a “No call registry” to prevent such calls to those (a)
BR

who did not wish to receive them, but today, they want
a blanket ban on telemarketing. And that call for a prohibition (b)
and not a restriction on telemarketing was slowly becoming louder. (c)
Ans. Error Correction
L
YA

(a) want wanted


(b) that this
(c) was is
O

5. Read the given conversation between Shilpa and Neetu and complete the passage that follows. 1*2
shilpa: Can I borrow your English notebook to complete my pending work?
G

Neetu: Sure, you can. But when will you return as I also need it to do my work?
shilpa: I will return it to you after two days from today.

Shilpa asked Neetu (a) ________ her English notebook to complete her pending work. Neetu
replied that she could and (b) ________ return it as she also needed it to do her work. Shilpa
said she would return it to her after two days from that day.

Score Plus CBSE Sample Question Paper with MTPs in English Language & Literature for Class 10 (Term II) 91
GOYAL A DIGITAL LEARNING APP
BROTHERS
PRAKASHAN
Learn @ ` 1 Per Day

Ans. (a) if she could borrow


(b) asked when she would

sECTioN C - LiTERATuRE (20 marks)


6. Answer ANY siX questions in 30-40 words each. 2*6
(i) As stated in the lesson, ‘Coorg’, what do you know about ‘Bylakuppe’? 2
Ans. Bylakuppe in Coorg is India’s largest settlement of Buddhist monks. These Buddhist

N
monks can be seen here dressed in red, ochre and yellow robes. They are amongst the
many surprises that wait to be discovered by visitors searching for the heart and soul of

HA
India.
(ii) Standing on the bus seat is dangerous for Valli. What explanation does the conductor
2

AS
give in support of this statement?
Ans. Valli’s view was obstructed because of a canvas blind on her window, so she stood up
on her seat to get a better view of the scenery outside. The conductor said that if she

AK
stood on the seat, she might fall and hurt herself when the bus made a sharp turn or hit
a bump.
(iii) ‘I’ll certainly never let down my bright hair!’ Explain this with reference to the poem
PR
'Amanda!'. 2
Ans. Amanda wants to be Rapunzel because she imagines life in the tower as serene and
unusual. She believes that she will be free in the tower and live in a serene environment.
RS

She also states that she will never let her hair down in front of anyone, preventing
anyone from approaching her in the tower.
HE

(iv) What is the relevance of ‘tokens’ in the poem ‘Animals’? Who brings them to the poet? 2
Ans. Animals show tokens of goodness. They are self-contained, peaceful, and happy
creatures. Animals are not unhappy and, indeed, bring out tokens of man’s good nature,
OT

lost long ago. Animals do not grumble about their lives.


(v) What values should Belinda have possessed so that Custard, too, could have been among
BR

her favourites? 2
Ans. Belinda should have treated him with respect. Instead of laughing and making fun of
him with the other pets, she should have made him feel at ease and safe in the house.
L

Belinda should have guarded him.


YA

(vi) What was the common belief about the twelve tiny gold spots on a monarch pupa? What
is the actual purpose of these tiny gold spots? 2
Ans. These twelve tiny gold spots were believed to be ornamental only. The actual purpose
O

of these tiny gold spots is to produce a hormone necessary for the butterfly’s full
development.
G

(vii) Why could Mme Forestier not recognise her friend Matilda at the end of the story? 2
Ans. Mme Frostier met her friend Matilda after ten long years. During these years, Matilda
had gone through all the sufferings and hardships to repay the debts of the lost necklace.
She lost her youth, softness, beauty and comforts due to false riches. This is the reason
Mme Frostier could not recognise her friend Matilda.

92 Score Plus CBSE Sample Question Paper with MTPs in English Language & Literature for Class 10 (Term II)
GOYAL A DIGITAL LEARNING APP
BROTHERS
PRAKASHAN
Learn @ ` 1 Per Day

7. Answer ANY TWo of the following in about 120 words each. 4*2
(i) What did Kisa Gotami do after the Buddha had asked her for a handful of mustard
seeds? How did the Buddha teach Kisa Gotami the truth of life? What does the Buddha
say about the world? 4
Ans. Kisa Gotami went from house to house to get a handful of mustard seeds, and people
gave her the mustard seed. When she asked if anyone had died in their family, they
regretfully told her that the livings were few, but the deads were many. Kisa Gotami
didn’t find any house where someone had not died.

N
The Buddha wanted Kisa Gotami to understand that man cannot get peace of mind by
grieving. On the contrary, if he grieves, his pain will be greater, and his body will suffer.

HA
He will make himself sick and pale. A man who does not lament gets a piece of mind. A
man who has overcome all sorrow will be free from grief and will be blessed.
The Buddha told Kisa Gotami that the life of mortals in this world was troubled and

AS
brief, and it was combined with pain. Those who were born could not avoid dying, and
such was the nature of living beings. All earthen pots made by the potter end in being
broken similarly, all were subject to death.

AK
(ii) Rajvir seemed to have a lot of information about tea. What all does he tell? Which
character trait about him is revealed? Is it essential for children? 4
PR
Ans. In this lesson, the writer describes how popular tea has become as a beverage globally.
Over 80 crores of cups of tea are drunk every day throughout the world. It displays
the growing recognition of tea in the world. Rajvir seemed to enjoy the delicacy of
RS
the beauty of tea plantations. Rajvir looked out of the train and experienced beautiful
scenery. Soon the soft green paddy fields were left behind, and tea bushes were
everywhere. He was fascinated by the splendid view of tea gardens. It was all green
HE

ubiquitously. Rajvir is ecstatic to hold countless rows of tiny tea bushes to the extent
that his eye could see. Doll-like figures were stirring firmly in those tea bushes. Some
women were plucking tea leaves from the bushes. It was the next sprouting period,
OT

and both the sides of the gravel road were acres and acres of neatly pruned tea bushes.
Women were wearing plastic aprons, plucking new tea leaves, and putting them in
the bamboo baskets on their backs. This information gives us an insight into our rich
BR

heritage in the world. It gives us in-depth knowledge of the subject – tea plantation.
(iii) ‘Matilda’s craving for a bubble reputation was the cause of her ruin.’ Conclude how she
could have avoided the misfortune that dawned upon her. 4
L

Ans. When her husband shows her the formal party invitation, Matilda becomes agitated and
YA

angry. Her thoughts are consumed by how she can display herself and be the prettiest
and most attractive person at the party. She is blinded by her vanity and false pride when
she decides to spend her husband’s money on a new dress for the party. She then decides
to borrow a necklace to complete her look. She has no idea that any goodwill she may
O

acquire due to these actions will be fleeting.


G

Matilda should not have succumbed to her desires. She should have assessed the benefits
and drawbacks of borrowing a friend’s jewel. The jewel may or may not be a diamond,
but the responsibility of keeping it safe and returning it in excellent condition is one that
a person with Matilda’s mindset could hardly undertake. She was dissatisfied with her
lifestyle, and even a single night of opulence would be welcome. Upon losing the jewel,
the misfortune that befell her and her husband could have been prevented if she had
been pleased with her life and lived within her means.

Score Plus CBSE Sample Question Paper with MTPs in English Language & Literature for Class 10 (Term II) 93
GOYAL A DIGITAL LEARNING APP
BROTHERS
PRAKASHAN
Learn @ ` 1 Per Day

N
HA
AS
AK
PR
RS
HE
OT
BR
L
YA
O
G

27
BROTHERS
PRAKASHAN
GOYAL A DIGITAL
Learn @LEARNING
` 1 Per DayAPP
BROTHERS
PRAKASHAN
Learn @ ` 1 Per Day

N N
HA HA
AS AS
AK AK
PR PR
RS RS
HE HE
OT OT
BR BR
L L
YA YA
O O
G G

To view Sample or Purchase of Books


WWW.goyal-books.com/mtpcx
To view Sample or Purchase of Books
7R YLHZ 6DPSOH RU 3XUFKDVH RI %RRNV 8
www.goyal-books.com/mtpcx
To view Sample or Purchase of Books
:::JR\DOERRNVFRPPWSF[
www.goyal-books.com/mtpcx
28

To view Sample or Purchase of Books
G

WWW.goyal-books.com/mtpcx
To view Sample or Purchase of Books
7R YLHZ 6DPSOH RU 3XUFKDVH RI %RRNV 8
www.goyal-books.com/mtpcx
To view Sample or Purchase of Books
:::JR\DOERRNVFRPPWSF[
www.goyal-books.com/mtpcx

28
GOYAL A DIGITAL LEARNING APP
BROTHERS
PRAKASHAN
Learn @ ` 1 Per Day

MODEL TEST PAPER–10 (Solved)


(Based on the latest CBsE sample Paper)

Time allowed: 2 Hours Maximum Marks: 40

General Instructions:
1. The Question Paper contains THREE sections-READING, WRITING & GRAMMAR and

N
LITERATURE.
2. Attempt questions based on specific instructions for each part.

HA
sECTioN A - REAdiNG (10 marks)

AS
1. Read the passage given below.
As you approach your career goals, you feel the need of having skill in making presentations.

AK
These presentations open the door to a brighter, bigger and more prestigious career. This
works in two ways: they open closed avenues and also bigger ones for those who are already
in the queue. These will lead them up the pole of success, promotion and prosperity. These
attempts should not be taken carelessly or casually because they put you on the path of
PR
realization of your ambition. As in many other spheres, marketing yourself for a job in a
career stream too needs your meticulous attention and preparation. This also shows your inner
urges; how you look at your job, your long term career perspectives and the ultimate result
RS

from it. Failing to prepare is preparing to fail! This should be your guideline. There is no
reason to fear making a presentation if one is aptly prepared for it. Plodding is not enough to
create an impression and get a concept approved. You have to be able to inform and convince
HE

others to get their approval. This is where your presentation skills come into play. Absence of
these skills mars your chances.
OT

Genuine preparation is the first step. This begins with mental readiness and storing the mind
with sufficient relevant matter. An empty mind cannot deliver the goods. Fill the mind with
ideas, concepts, facts and figures. You will have a lot of material for presentation. Add to
BR

your information base. When your research base is extensive, you can select the best for
the presentation. Conduct research through books, magazines, newspaper and publications,
interviews, informal chats and letters exchanged with experts in the field. The presentation
will be most appreciated when you are able to give the audience something new, more than
L

what it already knows. Add some new slants. Find a presentation with an action point to make
YA

it more persuasive. If you know who is going to form the majority of your audience, you can
tailor to make your presentation to suit it. Make every effort to hit the bull’s eye.
O

Begin by stating the purpose and providing a preview of what will be created. The preview
should be a quick summary of the outline of what is going to follow. Arouse curiosity and
G

anticipation. Go through the outline point by point. Be sure to cover every topic promised
in the preview. In the end, sum up your points briefly, but effectively. Think of some punch
lines. Arrange your material in a logical sequence. Put down one idea or one piece of
information and carefully link it to the next. One technique you can employ is the use of
semantic bridges. For example, “Amazingly, Interestingly” can be put before the sentence runs
its course. The duration is dependent on the nature of the topic, its importance, relevance to a
particular situation and audience. The main aim is to express as well as to impress.

94 Score Plus CBSE Sample Question Paper with MTPs in English Language & Literature for Class 10 (Term II)
GOYAL A DIGITAL LEARNING APP
BROTHERS
PRAKASHAN
Learn @ ` 1 Per Day

Based on your understanding of the passage, answer ANY FiVE questions from the six
given below. 1*5
(i) What does the writer mean by saying, “mars your chances”? 1
Ans. By saying, “mars your chances”, the writer meant that the absence of the required skills
for making a presentation might spoil the chances of one's promotion.
(ii) Rewrite the following sentence by replacing the underlined phrase with a word that
means the same from para 2. 1

N
The book he recently got published had a sociological point of view.
Ans. The book he recently got published had sociological slants.

HA
(iii) State two elements essential for an effective presentation. 1
Ans. Two elements essential for an effective presentation are that it should have a clear

AS
objective, and it must be well-rehearsed before presenting.
(iv) “An empty mind cannot deliver the goods.” How does this statement prove relevant to
1

AK
making presentations?
Ans. This statement proves relevant to making presentations because one can’t do anything
efficiently without research and preparation. An empty mind is an open mind that cannot
deliver goods with perfection. PR
(v) Describe one likely step in preparing an impressive presentation. 1
Ans. Genuine preparation is the first step. This begins with mental readiness and storing the
RS

mind with sufficient relevant matter.


(vi) How does the writer justify that "failing to prepare is preparing to fail!"? 1
HE

Ans. The writer explains this line by saying that this principle is very important for making
presentations. It is clear that if one fails to prepare well, then he/she is heading towards
failure and will ultimately face defeat.
OT

2. Read the passage given below.


India is a part of the worldwide pandemic of COVID-19. The first case of COVID-19 in
BR

India, which originated from China, was reported on 30 January 2020. India currently has the
highest number of confirmed cases in Asia,[ and has the second-highest number of confirmed
cases in the world after the US, with more than 9 million reported cases of COVID-19
infection and more than 100 thousand deaths. By mid of 2020, India had approached in a
L

position of conducting the highest number of daily tests in the world which subsequently
YA

translated into highest number of daily new cases in the world and has sustained highest
number of daily cases spike since then.
On 22 March, India observed a 14-hour voluntary public curfew at the insistence of Prime
O

Minister Narendra Modi. It was followed by mandatory lockdowns in COVID-19 hotspots and
G

all major cities. Further, on 24 March, the prime minister ordered a nationwide lockdown for
21 days, affecting the entire 1.3 billion population of India. On 14 April, India extended the
nationwide lockdown till 3 May which was followed by two-week extensions starting 3 and
17 May with substantial relaxations. From 1 June, the government started “unlocking” the
country (barring “containment zones”) in three unlock phases.
In March, after the lockdown was imposed, the United Nations (UN) and the World
Health Organization (WHO) praised India’s response to the pandemic as ‘comprehensive
Score Plus CBSE Sample Question Paper with MTPs in English Language & Literature for Class 10 (Term II) 95
GOYAL A DIGITAL LEARNING APP
BROTHERS
PRAKASHAN
Learn @ ` 1 Per Day

and robust’, terming the lockdown restrictions as ‘aggressive but vital’ for containing the
spread and building necessary healthcare infrastructure.. Also in March, Michael Ryan,
chief executive director of the WHO’s health emergencies program noted that India had
tremendous capacity to deal with the outbreak owing to its vast experience in eradicating
smallpox and polio.
In July 2020, India’s Ministry of Information and Broadcasting claimed the country’s case
fatality rate was among the lowest in the world at 2.41% and was “steadily declining”. By
mid-May 2020, six cities accounted for around half of all reported cases in the country

N
– Mumbai, Delhi, Ahmedabad, Chennai, Pune and Kolkata. As of 10 September 2020,

HA
Lakshadweep is the only region which has not reported a case.
The pandemic has left a severe impact on Indian economy, leading to a negative growth
rate for the first time in decades. Nevertheless, the economy started to rebound after the

AS
lockdown was eased. The Union Government also launched a major self-sufficiency campaign
to substitute imported products with domestically produced counterparts, especially to replace
goods imported from China.

AK
on the basis of your understanding of the passage, answer ANY FiVE questions from
the six given below. PR 1*5
(i) Why did the UN and the WHO praise India’s response to the pandemic? 1
Ans. The UN and the WHO praised India’s response to the pandemic because imposing the
lockdown was a comprehensive, aggressive but vital step required to control the spread
RS

of the virus, and further protect the healthcare infrastructure.


(ii) List the inherent features of COVID based on your understanding of the report. 1
HE

Ans. The inherent features of COVID are that it originated in China and can also lead to
death.
(iii) Justify that the step taken to unlock the country was favourable with reference to para 2. 1
OT

Ans. The step taken to unlock the country was favourable because, from 1 June, the
government started “unlocking” the country (barring “containment zones”) in three
BR

unlock phases, and not altogether, as that could have created chaos.
(iv) What did Michael Ryan, chief executive director of the WHO's health emergencies
program, note about India while handling the situation of a crisis? 1
Ans. Michael Ryan noted that India had a tremendous capacity to deal with the outbreak
L
YA

because it had vast experience in eradicating smallpox and polio.


(v) "In July 2020, India's Ministry of Information and Broadcasting claimed the country’s
case fatality rate was among the lowest in the world." How does the evidence from the
O

report support this statement? 1


Ans. The report clearly stated that the country's case fatality rate was among the lowest in the world
G

at 2.41% and was “steadily declining”. By mid-May 2020, six cities accounted for around
half of all reported cases.
(vi) What impact did the report have on India's economy as a result of the pandemic? 1
Ans. The pandemic has had a severe impact on the Indian economy, leading to a negative
growth rate for the first time in decades.

96 Score Plus CBSE Sample Question Paper with MTPs in English Language & Literature for Class 10 (Term II)
GOYAL A DIGITAL LEARNING APP
BROTHERS
PRAKASHAN
Learn @ ` 1 Per Day

sECTioN B - WRiTiNG ANd GRAMMAR (10 marks)


3. Attempt ANY oNE from (i) and (ii).
(i) Given below is the pie chart depicting the percentage of the market that the top brands
have acquired in the Indian streaming market. Draw a comparison in the form of an
analytical paragraph on the given data. 5

N
HA
AS
AK
PR
RS

Ans. The above pie chart represents International brands rising to the top of Indian streaming
market. Due to the coronavirus pandemic, the demand for streaming has increased
HE

drastically allowing many other players to emerge.


Based on the figure, Netflix and Amazon Prime captured a 20% each of the market in Q2
of 2020, while the cooperation of Disney+ and local player Hotstar reached 17%.Players
OT

like Jio Cinema and ZEE5 were seen emerging at 7% and 9% respectively, while ALT
Balaji and SONY liv got 4% each in their share of the market.
The giants like Netflix and Amazon prime video still seem to be maintaining their hold
BR

over most of the market, while the rest of the majority is fragmented among unspecified
‘others’.
(ii) You are the Secretary of the Rock Climbing Unit of your school, MG Higher Secondary
L

School, Dehradun. Write a letter ordering a complete list of equipment needed for rock
YA

climbing shoes, jackets, ropes, helmets, etc., from the sports shop in your city. 5
Ans. M.G. Higher Secondary School
Dehradun
O

Uttaranchal
G

January 20, 20XX

Relix Sports Shop


10/2A, Indirapuram
Ghaziabad

Score Plus CBSE Sample Question Paper with MTPs in English Language & Literature for Class 10 (Term II) 97
GOYAL A DIGITAL LEARNING APP
BROTHERS
PRAKASHAN
Learn @ ` 1 Per Day

Sir
Subject: Order for Sports Materials
We want to place a bulk order of equipment of needed for such climbing at Dehradun.
Kindly send the following sports materials to the above address through transport with
proper cash bill.
The items are listed below:
1. Rock climbing shoes — 15 pairs

N
2. Jackets — 20 pairs

HA
3. Ropes — 10 pairs
4. Helmets — 15 pairs
All the items should be in good condition, well-bound and appropriately packed.

AS
Any damage to the items during transportation will be your responsibility. The items
should reach the school within ten days after receiving the letter. Otherwise, the order
shall stand cancelled.

AK
Thanking you PR
Yours faithfully
Anamika Khanna
(Secretary)
RS

4. The following paragraph has not been edited. There is one error in each line. Identify the
error and write its correction against the correct blank number. Remember to underline the
correction. The first one has been done for you. 1*3
HE

Error Correction
Children bound by marriage is victims. Why do child E.g. is are
OT

marriages take place at all and what could be done to (a)


awake the people? This evil thrives in these society because (b)
of illiteracy and other related causes—a most important (c)
BR

of which is the anxiety of parents to marry off their daughters to a student.


Ans. Error Correction
(a) might can
L

(b) these this


YA

(a) a the
5. Read the given conversation between a doctor and patient, Mayuri, and complete the passage
O

that follows. 1*2


Mayuri: May I come in?
G

doctor: Yes, sit down. What is the problem?


Mayuri: I have a stomachache.
doctor: What have you taken in breakfast?
Mayuri: I had bread and milk.

98 Score Plus CBSE Sample Question Paper with MTPs in English Language & Literature for Class 10 (Term II)
GOYAL A DIGITAL LEARNING APP
BROTHERS
PRAKASHAN
Learn @ ` 1 Per Day

Yesterday, Mayuri went to a doctor. Mayuri asked the doctor respectfully if she might come
in. The doctor asked her to come and sit down and further asked (a) ________. Mayuri
answered that she had a stomachache. The doctor (b) ________ in breakfast. Mayuri replied
bread and milk.
Ans. (a) what the problem was
(b) enquired what she had taken

sECTioN C - LiTERATuRE (20 marks)

N
6. Answer ANY siX questions in 30-40 words each. 2*6

HA
(i) What are other remains of the Portuguese days and culture still found in Goa? 2
Ans. Although the Portuguese have left India, those old, time-tested furnaces still exist. The

AS
Portuguese traditions are very much alive in Goa that it feels like the fire in the furnaces
has not yet been extinguished. One can hear the thud and jingle of the traditional baker’s
bamboo, announcing his arrival in the morning.

AK
(ii) Natalya repeatedly insisted that Oxen Meadows are theirs and told Lomov that she
considered Lomov a good neighbour and friend. This excited Lomov to the point of
PR
verbal fighting. What does this show about their character? 2
Ans. This shows that both Lomov and Natalya are immature. They are quarrelsome and rude.
They have a habit of arguing over petty issues. Both are argumentative, full of pride and
possessiveness. They are ready to argue for a piece of land instead of the important things.
RS

(iii) According to you, what is the age of Amanda? What was she asked by her mother? 2
Ans. Amanda was merely a teenage girl facing common adolescent issues with her mother.
HE

Amanda’s mother stopped her from biting nails, asked her to keep her back straight
while sitting, avoid eating chocolates and later, not to be sad and moody.
OT

(iv) How are animals different from humans about owning things or kneeling to one another? 2
Ans. Humans are obsessed with material possessions. Animals, on the other hand, are
unconcerned about possessions, and it is only humans that find it insane. Humans kneel
BR

in front of others, but animals do not. All are equal in their eyes.
(v) ‘Appearances are deceptive’ is an apt summation of the poem. Elucidate. 2
Ans. Despite his ferocious appearance, Belinda’s pet dragon appeared to be a coward at heart.
L

The other pets brag about their bravery, ability to chase lions, and tiger screams, while
YA

the dragon whimpers for a safe cage to hide in. While the other creatures scurried away
to hide, the dragon measured up to the pirate’s threats by snorting at him, clattering his
O

steely tail, and finally eating him up.


(vi) What do you think of M. Loisel as a husband? Cite instances from the text to support
G

your answer. 2
Ans. Monsieur Loisel is a devoted, if not entirely submissive, husband. He desires to
accomplish anything his wife desires or whatever it takes to make her happy. He
foregoes his own comfort in favour of hers. He agrees to give her the money he has set
aside for himself when she asks to buy a dress for the ball.

Score Plus CBSE Sample Question Paper with MTPs in English Language & Literature for Class 10 (Term II) 99
GOYAL A DIGITAL LEARNING APP
BROTHERS
PRAKASHAN
Learn @ ` 1 Per Day

(vii) The narrator was happy though he had not found Lutkins. Why? 2
Ans. The narrator despised city living. He was overjoyed at the prospect of riding through the
village. He was ecstatic to meet the hack driver. Although he had not discovered Lutkins,
he was content.
7. Answer ANY TWo of the following in about 120 words each. 4*2
(i) Why was Rajvir excited when visiting the tea garden at Dhekiabari? How did he
describe the tea garden? 4

N
Ans. Pranjol’s father was the manager of a tea garden at Dhekiabari. It was greenery all
around. Clusters of tea-pluckers, who hung bamboo baskets on their backs, were plucking

HA
the newly sprouted leaves. Rajvir had never visited a tea garden in his life before, so he
was very excited when he got a chance to see one at Dhekiabari in Assam.
Rajvir described the tea garden at Dhekiabari by saying that there was a road of gravel

AS
where tea bushes were cut neatly. When they visited the tea garden, it was the second
sprouting period of the tea plantation. Women wearing plastic aprons were plucking new

AK
tea leaves and putting them in the bamboo baskets.
(ii) What kind of a person is Valli? Illustrate your answer from the text that you have read. 4
Ans. Valli was a young, self-confident girl. She wanted a bus ride to the town. So, she
PR
overcame every temptation to save money. Valli had diligently saved many spare coins
that came her way, resisting the urge to buy peppermints, toys, or balloons. She had
finally saved sixty paise. It was not easy for her, particularly at the village fair, where
RS

she was tempted to ride the merry-go-round because she had the money. She resisted
her great desire and put the money aside for the bus ride. She was a determined girl
HE

with a commanding nature. She was also inquisitive. She didn’t like to be called a child.
She got irritated whenever somebody called her a child. She was also a self-respecting
girl. She declined the conductor’s offer of cold drink. She was also a sensitive girl. She
OT

couldn’t bear the sight of the dead cow and felt very sad.
(iii) ‘Failure is the stepping stone to success.’ Elucidate with reference to Ebright and his
struggles. 4
BR

Ans. You must have heard that “Success is the fruit of failure”. One surely reaps the sweet
result of failure if one has experienced failure. Like Richard, every success comes after
failure and disappointments. Although he was an eccentric scientist with mind-blowing
L

knowledge and skills, he could not prove his experiments early in his life. After a few
YA

failures in his life, he realised the importance of success for himself. He had strengthened
his willpower to succeed. In grade seven, he participated in the county science fair with
his slides of frog tissues. This experiment did not get him any prizes. He was upset yet
O

determined to focus on his next experiment. The very next year, he participated in the
Science Fair when he was in grade eight. This time, he turned the tables. He worked
G

hard and made a project with his friend, who was also a scientist. They built a device
that showed the gold spots of a pupa which are responsible for the growth of butterfly
wings. This project made them win first prize in the county Science Fair and third prize
in zoology in the International Science and Engineering Fair. Ebright’s overall incidents
and journey of his life teach us great lessons. We must aspire to him, stay motivated and
remain positive in our lives.

100 Score Plus CBSE Sample Question Paper with MTPs in English Language & Literature for Class 10 (Term II)
GOYAL A DIGITAL LEARNING APP
BROTHERS
PRAKASHAN
Learn @ ` 1 Per Day

N
HA
AS
AK
PR
RS
HE
OT
BR
L
YA
O
G

27
BROTHERS
PRAKASHAN
GOYAL A DIGITAL
Learn @LEARNING
` 1 Per DayAPP
BROTHERS
PRAKASHAN
Learn @ ` 1 Per Day

N N
HA HA
AS AS
AK AK
PR PR
RS RS
HE HE
OT OT
BR BR
L L
YA YA
O O
G G

To view Sample or Purchase of Books


WWW.goyal-books.com/mtpcx
To view Sample or Purchase of Books
7R YLHZ 6DPSOH RU 3XUFKDVH RI %RRNV 8
www.goyal-books.com/mtpcx
To view Sample or Purchase of Books
:::JR\DOERRNVFRPPWSF[
www.goyal-books.com/mtpcx
28

To view Sample or Purchase of Books
G

WWW.goyal-books.com/mtpcx
To view Sample or Purchase of Books
7R YLHZ 6DPSOH RU 3XUFKDVH RI %RRNV 8
www.goyal-books.com/mtpcx
To view Sample or Purchase of Books
:::JR\DOERRNVFRPPWSF[
www.goyal-books.com/mtpcx

28
GOYAL A DIGITAL LEARNING APP
BROTHERS
PRAKASHAN
Learn @ ` 1 Per Day

MODEL TEST PAPER–11 (Unsolved)


(Based on the latest CBsE sample Paper)

Time allowed: 2 Hours Maximum Marks: 40

General Instructions:
1. The Question Paper contains THREE sections-READING, WRITING & GRAMMAR and

N
LITERATURE.
2. Attempt questions based on specific instructions for each part.

HA
sECTioN A - REAdiNG (10 marks)

AS
1. Read the passage given below.

All around me, I see people romanticising the “old times”. The old times were good, they were

AK
peaceful; the old times were happy, they were simpler. My mother would tell tales of the old
times, when world was simpler, people were kinder, there was no social platform to make you
feel bad about yourself, and the technology wasn’t sucking everyone in. People in old times still
PR
appreciated the nature around them, children played outside, greed wasn’t so blunt, cities were
not overpopulated, and skirts were just a fashion; not a subject of debate.

She intricately painted a picture of a world that was much better than the one I lived in. Sitting
RS

on the counter-top of our marbled kitchen slab, I listened avidly to many stories of her ‘old
times’. That was a good time too.
HE

It’s amazingly bemusing how time just passes us by, it flies for sure, but we crawl in the moment,
burdened by the yearning for the past, never realising that this moment is all we have and it is
soon going to wither. By the time fog clears, we forget the lesson we’ve learnt. We again yearn
OT

for the simpler times, but in the world that is so fast paced, how is one supposed to hold onto a
moment anyway? I think you just need to live in it. It is destined to become a memory so why
BR

not make sure that your old times were good?

I still remember my grandparent’s house in the capital. It had a long balcony that faced a park,
a tall banyan tree stood at a distance, towering even the third floor of the nearest flats. I have
L

many good memories from that time, but, and this is peculiar, curiously, I remember the smells
YA

the most vividly. Sunlight against the leaves on a summer afternoon, the chilly breeze of a spring’s
evening, the dairy shop and the milk canister and even the token machines that dispelled milk,
my grandfather’s cigarette’s smoke, the main park after it had rained, the markets in the evening,
O

the smell of certain houses we went past when my grandfather took me out for a walk, all these
smells constitute my childhood.
G

I always loved the idea of going back to those “old times”. That was until one day I smelled the
similar smells around my own house, and noticed that there was a similar tall tree, not banyan
though, towering even the third floor of the flats in front of ours, it’s leaves rusting beautifully
along the breeze of a spring evening. The epiphany tip-toed right into my heart. How have I not
noticed this tree before? I’ve been letting time pass me by again. At least this time, I am well
aware to value it.
Score Plus CBSE Sample Question Paper with MTPs in English Language & Literature for Class 10 (Term II) 101
GOYAL A DIGITAL LEARNING APP
BROTHERS
PRAKASHAN
Learn @ ` 1 Per Day

Based on your understanding of the passage, answer ANY FiVE questions from the six
given below. 1*5
(i) What does the writer mean by "burdened by the yearning for the past"? 1
(ii) Rewrite the following sentence by replacing the underlined phrase with a word that
means the same from the last paragraph. 1
I experienced a sudden realisation that girls were always treated differently than boys.
(iii) Which elements were added to the author’s olfactory senses (sense of smell) during her
stays in her grandparents’ house? 1

N
(iv) Why has the writer stressed the fact that old times were good? 1
(v) “By the time the fog clears, we forget the lesson we’ve learned.” Write what lesson we

HA
learn from this statement. 1
(vi) How does the writer justify the statement that he/she had never noticed the tree before? 1

AS
2. Read the passage given below.
The graph below illustrates India’s age distribution in 2018.

AK
India age breakdown (2018)
30%
27.0%
26.0%
PR
25%
21.8%
20%
Percentage

RS

15.5%
15%
HE

10
7.7%

5%
1.7%
OT

0.3%
0%
under 15 15–29 30–44 45–59 60–74 75–84 85 and
over
Age (range)
BR

The above graph illustrates India’s age breakdown in the year 2018. It extends from under 15
age group to 85 and over, and skims over 7 different age ranges. Doing so, the graph provides
L

a sneak into India’s age distribution for the given year. It is well known that India is a young
YA

and able nation, a country that provided skilled personnel to many first-world countries and top
tech and otherwise industries. But how old is this rich and dense population?
Apparently, under 15 age group is the one with the highest number of nationals, at 27%. Whereas
O

the range of 15-29 is the second-highest as it constitutes 26% of the nation’s population. Next
up is the range of 30-44 which accounts for 21.8% of the total population. Together these three
G

groups made up for more than half of the population of India. The age group of 45-59 years
had 15.5% nationals fall into it. While there were only 7.7% of 60-74 years old people. Only
1.7% of the remaining individuals shared the age range of 75-84 and the rest 0.3% were either
85 or above.
Looking at the data, it is clear that most of the population constitutes of teens and young adults.
This could provide India a scope for growth led by the young generation in the coming years.

102 Score Plus CBSE Sample Question Paper with MTPs in English Language & Literature for Class 10 (Term II)
GOYAL A DIGITAL LEARNING APP
BROTHERS
PRAKASHAN
Learn @ ` 1 Per Day

Having more youth in the working class, in the coming years, could propel India’s growth
upwards and help it harness the mentality and strength of the youngsters, giving the nation and
its working a fresher perspective it needs to catch up with the first world nations.
on the basis of your understanding of the passage, answer ANY FiVE questions
from the six given below. 1*5
(i) What does India need that the younger population could provide? 1
(ii) Comment on the aspect that India’s young population ensures its growth in the future. 1
(iii) Justify the statement “...it needs to catch up with the first world nations.” 1

N
(iv) According to the above passage, what has leaked out of India into the First World

HA
countries? 1
(v) According to the given data, there are more teens and young adults in the population.
How does the evidence from the graph support this statement? 1

AS
(vi) What fact do you understand about India's population from the given graph? 1

sECTioN B - WRiTiNG ANd GRAMMAR (10 marks)

AK
3. Attempt ANY oNE from (i) and (ii).
(i) Below is a graph given showing birth and death rates in a country from 1901 to 2101.
PR
Write an analytical paragraph in about 120 words. 5
RS
HE
OT
BR

(ii) You are Raman/Anisha of Gandhi Nagar, Gujarat. You saw an advertisement about a
course in spoken English, published in “The Times of India” by Super Institute, Gujarat.
L

You wish to join the course. Write a letter to the advertiser, inquiring about the details
YA

you require. 5
4. The following paragraph has not been edited. There is one error in each line. Identify the
error and write its correction against the correct blank number. Remember to underline the
O

correction. The first one has been done for you. 1*3
G

Error Correction
Service to mankind are service to God. There is E.g. are is
not an single religion which does not preach the (a)
ideal of service to humanity. What is directly give to man (b)
is directly give to God. One dare remember this. (c)

Score Plus CBSE Sample Question Paper with MTPs in English Language & Literature for Class 10 (Term II) 103
GOYAL A DIGITAL LEARNING APP
BROTHERS
PRAKASHAN
Learn @ ` 1 Per Day

5. Read the conversation between Ankit and John and complete the passage that follows. 1*2
Ankit : John! Why didn’t you come to school yesterday?
John : I was not well.
Ankit : What happened to you?
John : I was running high fever. I am fine now.
Ankit : Do not exert yourself today.
Ankit inquired John (a) _______________. John replied that he was not well. Ankit again

N
inquired what had happened to him. John explained that he had been running high fever and
he was fine then. Ankit advised not (b) _______________.

HA
sECTioN C - LiTERATuRE (20 marks)
6. Answer ANY siX questions in 30-40 words each. 2*6

AS
(i) What did the bakers wear in the Portuguese days, and when the author was young? 2
(ii) Notice the use of the word ‘turn’ in the first line, “I think I could turn and live with

AK
animals…”. What is the poet turning from? 2
(iii) How old do you think Amanda is? How do you know this? 2
(iv) Why did Custard cry for a nice safe cage? Why is the dragon called “cowardly dragon”? 2
PR
(v) Which qualities are common in all three characters of the play ‘The Proposal’? 2
(vi) Do you think Richard’s mother was too indulgent when she bought him cameras,
telescopes, microscopes? Give two reasons to support your answer. 2
RS

(vii) Mention two things you would have done, other than what M Loisel did, to help resolve
the problem of the lost necklace. 2
HE

7. Answer ANY TWo of the following in about 120 words each. 4*2
(i) The author describes the things that Valli sees from an eight-year-old’s point of view.
Can you find evidence from the text for this statement? 4
OT

(ii) You and your friend have a conversation about prevalent discrimination between boys
and girls, even today. Write the dialogue. 4
BR

You may begin like this:


Friend: I don’t think discrimination in terms of gender is a thing of the past.
You: I also agree with you. I see such a discrimination everywhere.
Friend: Hmmm. Yes, I agree; though we are in the 21st century, it still continues in a
L

blatant manner in many parts of the country.


YA

(iii) What does Chubukov at first suspect that Lomov has come for? Is he sincere when he
later says “And I’ve always loved you, my angel, as if you were my own son”? Find
reasons for your answer from the play. 4
O
G

104 Score Plus CBSE Sample Question Paper with MTPs in English Language & Literature for Class 10 (Term II)
GOYAL A DIGITAL LEARNING APP
BROTHERS
PRAKASHAN
Learn @ ` 1 Per Day

N
HA
AS
AK
PR
RS
HE
OT
BR
L
YA
O
G

27
BROTHERS
PRAKASHAN
GOYAL A DIGITAL
Learn @LEARNING
` 1 Per DayAPP
BROTHERS
PRAKASHAN
Learn @ ` 1 Per Day

N N
HA HA
AS AS
AK AK
PR PR
RS RS
HE HE
OT OT
BR BR
L L
YA YA
O O
G G

To view Sample or Purchase of Books


WWW.goyal-books.com/mtpcx
To view Sample or Purchase of Books
7R YLHZ 6DPSOH RU 3XUFKDVH RI %RRNV 8
www.goyal-books.com/mtpcx
To view Sample or Purchase of Books
:::JR\DOERRNVFRPPWSF[
www.goyal-books.com/mtpcx
28

To view Sample or Purchase of Books
G

WWW.goyal-books.com/mtpcx
To view Sample or Purchase of Books
7R YLHZ 6DPSOH RU 3XUFKDVH RI %RRNV 8
www.goyal-books.com/mtpcx
To view Sample or Purchase of Books
:::JR\DOERRNVFRPPWSF[
www.goyal-books.com/mtpcx

28
GOYAL A DIGITAL LEARNING APP
BROTHERS
PRAKASHAN
Learn @ ` 1 Per Day

MODEL TEST PAPER–12 (Unsolved)


(Based on the latest CBsE sample Paper)

Time allowed: 2 Hours Maximum Marks: 40

General Instructions:
1. The Question Paper contains THREE sections-READING, WRITING & GRAMMAR and

N
LITERATURE.
2. Attempt questions based on specific instructions for each part.

HA
sECTioN A - REAdiNG (10 marks)

AS
1. Read the passage given below.
Many of us are clueless about what we wish to pursue in our life. Some of us want to be an

AK
engineer, some maybe a doctor, some might even want to be a teacher. However, few of us
think about an unconventional career. We want to be successful, undoubtedly. But, is success
synonymous to a professional degree that can earn us a fat cheque?
PR
We should probably begin by asking ourselves a few questions. The mantra of choosing a
career is simple — we will have to be at that job for the rest of our lives. So can we spend
time at it without losing interest?
RS

For instance, a friend quits an extremely lucrative job at a multinational corporate firm
because he was weary of the drudgery that it entailed. One day, when I called upon him, he
was baking a cake. He seemed delighted to be doing that. I had never seen him so happy. I
HE

asked him what the reason was behind that infectious smile of his. He said that baking made
him happy and sometimes, he had the urge to just quit his job, sit at home and bake. The
OT

only thing that was holding him back was the fear that probably his cakes would not really
sell like hot cakes. What I didn’t expect from him was that he would take that risk. After
quitting his job, he started by taking a few online orders. The journey wasn’t a piece of cake.
BR

He had a fair share of his struggles, but what kept him going was perhaps the happiness and
contentment he drew from his work.
The problem with many of us is that we are unwilling to put ourselves through the trouble of
L

trying new things. Could that be because of our fear of the unknown? Are we afraid to take
the road less travelled?
YA

Certainly, if we take a path that has not been explored well, we will have to make a way for
ourselves. There may not be many people to guide us through and we might find ourselves
O

lost too. Nonetheless, what we should remember is that the joy of carving our niche and
finding our way would be unmatched. All we need to do is understand what makes us happy,
G

figure out a way that will lead us towards our aim and then, work towards it relentlessly. We
will have to be not only consistent, but also persistent and extremely optimistic. That is what
will make all the difference.
Based on your understanding of the passage, answer ANY FiVE questions from the six
given below. 1*5
(i) What does the writer refer to as "a piece of cake"? 1

Score Plus CBSE Sample Question Paper with MTPs in English Language & Literature for Class 10 (Term II) 105
GOYAL A DIGITAL LEARNING APP
BROTHERS
PRAKASHAN
Learn @ ` 1 Per Day

(ii) Rewrite the following sentence by replacing the underlined phrase with a word that
means the same from para 3. 1
People want to get away from the donkey work of their everyday professional lives.
(iii) Are success and a professional degree synonymous? What questions must one ask oneself
to answer this question? 1
(iv) Why and what “are we afraid to take the road less travelled”? 1
(v) List one likely impact that was holding back the writer's friend after deciding on baking
cakes after quitting his job. 1

N
(vi) How does the writer justify the statement "…pursue a career that gives them happiness and
contentment…"? 1

HA
2. Read the passage given below.
The diagnosis and treatment of leprosy is easy and effective. The best way of preventing

AS
disabilities associated with it, as well as preventing further transmission, lies in early diagnosis
and treatment. Since 1983 the disease has been curable with multidrug therapy, and since 1985
this therapy has been made available by the World Health Organization (WHO) free of charge

AK
around the world. The greatest barriers to eliminating the disease are ignorance and stigma.
Information campaigns about leprosy in endemic areas are of supreme importance so that
people affected by leprosy and their families – historically ostracized from their communities
PR
– come forward and receive treatment. The campaign stressed that leprosy is curable, that
drugs to cure it are available free throughout India, and that people affected by leprosy should
not be excluded from society. The central messages of the campaign were:
• Leprosy is not hereditary.
RS

• Leprosy is not caused by bad deeds in previous life.


• Leprosy is not spread by touch.
HE

The campaign used 50 television and 213 radio programmes in 20 languages, and 85 000
information posters. More than 1700 live drama shows, 2746 mobile video screenings, and
OT

3670 public events or competitions were performed in remote areas. Independent market
surveys conducted before, during, and after the campaign found:
• Reach of the media campaign. The radio and TV spots were seen by 59% of
BR

respondents, or 275 million people.


• Transmissibility and curability: The proportion of people who believed leprosy was
transmitted by touch fell from 52% to 27%. The proportion believing that people with
L

leprosy who take multidrug therapy are still infectious fell from 25% to 12%. Those who
YA

knew that leprosy was curable rose from 84% to 91%.


• symptoms: Awareness that loss of sensation could be a possible symptom of leprosy
rose from 65% to 80%. Awareness of pale reddish patches as a possible symptom
O

remained unchanged at 86%. Awareness of non-itchy patches as a possible symptom rose


from 37% to 55%.
G

• Therapies: The awareness rate in control villages (not covered in the campaign) that
multidrug therapy was a cure for leprosy was only 56%, but in villages that had been
shown live drama, it was 82%. In rural areas, awareness that the treatment was free was
89% among those exposed to the poster campaign, against 20% in those not exposed.
• stigma: The proportion of people saying they would be willing to sit next to a person
affected by leprosy was 10% higher in villages where drama shows had been used than
in those without. Similarly, the proportion of those claiming they would be willing to
106 Score Plus CBSE Sample Question Paper with MTPs in English Language & Literature for Class 10 (Term II)
GOYAL A DIGITAL LEARNING APP
BROTHERS
PRAKASHAN
Learn @ ` 1 Per Day

eat food served by somebody affected by leprosy was 50% in villages covered by the
campaign, against 32% in those not covered.
on the basis of your understanding of the passage, answer ANY FiVE questions
from the six given below. 1*5
(i) What factors does the campaign state when it says that "leprosy is curable"? 1
(ii) Comment on the significance of the media’s involvement in the country's leprosy control
campaign with reference to para 1. 1
(iii) Justify the researchers’ recommendation that conducting information campaigns about

N
leprosy in endemic areas is of supreme importance with reference to para 1. 1

HA
(iv) What does the market survey reveal about the village people? 1
(v) The findings of the independent market surveys while spreading awareness about the
symptoms of leprosy proved very beneficial. How does the evidence from the report

AS
support this statement? 1
(vi) According to the survey, what is the most powerful driving force behind the success of
the campaign? 1

AK
sECTioN B - WRiTiNG ANd GRAMMAR (10 marks)
3. Attempt ANY oNE from (i) and (ii). PR
(i) The diagram below shows the growth factors and challenges in the ready-to-eat food
market. Study the diagram carefully and write a paragraph analyzing the factors. 5
RS

Increase in the number


of food delivery apps
HE

Fast-paced lifestyle among


Ease of availability at
working professionals and
supermarkets and malls
students
OT

Companles compets over


More women joining the market share by crafting
workforce and increased lucrative acts and offers
family consumption Growth drivers
BR

Complex food distribution Challenges Frequent consumption of RTE


system owing to many food products might lead to high
small stakeholders in the blood pressure and cholesterol
L

supply chain
YA

Considerable food The belief that preservations


wastage due to lack of used in the RTE products reduce
proper storage facilities nutritional value
O
G

(ii) You are Ali/Aliya teaching at Central Public School, Lucknow. On behalf of your school,
you need to place an order for the laboratory equipment needed by the Geography
department. You feel that the quality check should be done before the material is
supplied. Write a letter to the Assistant Manager of Global Equipment P. Ltd, 1, Ashok
Road, Lucknow requesting him to send the globes, maps and rainwater harvesting 3d
model at the earliest. 5

Score Plus CBSE Sample Question Paper with MTPs in English Language & Literature for Class 10 (Term II) 107
GOYAL A DIGITAL LEARNING APP
BROTHERS
PRAKASHAN
Learn @ ` 1 Per Day

4. The following paragraph has not been edited. There is one error in each line. Identify the
error and write its correction against the correct blank number. Remember to underline the
correction. The first one has been done for you. 1*3
Error Correction
Today, we all am familiar with the importance of E.g. am are
fingerprints in identifying the culprit. The recovery of partial
fingerprints from a crime scene is a important method of (a)
forensic science. Moisture and grease on a finger results (b)

N
in fingerprints on surfaces such as glass or metal.

HA
Human fingerprints is detailed, nearly unique, and difficult to alter. (c)
5. Read the given conversation between Ram and Sita and complete the passage given below. 1*2
Rama: I am going to the jungle.

AS
sita: Why are you going to the jungle?
Rama: I have been banished for fourteen years by Father.

AK
sita: Oh! It is very sad.
PR
On being banished by his father, Rama told Sita (a) _____________ to the jungle. She asked
him why he was going to the jungle. Rama told that (b) ________________ for fourteen years
by Father. Sita exclaimed sorrowfully that it was very sad.
RS

sECTioN C - LiTERATuRE (20 marks)


6. Answer ANY siX questions in 30-40 words each. 2*6
HE

(i) What was the baker’s role in the author’s childhood? 2


(ii) How did Chubukov react when Lomov said he had come to ask for his daughter's hand? 2
(iii) After reading the poem 'Amanda', what impression do you form about Amanda? 2
OT

(iv) What does the poet mean by "Not one is respectable or unhappy over the whole earth"? 2
(v) Sometimes, situations demand prompt actions like bravery and courage. Express your
BR

views, referring to the poem 'The Tale of Custard the Dragon'. 2


(vi) What all hobbies did Ebright develop in kindergarten? 2
(vii) What do you think were Mme Forestier’s thoughts when she found that Matilda had
2
L

given her a diamond necklace instead of an artificial one?


YA

7. Answer ANY TWo of the following in about 120 words each. 4*2
(i) After reading the story ‘A Baker from Goa’, do you think our traditions, heritage, values
and practices are the roots that nourish us? Why/why not? 4
O

(ii) Timely help and sympathy are great help for those who are in grief. Elaborate this
statement, referring to the lesson ‘The Sermon at Benaras’. 4
G

(iii) ‘Love is blind’ is a phrase that Monsieur Loisel justified in the lesson. Substantiate the
answer from the story ‘The Necklace’. 4

108 Score Plus CBSE Sample Question Paper with MTPs in English Language & Literature for Class 10 (Term II)
GOYAL A DIGITAL LEARNING APP
BROTHERS
PRAKASHAN
Learn @ ` 1 Per Day

N
HA
AS
AK
PR
RS
HE
OT
BR
L
YA
O
G

27
BROTHERS
PRAKASHAN
GOYAL A DIGITAL
Learn @LEARNING
` 1 Per DayAPP
BROTHERS
PRAKASHAN
Learn @ ` 1 Per Day

N N
HA HA
AS AS
AK AK
PR PR
RS RS
HE HE
OT OT
BR BR
L L
YA YA
O O
G G

To view Sample or Purchase of Books


WWW.goyal-books.com/mtpcx
To view Sample or Purchase of Books
7R YLHZ 6DPSOH RU 3XUFKDVH RI %RRNV 8
www.goyal-books.com/mtpcx
To view Sample or Purchase of Books
:::JR\DOERRNVFRPPWSF[
www.goyal-books.com/mtpcx
28

To view Sample or Purchase of Books
G

WWW.goyal-books.com/mtpcx
To view Sample or Purchase of Books
7R YLHZ 6DPSOH RU 3XUFKDVH RI %RRNV 8
www.goyal-books.com/mtpcx
To view Sample or Purchase of Books
:::JR\DOERRNVFRPPWSF[
www.goyal-books.com/mtpcx

28
GOYAL A DIGITAL LEARNING APP
BROTHERS
PRAKASHAN
Learn @ ` 1 Per Day

MODEL TEST PAPER–13 (Unsolved)


(Based on the latest CBsE sample Paper)

Time allowed: 2 Hours Maximum Marks: 40

General Instructions:
1. The Question Paper contains THREE sections-READING, WRITING & GRAMMAR and

N
LITERATURE.
2. Attempt questions based on specific instructions for each part.

HA
sECTioN A - REAdiNG (10 marks)

AS
1. Read the passage given below.
Not long ago, I came across a film by CNBC MakeIt titled Why Finland And Denmark Are
Happier Than The U.S. To start with, Finland and Denmark featured in the World Happiness

AK
Report 2019 as the top-two happiest countries in the world—so it is justified to just call Finnish
and Danish people as “happy;” right?
PR
In the above film, people (natives and otherwise) try to explore, or more so explain, why they
might be the happiest, which was informative if not highly exciting to find out. As someone,
whose country ranked at 140 in the same report, I was awestruck by the simplicity that
powers the ability of these people to live their lives. Government and the care they offer to
RS

the individuals was also a big factor, no doubt. But as you watch on, the same people mention
the dark side of this happiness. Wait, happiness is dark?
HE

Basically, Danish and Finnish people may be less prone to the sadness that takes seed
in poverty or corruption but as people, they are also vulnerable to the harshness of life,
especially stress. When a young girl from Copenhagen explained how the very same lifestyle
OT

that offers them productivity and richness can also drive them to wear out, I realised why
they termed happiness as dark, suddenly, it was not ironical at all. It was also mentioned how
feeling blue in a society that is rose-coloured may make someone feel out of place or even
BR

worse about themselves sometimes.


It is understandable how much pressure one may feel when their country is coined as the
happiest worldwide. It may even give way to making depression a taboo amongst people who
L

are otherwise seen as the happiest, which I hope is not going to happen. I can also grasp why
YA

the world needs to know which countries are the “happiest”. It can help immigrants decide
where to move, create traction in the country’s economy by inviting more skilled personnel
and reassure natives that their system is still functioning efficiently.
O

My concern is, shall we really call such countries as the “Happiest”? Maybe we can use kinder
synonyms like “Most Liveable”. Think about it—Finland is the Most Liveable! Absolutely!
G

Happiest? Well, that’s a subjective opinion hence not really any of our business. Because
happiness is not so easy to define; it’s a feeling. Yes, but it is also an effect and a cause.
(Source: thelunasol.org)
Based on your understanding of the passage, answer ANY FiVE questions from the six
given below. 1*5
(i) What does the writer refer to by using the phrase “making depression a taboo”? 1
Score Plus CBSE Sample Question Paper with MTPs in English Language & Literature for Class 10 (Term II) 109
GOYAL A DIGITAL LEARNING APP
BROTHERS
PRAKASHAN
Learn @ ` 1 Per Day

(ii) Rewrite the following sentence by replacing the underlined phrase with a word that
means the same from para 3. 1
At the end of the day, he looks at the world with optimistic and idealistic glasses.
(iii) State two reasons why the world needs to know which countries are the “happiest”. 1
(iv) "There is a dark side of happiness." Why is happiness termed as dark? 1
(v) List one reason why Finland and Denmark are known as the happiest. 1
(vi) How does the writer justify that Finnish and Danish people are still prone to the severity of
life? 1

N
2. Read the passage given below.

HA
The main purpose of the market survey is to offer marketing and business managers a platform
to obtain critical information about their consumers so that existing customers can be retained
and new ones can be got onboard.

AS
How is a survey done? By examining qualitative answers for telling quotes and Interrogate
quantitative answers to calculate averages. Compare results against global and local data with
large-scale market surveys.

AK
How Relevant Personalization Drives Business Growth And Customer Value
1. KEY FiNdiNGs : Customers expect relevant e-commerce experiences while brands
PR
struggle to deliver. Customer expectations are on the rise. In fact, customers expect
brands to make investments about them. Forrester found that 64% of customers expect
e-commerce brands to invest in technology that makes their communications more
RS

relevant. Brands also believe that delivering relevant and personalized experiences are
high-priority initiatives, and 83% believe personalization will be more important to their
e-commerce and marketing efforts, but they fall short in execution.
HE

2. E-commerce brands risk alienating customers by failing to personalize experiences.


E-commerce organizations must align to the needs and expectations of their customers.
Brands face a paradox when they recognize that failure to optimize personalization
OT

efforts will negatively impact their bottom line, but they don’t feel comfortable pursuing
greater investment. Despite the fact that 77% of brands believe personalization creates a
BR

better customer experience, 58% say they are reluctant to further invest in personalization
because they believe building or enhancing their personalization efforts would require too
many resources.
3. Personalization in e-commerce drives better business outcomes. Customers reward
L

relevant and personalized experiences with greater loyalty, more purchases, and deeper
YA

engagement. The majority of surveyed customers (56%) say they are more likely to
spend more with brands that tailor experiences for them, and 68% say they are more
likely to join a loyalty program with a brand that delivers a relevant and personalized
O

experience. In short, brands that deliver relevant and personalized experiences will be
best poised to stand out from their competition.
G

on the basis of your understanding of the passage, answer ANY FiVE questions
from the six given below. 1*5
(i) Who falls short of execution when the writer says, “...they fall short in execution..."? 1
(ii) Comment on the significance of the detailed feedback about the attitudes and behaviours
of the customers. 1

110 Score Plus CBSE Sample Question Paper with MTPs in English Language & Literature for Class 10 (Term II)
GOYAL A DIGITAL LEARNING APP
BROTHERS
PRAKASHAN
Learn @ ` 1 Per Day

(iii) Justify the statement in reference to the customers' viewpoint—"Forrester found that
64% of customers expect e-commerce brands to invest in technology that makes their
communications more relevant" with reference to Point 1 of the Key Findings. 1
(iv) 58% of brands say they are reluctant to further invest in personalization. What is meant
by “personalization”? 1
(v) “Personalization in e-commerce drives better business outcomes.” How does the evidence
from the survey support this statement? 1
(vi) According to the survey, what percentage of customers is likely to join a loyalty

N
programme with a brand that delivers a relevant and personalized experience? 1

HA
sECTioN B - WRiTiNG ANd GRAMMAR (10 marks)
3. Attempt ANY oNE from (i) and (ii).

AS
(i) Read the following excerpt from an article that appeared in an online magazine:
Academic achievement was once thought to be the most important outcome of formal
educational experiences and while there is little doubt as to the vital role such achievements

AK
play in student life and later, researchers and policy makers are ever increasingly turning to
social and emotional factors, as well as the relationships among them, as indicators of student
well-being and psychological development. PR
Write a paragraph to analyse the given argument.
You could think about what alternative explanations might weaken the given conclusions
and include rationale / evidence that would strengthen / counter the given argument. 5
RS

(ii) As Raman/Ramya, Librarian, Arya Bharti School, Noida, write a letter to Durable
Furnishers, Ghaziabad, placing an order for some furniture for the school library. 5
HE

4. The following paragraph has not been edited. There is one error in each line. Identify the
error and write its correction against the correct blank number. Remember to underline the
1*3
OT

correction. The first one has been done for you.


Error Correction
La Tomatina are the largest food fight festival in the E.g. are is
BR

world which is hold in the Valencian town of Bunol, (a)


Spain on the last Wednesday of August some year. (b)
This festival requires the visitors to throw
L

approximately 40,000 tomatoes and got involved in (c)


YA

this tomatoes' fight just for fun every year.


5. Read the conversation between Joseph and Vijay and complete the passage that follows. 1*2
Joseph: Hey, Vijay! What a pleasant surprise to see you here!
O

Vijay: Hi, Joseph! I feel happy to see you here.


G

Joseph: That’s a nice shirt!


Vijay: Thank you. My brother gifted this to me.
Joseph: Wow! I must say your brother has a nice choice.

The other day, I met Vijay. On seeing him, I exclaimed that (a) ________. He told me that he
too was happy to see me there. When I complimented him for his shirt, he informed me that
(b) ___________. I remarked that his brother had a nice choice.
Score Plus CBSE Sample Question Paper with MTPs in English Language & Literature for Class 10 (Term II) 111
GOYAL A DIGITAL LEARNING APP
BROTHERS
PRAKASHAN
Learn @ ` 1 Per Day

sECTioN C - LiTERATuRE (20 marks)


6. Answer ANY siX questions in 30-40 words each. 2*6
(i) Why has Coorg been referred to as ‘a piece of heaven drifted from the kingdom of
God’? 2
(ii) Why was Kisa Gotami depressed? What did she do to overcome her grief? 2
(iii) Amanda is a child, and she does not like to be controlled. What does this show about
children? 2
(iv) The poet says that no animal is unhappy. What does this mean? 2

N
(v) The poet feels that there is no point in calling the dragon, a brave animal. Why? 2

HA
(vi) What lesson did Ebright learn when he did not win anything at a science fair? 2
(vii) M. Loisel over-pampered his wife and readily accepted her demands. What does this
show about M. Loisel? 2

AS
7. Answer ANY TWo of the following in about 120 words each. 4*2
(i) Describe the childhood memories of the author’s time in Goa and his fondness for bread

AK
and cakes. 4
(ii) “Just think, what a misfortune I've had.” Who is the speaker? What is the misfortune that
is being referred to? How does this statement trigger another argument? Take reference
from the play ‘The Proposal’.
PR 4
(iii) ‘Bill made me sound very important, and the woman was impressed.’ 4
(a) Why did this particularly gladden the narrator’s heart?
RS

(b) What does it tell us about human nature and the young man in particular?
HE
OT
BR
L
YA
O
G

112 Score Plus CBSE Sample Question Paper with MTPs in English Language & Literature for Class 10 (Term II)
GOYAL A DIGITAL LEARNING APP
BROTHERS
PRAKASHAN
Learn @ ` 1 Per Day

N
HA
AS
AK
PR
RS
HE
OT
BR
L
YA
O
G

27
BROTHERS
PRAKASHAN
GOYAL A DIGITAL
Learn @LEARNING
` 1 Per DayAPP
BROTHERS
PRAKASHAN
Learn @ ` 1 Per Day

N N
HA HA
AS AS
AK AK
PR PR
RS RS
HE HE
OT OT
BR BR
L L
YA YA
O O
G G

To view Sample or Purchase of Books


WWW.goyal-books.com/mtpcx
To view Sample or Purchase of Books
7R YLHZ 6DPSOH RU 3XUFKDVH RI %RRNV 8
www.goyal-books.com/mtpcx
To view Sample or Purchase of Books
:::JR\DOERRNVFRPPWSF[
www.goyal-books.com/mtpcx
28

To view Sample or Purchase of Books
G

WWW.goyal-books.com/mtpcx
To view Sample or Purchase of Books
7R YLHZ 6DPSOH RU 3XUFKDVH RI %RRNV 8
www.goyal-books.com/mtpcx
To view Sample or Purchase of Books
:::JR\DOERRNVFRPPWSF[
www.goyal-books.com/mtpcx

28
GOYAL A DIGITAL LEARNING APP
BROTHERS
PRAKASHAN
Learn @ ` 1 Per Day

MODEL TEST PAPER–14 (Unsolved)


(Based on the latest CBsE sample Paper)

Time allowed: 2 Hours Maximum Marks: 40

General Instructions:
1. The Question Paper contains THREE sections-READING, WRITING & GRAMMAR and

N
LITERATURE.
2. Attempt questions based on specific instructions for each part.

HA
sECTioN A - REAdiNG (10 marks)

AS
1. Read the passage given below.
There came a time when I began to find things in my daughter’s pockets—crayons or money.
I chanced upon a packet of biscuits in the drawer of her study desk. I stayed calm. It’s all

AK
right, all kids steal. “It is normal for a very young child to take something which excites his
or her interest,” Google confirmed in 0.27 seconds. Yet there was the unmistakable soundtrack
of panic galloping towards me. Despite my highfalutin decisions to rewrite the family script, I
PR
had to be doing something exactly like my parents for my child to be behaving as I did at her
age. I walked into the park nearby to breathe out a silent scream.
A while ago, I read Amy Chua’s Battle Hymn of the Tiger Mother, where she explained
RS

how Chinese parents produced successful kids. I was hooked and amused by the bluntness,
clarity, self-parody and wry humour. I shared the article online. That is when I began to
HE

realize the enormity of what this piece was doing to its readers. It was dredging up anger,
fear, self-doubt, judgment and passionate counter-arguments.
Describing how she pushed her seven-year-old Lulu to master a piano piece, Chua wrote: I
OT

threatened her with no lunch, no dinner, no Christmas or Hanukkah presents, no birthday


parties for two, three, four years. When she still kept playing it wrong, I told her she was
purposely working herself into frenzy because she was secretly afraid she could not do it. I
BR

told her to stop being lazy, cowardly, self-indulgent and pathetic.


Chua clarified in interviews that her book was the story of her own eventual transformation as
a mother. At first glance, it may appear that the parent in the trenches with her kid is doing
L

all the hard work, and the lenient ones are lazy. It may seem that the ‘tiger mother’s’ kids are
YA

soaring, while others are still playing in the mud, their potential unrealized. The truth is that
it is easy to be the ‘tiger’. It is the ‘mother’ part that demands courage, as Chua discovered.
Parents make mistakes, they are vulnerable. They learn to back off and cede territory.
O

Parents need permission to fail without feeling like failures. This parenting business is a
G

complex web. We source the design from deep subconscious wells, from our memory and
experience. We repeat patterns from our own childhood. We are the agents of our culture. If
Chua decided that her daughters would play instruments and excel academically, this was a
function of her background—she was the child of immigrants, in single-minded pursuit of
praise, excellence and admiration in America. She had no clue about the value of fun and
games. Most Indians will recognize the type of tiger mom Chua is. The word love was never
used in Chua’s childhood home.

Score Plus CBSE Sample Question Paper with MTPs in English Language & Literature for Class 10 (Term II) 113
GOYAL A DIGITAL LEARNING APP
BROTHERS
PRAKASHAN
Learn @ ` 1 Per Day

When I was twelve, I pasted an article in my diary, titled, ‘The Greatest Gift you can Give
your Child: Self-Esteem’. I don’t think I knew what self-esteem was, but I must have wanted
it badly, because we were not allowed to cut out pages from Reader’s Digest. I now know
that self-confidence is not something anyone can give you to keep forever. It is like a valley
of flowers—it must be discovered again and again.
Excerpted from My daughters’ Mum: A book of permissions to love, laugh, heal
and find one’s way home by Natasha Badhwar, relaunched by Simon & Schuster
India in November 2018.

N
Based on your understanding of the passage, answer ANY FiVE questions from the six
given below. 1*5

HA
(i) What does the writer mean by using the phrase ‘cede territory’ for the parents? 1
(ii) Rewrite the following sentence by replacing the underlined phrase with a word that
1

AS
means the same from para 2.
I was appalled by the immensity and extent of the task ahead of us.
(iii) State any two reasons how Chua's book was the story of her own eventual transformation

AK
as a mother. 1
(iv) Why is it normal for a child to steal or take things? 1
(v) Chua’s daughter was repeatedly making mistakes. What one thing did Chua ask her
PR
daughter to do? 1
(vi) How does the writer justify the statement, "Parents need permission to fail without feeling
like failures."? 1
RS

2. Read the passage given below.


Rolling Returns show how stock markets perform during good and bad times.
HE

80.000%
S&P 500 Index
Rollling Returns
OT

60.000%

40.000%
BR

20.000%

0.000%
1 Year 3 Year 5 Year 10 Year 15 Year 20 Year
L

–20.000%
YA

–40.000%
O

–60.000%
S&P 500 Index Best Return S&P 500 Index Worst Return
G

The given bar graph depicts Rolling Returns based on S&P 500’s performance in the stock
market. Rolling Returns draw out how stock markets perform during good and bad times,
above it does so by illustrating the best and worst return on S&P 500 index. S&P 500
(short for Standard & Poor) is a stock market index that measures the stock performance of
500 large companies listed on stock exchanges in the United States. It is one of the most
commonly followed equity indexes.

114 Score Plus CBSE Sample Question Paper with MTPs in English Language & Literature for Class 10 (Term II)
GOYAL A DIGITAL LEARNING APP
BROTHERS
PRAKASHAN
Learn @ ` 1 Per Day

Within the period of 1 year, the index performs its best at 60% and its worst at -40%. In the
next 3 years, it scores a bit below its previous high, that is, 40% but its worst also goes down
as it scores a little less than -20%. Next 5 years see a drop in its best performance, when the
index manages to reap a little less than it did previously, while its worst performance, in a
way, improves as it drops even less than before. Next 10 years yield a little under 20%. As
for the worst side of it, the index seems a lot closer to 0%.
The following 15 years look good when looking at the worst part of its performance, the
index, even at its worst, reaps a little more than 0%, while at its best, the index reaps

N
somewhat same as before. Within next 20 years, index’s worst performance improves and it
yields between 5-10%,. while its best performance plummets a little more. It’s clear that S&P

HA
500 index has declined in returns over the years yet its worst performance has improved,
making it a safer option with lesser returns.
on the basis of your understanding of the passage, answer ANY FiVE questions

AS
from the six given below. 1*5
(i) What do the researchers mean by "yield"? 1
(ii) List any two inherent qualities of the Rolling Returns. 1

AK
(iii) Justify the statement, "The following 15 years look good when looking at the worst part
of its performance" with reference to the last paragraph. 1
(iv) What does the Rolling Returns indicate over the long-time in regards to its performance? 1
PR
(v) "Next 5 years see a drop in its best performance." How does the evidence from the
graphical representation support this statement? 1
(vi) What does the graph depict? 1
RS

sECTioN B - WRiTiNG ANd GRAMMAR (10 marks)


HE

3. Attempt ANY oNE from (i) and (ii).


(i) The chart below displays data about the boys and girls engrossed in various cultural and
leisure activities. Write a paragraph analyzing the given data. 5
OT

Boys’ Cultural and Leisure Activities Girls’ Cultural and Leisure Activities
Basketball
BR

26%
Gymnastics
11%
Soccer Dancing
17% 27%
Listening to
L

Music Reading
21%
YA

10%
Reading
2%
Skateboarding Listening to
11% Music
10%
O

Computer Netball Computer


Games
G

Games 15%
34% 16%

(ii) You are the Resource Manager of AVS Ltd. Your company is coming up with a new
branch office. You met the head of Vidyut Electricals, a store in the nearby market and
discussed the price quotations of various electrical appliances for the new office. Your
company has approved the quotation and you have been asked to place the order. Write a
letter placing the order. Sign yourself as Savita Rai / Samarth Sharma. 5
Score Plus CBSE Sample Question Paper with MTPs in English Language & Literature for Class 10 (Term II) 115
GOYAL A DIGITAL LEARNING APP
BROTHERS
PRAKASHAN
Learn @ ` 1 Per Day

4. The following paragraph has not been edited. There is one error in each line. Identify the
error and write its correction against the correct blank number. Remember to underline the
correction. The first one has been done for you. 1*3
Error Correction
Lakshadweep, the coral paradise of India, has the E.g. has is
tiniest but most fascinating union territory of India.
Marco Polo, in his travelogue, refers to Minicoy (a)

N
as “an island of females”, perhaps due to the majority of (b)
its inhabitants being females and her predominant role (c)

HA
in every walk of life.
5. Read the conversation between a couple and complete the passage that follows. 1*2
Mom: Have you heard from our son lately?

AS
dad: Nope.
dad: I left him a message to see if he is coming to dinner this weekend, but he hasn’t called

AK
back.

Mom asked Dad if (a) _________ from their son lately. Dad replied that he (b) _________ if
PR
he was coming to dinner that weekend, but he hadn’t called back.

sECTioN C - LiTERATuRE (20 marks)


RS

6. Answer ANY siX questions in 30-40 words each. 2*6


(i) Rajvir was thrilled and amazed all through the way to Assam. Do you agree? 2
HE

(ii) Why did Valli not get off the bus when it was terminated at the bus stand? 2
(iii) ‘There is a languid, emerald sea.’ What does this indicate? Refer to the poem ‘Amanda!’. 2
(iv) ‘Do not complain, but accept the situation’ is the key to happiness. Elaborate it in the
OT

context of the poem ‘Animals’. 2


(v) ‘Bravery is a spur-of-the-moment.’ Justify the statement with reference to the poem. 2
(vi) What were the factors which contributed to making Ebright a scientist? 2
BR

(vii) Do you find the chapter’s elements hilarious, fun, and entertaining? Cite examples with
reference to the chapter ‘The Hack Driver’. 2
7. Answer ANY TWo of the following in about 120 words each. 4*2
L

(i) Did Lutkins do right with the lawyer? What securities should be taken to shun a
YA

situation like this? 4


(ii) Pen down a brief travelogue entry, narrating any personal impression/s about a visit to
O

Coorg. Include your reflections about the differences between the place you live and the
place you’ve visited. 4
G

You may begin like this...


As I stepped into the land of evergreen forests, I was blown away by the pristine beauty
and ………………………………(continue)
(iii) Why is the marriage proposal in the play ‘The Proposal’ important to all the characters? 4

116 Score Plus CBSE Sample Question Paper with MTPs in English Language & Literature for Class 10 (Term II)
GOYAL A DIGITAL LEARNING APP
BROTHERS
PRAKASHAN
Learn @ ` 1 Per Day

N
HA
AS
AK
PR
RS
HE
OT
BR
L
YA
O
G

27
BROTHERS
PRAKASHAN
GOYAL A DIGITAL
Learn @LEARNING
` 1 Per DayAPP
BROTHERS
PRAKASHAN
Learn @ ` 1 Per Day

N N
HA HA
AS AS
AK AK
PR PR
RS RS
HE HE
OT OT
BR BR
L L
YA YA
O O
G G

To view Sample or Purchase of Books


WWW.goyal-books.com/mtpcx
To view Sample or Purchase of Books
7R YLHZ 6DPSOH RU 3XUFKDVH RI %RRNV 8
www.goyal-books.com/mtpcx
To view Sample or Purchase of Books
:::JR\DOERRNVFRPPWSF[
www.goyal-books.com/mtpcx
28

To view Sample or Purchase of Books
G

WWW.goyal-books.com/mtpcx
To view Sample or Purchase of Books
7R YLHZ 6DPSOH RU 3XUFKDVH RI %RRNV 8
www.goyal-books.com/mtpcx
To view Sample or Purchase of Books
:::JR\DOERRNVFRPPWSF[
www.goyal-books.com/mtpcx

28
GOYAL A DIGITAL LEARNING APP
BROTHERS
PRAKASHAN
Learn @ ` 1 Per Day

MODEL TEST PAPER–15 (Unsolved)


(Based on the latest CBsE sample Paper)

Time allowed: 2 Hours Maximum Marks: 40

General Instructions:
1. The Question Paper contains THREE sections-READING, WRITING & GRAMMAR and

N
LITERATURE.
2. Attempt questions based on specific instructions for each part.

HA
sECTioN A - REAdiNG (10 marks)

AS
1. Read the passage given below.
1. These days, it is not unusual to see people listening to music or using their electronic
gadgets while crossing busy roads or travelling on public transports, regardless of the

AK
risks involved. I have often wondered why they take such risks: is it because they
want to exude a sense of independence, or is it that they want to tell the world to stop
bothering them? Or is it that they just want to show how cool they are? Whether it is
PR
a workman or an executive, earphones have become an inseparable part of our lives,
sometimes even leading to tragicomic situations.
2. The other day, an electrician had come to our house to fix something. We told him in
RS

detail what needed to be done. But after he left, I found that the man had done almost
nothing. It later turned out that he could not hear our directions clearly because he had
an earphone on. Hundreds of such earphones’ addicts commute by the Delhi Metro every
HE

day. While one should not begrudge anyone their moments of privacy or their love for
music, the fact is ‘iPod oblivion’ can sometimes be very dangerous.
3. Recently, I was travelling with my wife on the Delhi Metro. Since the train was
OT

approaching the last station, there weren’t too many passengers. In our compartment,
other than us, there were only two women sitting on the other side of the aisle. And
then suddenly, I spotted a duffel bag. The bomb scare lasted for several minutes. Then
BR

suddenly, a youth emerged from nowhere and picked up the bag. When we tried to stop
him, he looked at us, surprised. Then he took off his earpieces, lifted the bag, and told
us that the bag belonged to him and that he was going to get off at the next station.
L

4. We were stunned but recovered in time to ask him where he was all this while. His
YA

answer : he was in the compartment, leaning against the door totally immersed in the
music. He had no clue about what was going on around him. When he got off, earplugs
in his hand, we could hear strains of the song.
O

Based on your understanding of the passage, answer ANY FiVE questions from the six
given below. 1*5
G

(i) What does the phrase "tragicomic situations" refer to? 1


(ii) Rewrite the following sentence by replacing the underlined phrase with a word that
means the same from para 4. 1
Renu got some books from the library and completely immersed herself in Roman history
and culture.

Score Plus CBSE Sample Question Paper with MTPs in English Language & Literature for Class 10 (Term II) 117
GOYAL A DIGITAL LEARNING APP
BROTHERS
PRAKASHAN
Learn @ ` 1 Per Day

(iii) State any two reasons why people, nowadays, risk their lives while listening to music in
public places using their electronic gadgets. 1
(iv) What kind of feeling does the expression "emerged from nowhere" create? 1
(v) How does the writer explain the consequences of iPod oblivion? 1
(vi) How does the writer justify the observation, "I found that the man had done almost
nothing."? 1
2. Read the passage given below.

N
A recent report by the National Association of Software and Service Companies (NASSCOM) has
revealed that school students are spending 50 per cent more time online than they would have

HA
before COVID-19 hit. The sudden boom of EdTech firms has also encouraged other corporates
and start-ups to expand into the education sector.
A nationwide lockdown was announced in India on 24 March 2020. As a result, schools were

AS
closed till further notice and the classes for the new session were held online. Since then, students
have explored many avenues of learning.

AK
The students are now spending 30 per cent more time on education apps which actually led to
over 120 per cent growth in the investments on the promotion of EdTech apps. A total of 88
per cent rise in the number of people downloading EdTech apps has also been recorded.
PR
But the challenge that India has faced since the beginning of the lockdown is the low penetration
of internet connectivity and errant electricity. This problem is prevalent across not just remote
areas but even tier 2 and 3 cities. Additionally, the teachers and schools have not been very
RS

adept in handling even basic tech. But things are changing, over the past few months, teachers
and parents have adopted newer technologies and have developed a level of comfort with the
HE

“new normal”. Education is not confined to the text anymore, it is more about its application in
reality. Our skills and learnings have been put to test by these trying times.
Democratising online education is an absolute necessity to bridge the gap and to penetrate the
OT

interiors of the country. There is also a dearth of educational tools and content in vernacular
languages. The EdTech sector must work towards creating tools, services and content that are
BR

easily accessible and affordable to a larger section of the society.

Category-wise split of online education market in India.


Market size – 2016 Market size – 2021
L

Test
Preparation
YA

Test Reskilling and online 27%


Preparation certifications Reskilling and online
17% 38% Higher certifications
Education 23%
9%
O

Higher 2016 2021P


Education
13% Language and
G

casual learning Language and


2% casual learning
Primary and 1%
Primary and Secondary Secondary
supplemental education supplemental
30% education
40%

118 Score Plus CBSE Sample Question Paper with MTPs in English Language & Literature for Class 10 (Term II)
GOYAL A DIGITAL LEARNING APP
BROTHERS
PRAKASHAN
Learn @ ` 1 Per Day

on the basis of your understanding of the passage, answer ANY FiVE questions
from the six given below. 1*5
(i) What has been the major challenge faced by India since the beginning of the lockdown? 1
(ii) Which company's report revealed that school students are spending 50 per cent more
time online than they would have before COVID-19 hit? 1
(iii) Justify the statement, "Many companies are expanding into the EdTech sector due to the
sudden boom," with reference to para 3. 1
(iv) According to the research, write one of the necessities for bridging a gap in the interiors

N
of the country. 1
(v) According to the survey conducted by KPMG research and analysis, the language and

HA
casual learning market have the least share in the Indian online market in 2016 as well
as in 2021. How does the evidence from the pie chart support this statement? 1
(vi) How much of the market share of primary and secondary supplemental education in the

AS
overall online education market across India has been estimated? 1

sECTioN B - WRiTiNG ANd GRAMMAR (10 marks)

AK
3. Attempt ANY oNE from (i) and (ii).
(i) Here is a pie chart showing how youngsters prefer to access internet at home and other
PR
places. Write a paragraph analyzing the given data. 5
RS
HE
OT
BR

(ii) Write a letter on behalf of Kamal Associates to Write Well Stationery Mart, Mumbai
asking for the quotation of prices for supplying office stationery at your office. Inquire
L

about the concession on bulk orders and commission if any, along with the mode of
5
YA

payment. Sign yourself as their Senior Resource Manager.


4. The following paragraph has not been edited. There is one error in each line. Identify the
error and write its correction against the correct blank number. Remember to underline the
O

correction. The first one has been done for you. 1*3
Error Correction
G

David is an influential piece among renaissance sculptures


creates by the Italian artist, Michelangelo. David was E.g. creates created
originally commissioned as one of the series of statue (a)
of prophets to be positioned along the roofline of
the Florence Cathedral, but is instead placed (b)
in a public square outside there Palazzo Vecchio. (c)
Score Plus CBSE Sample Question Paper with MTPs in English Language & Literature for Class 10 (Term II) 119
GOYAL A DIGITAL LEARNING APP
BROTHERS
PRAKASHAN
Learn @ ` 1 Per Day

5. Read the given conversation between a mother and son and complete the passage that follows. 1*2
Mother: Why are you late from school, Nitin?
Nitin: While coming from school to the bus stop, I slipped by stepping on a banana peel.
Mother: Did you hurt yourself?
Nitin: I got a bad bruise on my left knee.

Mother asked Nitin (a) ___________________. Nitin replied that while coming from school to
the bus stop, (b) ___________________ by stepping on a banana peel. Mother was shocked to

N
hear that.
sECTioN C - LiTERATuRE (20 marks)

HA
6. Answer ANY siX questions in 30-40 words each. 2*6
(i) Rajvir did his study before his visit to the tea plantation. 2

AS
Is it good to do one’s research before the start of a new venture, or does it take away
from the thrill of discovery? Elucidate your stance.
(ii) Do you think being enlightened placed a far greater responsibility on the 2

AK
Buddha than being king would have? Justify your stance.
(iii) What do you think is the significance of the exclamation mark in the title of the poem
2
‘Amanda!’?
PR
(iv) A misanthrope is a person who dislikes humankind. Do you think the poet is a
misanthrope? Provide a rationale to support your response. 2
(v) Why is it fair to say that Custard could be the ‘poster-boy’ for the belief that the real
RS

nature of a person is revealed at times of the greatest difficulty? 2


(vi) Imagine Matilda got to know that M Loisel had given her four hundred francs, which he
HE

had saved to buy a gun. How do you think she would have reacted? 2
(vii) Oliver Lutkins had to be served the summons to appear as a witness in a law case. Why
do you think he had ignored all their letters thus far and successfully thwarted their
OT

attempt at serving summons? 2


7. Answer ANY TWo of the following in about 120 words each. 4*2
(i) Travelling teaches us many life lessons and enriches us with experiences. You and your
BR

classmate discuss this statement and your love for travel. 4


Write the dialogue. You may begin like this...
Friend: I think travelling is one of the best ways to explore the world around you.
L

You: You are right. Travelling also helps you introspect about life itself.
YA

Friend: Travelling enriches us with so many varied experiences.


(ii) Though the play revolves around three people, Russian society emerges as an important
character through the ongoings of the play and the thoughts and interactions of its
O

characters. 4
Imagine yourself to be a historian who has been invited to speak about nineteenth-
G

century upper-class Russian society.


Based on your reading of the play, how might you describe it? Support your evaluation
with instances from the text.
(iii) After reading Bholi’s story, you decide to write a blog on the importance of educating
the girl child and empowering her. 4
Write that blog expressing your views.

120 Score Plus CBSE Sample Question Paper with MTPs in English Language & Literature for Class 10 (Term II)
GOYAL A DIGITAL LEARNING APP
BROTHERS
PRAKASHAN
Learn @ ` 1 Per Day

N
HA
AS
AK
PR
RS
HE
OT
BR
L
YA
O
G

27
BROTHERS
PRAKASHAN
GOYAL A DIGITAL
Learn @LEARNING
` 1 Per DayAPP
BROTHERS
PRAKASHAN
Learn @ ` 1 Per Day

N N
HA HA
AS AS
AK AK
PR PR
RS RS
HE HE
OT OT
BR BR
L L
YA YA
O O
G G

To view Sample or Purchase of Books


WWW.goyal-books.com/mtpcx
To view Sample or Purchase of Books
7R YLHZ 6DPSOH RU 3XUFKDVH RI %RRNV 8
www.goyal-books.com/mtpcx
To view Sample or Purchase of Books
:::JR\DOERRNVFRPPWSF[
www.goyal-books.com/mtpcx
28

To view Sample or Purchase of Books
G

WWW.goyal-books.com/mtpcx
To view Sample or Purchase of Books
7R YLHZ 6DPSOH RU 3XUFKDVH RI %RRNV 8
www.goyal-books.com/mtpcx
To view Sample or Purchase of Books
:::JR\DOERRNVFRPPWSF[
www.goyal-books.com/mtpcx

28

You might also like